10 Seconds Questions Set

You might also like

Download as pdf or txt
Download as pdf or txt
You are on page 1of 218

10 Seconds

Directions again turning to his left he walks for 60


metres. He then turns to his left & walks
Basics for 80 metres. How far is he from his initial
position?
1)Four main directions - North, South,
East, West A. 100 metres B. 60 metres
2) Four Cardinal Direction - North-East, C. 20 metres D. 120metres
North-West, South-East, South-West
E. 140 meters
3)At the time of sunrise, if a man stands
facing the east, his shadow will be towards Answer & Explanation
the west.
4)At the time of sunset, the shadow of an
object is always in the east.
5)If a man stands facing the North, at the
time of sunrise his shadow will be towards
his left and at the time of sunset, it will be
towards his right.
6)At 12:00 noon, the rays of the sun are
vertically downward hence there will be no
shadow.
Questions:
3)Varun drove his car for 80 kms due
1)Ajay walks 24 km towards East and North. Then he turned left and drove for
turns to right hand side and takes a drive 100 kms. Again he turned left & drove yet
of another 10 km. He then turning to his another 80 kms. Again he turned left and
right (drives towards West) another 10 km. drove his car 120 kms. How far do you
He then turns to his left & walks another 8 think he actually drove his car from the
km. After that, he turns to his right & initial position?
travels 14 km. How far is he from his initial
point & in which direction? A. 20 kms B. 100 kms
A. 20 km East B. 18 km south C. 60 kms D. None of these
C. 16 km West D. 10 km South
Answer & Explanation
Answer & Explanation

4)Sandeep walks 60m to the east, then he


2)Raju walks 80 m towards south. Then, turns left and walks for 50 m, then turns
turns to his right & starts walking straight right and went 70 m and then turns right
till he completes another 80 m. Then,

pg. 1
10 Seconds

again and went 50 m. How far was In the evening as sun is in the west, so the
Sandeep from the starting point? shadows fall towards east. Now, since
Manju's shadow fell towards right,
A. 90 m B. 70 m therefore, Manju is facing south. So, Sanju
C. 50 m D. 130 m standing face to face with Manju was
facing north.
Answer & Explanation
7)In the evening, Ashmita started walking
positioning his back towards the sun. After
sometime, she turned left, then turned
right and then towards the left again. In
which direction is she going now?
A. North B. East
C. West D. South
Answer & Explanation

5)One morning after sunrise, Amrit was


standing facing a pole. The shadow of the
pole was forming on the left side. Which
direction was Amrit facing?
A. East B. West
C. North D. South
Answer & Explanation
Thus, Ashmita is finally moving towards
North.
8)I drove 50 km towards east from a city
‘S’ and then turned right and drove
another 30 km. Now I turned to my left &
drove another 30 km. Finally I turned my
right & drove 30 km to reach a city ‘F’.
Find the shortest straight distance
between cities S and F.
A. 20 km B. 25 km
Sun always rises in the east . So in
C. 30 km D. 40 km
morning, the shadow falls towards the
west .now Pole's shadow falls to the left of E. 100 km
Amrit. So, he is standing, facing North.
Answer & Explanation
6)One evening just before sunset two
friends Sanju and Manju were talking to
each other face to face. If Manju’s shadow
was exactly to her left side, which direction
was Sanju facing?
A. North B. South
C. West D. Data inadequate
Answer & Explanation

pg. 2
10 Seconds

Answer & Explanation

Clearly by Pythagoras theorem SF =


100km
9)Ahmedabad is to the southwest of
Bangalore, Chennai is to the east of 11)If South-East becomes North, North-
Ahmedabad and southeast of Bangalore East becomes West and so on. What will
and Delhi is to the north of Chennai in line West become?
with Ahmedabad-Bangalore. In which A) North-East B) North-West
direction of Bangalore is Delhi located?
C) South-East D) South-West
A. South B. Southwest
Explanation:
C. North D. Northeast
E. East
Answer & Explanation

12)Rasik walked 20 m towards north.


Then he turned right and walks 30 m.
Then he turns right and walks 35 m. Then
he turns left and walks 15 m. Finally he
turns left and walks 15 m. In which
direction and how many metres is he from
the starting position?

Delhi is located in Northeast of Bangalore. A) 15 m West B) 30 m East

10)Ajay drives 3 kilometres North. Then C) 30 m West D) 45 m East


he turns right and drives 4 kilo metres. Explanation:
Now he turns right and drives 5
kilometres. Now turning left, he drives 2
kilometres. Again, he turns left and moves
2 kilometres. Finally turning left he again
walks 2 kilometres. In which direction &
how far is he from his starting position?
A. 1.5 kilometres East
B. 4 kilometres East
C. 1.5 kilometres West
D. 4.5 metres West

pg. 3
10 Seconds

A) South-West B) South-East
C) North-East D) North-West
Explanation:

16) One morning after sunrise, Suresh


was standing facing a pole. The shadow of
the pole fell exactly to his right. To which
direction was he facing?
A) East B) West
13)K is 40 m South-West of L. If M is 40 m
South-East of L, then M is in which C) South D) Data is inadequate
direction of K?
Explanation:
A) East B) West
Sun rises in the east in the morning. Since
C) North-East D) South the shadow of Suresh falls to his right. So
he is facing South.
Explanation:
17) A man walks 5 km toward south and
then turns to the right. After walking 3 km
he turns to the left and walks 4 km. And
then he goes back 10 km straight. Now in
which direction is he from the starting
place ?
A) South-East B) North-West
14) Rahul put his timepiece on the table in C) South D) West
such a way that at 6 P.M. hour hand
points to North. In which direction the Explanation:
minute hand will point at 9.15 P.M. ? From the given directions, now he is 1km
A) South-East B) South in the North-West direction.

C) North D) West 18) Two cars start from the opposite


places of a main road, 150 km apart. First
Explanation: car runs for 25 km and takes a right turn
and then runs 15 km. It then turns left and
then runs for another 25 km and then
takes the direction back to reach the main
road. In the mean time, due to minor break
down the other car has run only 35 km
along the main road. What would be the
distance between two cars at this point?
A) 65 km B) 75 km
15) If A x B means A is to the south of B; A
+ B means A is to the north of B; A % B
C) 80 km D) 85 km
means A is to the east of B; A - B means
A is to the west of B; then in P % Q + R - Explanation:
S, S is in which direction with respect to
Q?

pg. 4
10 Seconds

In the morning sun rises in the East.


Hence then any shadow falls in the West.
Since Nikharia's shadow was exactly to
the right of Nivedita. Hence Nikharia is
facing towards North.
21) Afreena walks 8 km towards East and
then walks 13 km back, then she turns left
and walks 4 km; then walks 5 km after
turning left; she turns left again and walks
3 km. How far is she from the starting
point ?
A) 3 km B) 2 km
C) 1 km D) 6 km

19) Amit started walking positioning his Explanation:


back towards the sun. After some time, he
turned left, then turned right and towards
the left again. In which direction is he
going now?
A) North or South B) North or West
C) East or West D) South or West
Explanation:

22) P started from his house towards west.


After walking a distance of 25 m. He
turned to the right and walked 10 m. He
then again turned to the right and walked
15 m. After this he is to turn right at 135o
and to cover 30 m. In which direction
If he starts walking the morning then finally
should he go?
he will face towards South and if he starts
in the evening then finally he will face A) West B) South
towards North.
C) South-West D) South-East
20) One morning after sunrise Nivedita
and Niharika were talking to each other Explanation
face to face at Dalphin crossing. If
Niharika's shadow was exactly to the right
of Nivedita, Which direction Niharika was
facing?
A) North B) South
C) East D) Data is inadequate
Explanation:
Hence he should go in the South-West
direction.
23) A child went 90 m in the East to look
for his father, then he turned right and
went 20 m. After this he turned right and
after going 30 m he reached to his uncle's

pg. 5
10 Seconds

house. His father was not there. From left ends in a theatre, straight ahead is the
there he went 100 m to his north and met hospital. In Which direction is the
his father. How far did he meet his father University?
from the starting point?
A) North B) South
A) 80 m B) 100 m
C) East D) West
C) 140 m D) 260 m
Explanation:
Explanation:
Starting from his house in the East, Ravi
moves westwards. Then, the theatre,
which is to the left, will be in the south.
The Hospital, which is straight ahead, will
be to the West. So, the University will be
to the North.

26) The length and breadth of a room are


24) A horse is facing north. It turns 90 8 m and 6 m respectively. A cat runs along
all the four walls and finally along a
degrees in the clockwise direction, then
180 degrees in the anti-clockwise and diagonal order to catch a rat. How much
then another 90 degrees in the same total distance is covered by the cat?
direction. Which direction is the horse A) 10 B) 14
facing now?
C) 38 D) 48
A) East B) South
Explanation:
C) Southwest D) Southeast
Explanation:
Starting from the initial point facing north,
the horse turns 90 degrees clockwise,
which means it turns towards East.
Similarly, the other directions can be
considered and written as : EWS Thus,
the direction it is facing now is South.
25) Ravi wants to go to the university. He
starts from his home which is in the East
and comes to a crossing. The road to the

pg. 6
10 Seconds

27) Reena walked from A to B in the East hospital. In which direction is the
10 feet. Then she turned to the right and university?
walked 3 feet. Again she turned to the
right and walked 14 feet. How far is she A) North B) South
from A? C) East D) West
A) 4 feet B) 5 feet Explanation:
C) 24 feet D) 27 feet
Explanation:

Therefore university is in North.


30)Early morning after sunrise, Karthik
was standing infront of his house in such a
way that his shadow was falling exactly
behind him. He starts walking straight and
walks 5 metres. He turns to his left and
walks 3 metres and again turning to his
left walks 2 metres. Now in which direction
is he from his starting point?
A) West B) North-East
28) Ravi started walking from his house C) East D) South-West
east direction on Bus stop which is
3km.away. Then he set off in the bus Explanation:
straight towards his right to the school 4
Early morning suns rise at east, if his
km away. what is the crow flight distance
shadow falls exactly behind him that
from his house to the school?
means he is facing towards east. now he
A) 1 km B) 5 km walks 5mt towards east, then he turns left
that means he facing towards north and
C) 7 km D) 12 km walk 3mt and if he turn left then he is
Explanation: facing towards west and he walk only 3
kms thats why man is in north east
direction from starting point.

29) Hemant in order to go to university


started from his house in the east and
came to a crossing. The road to the left
ends in a theatre, straight ahead is the

pg. 7
10 Seconds

Clocks C) 1pm D) 2pm

Basics Explanation:

1)A clock is a complete circle having 360 Time from 5 am. on a day to 10 pm. on 4th
degrees. It is divided into 12 equal parts day = 89 hours.
i.e. each part is 360/12 = 30°. Now 23 hrs 44 min. of this clock = 24
2)As the minute hand takes a complete hours of correct clock.
round in one hour, it covers 360° in 60 356/15 hrs of this clock = 24 hours of
minutes. correct clock
3)In 1 minute it covers 360/60 = 6°/ 89 hrs of this clock = (24 x 31556 x 89) hrs
minute. of correct clock.
4)Also, as the hour hand covers just one = 90 hrs of correct clock.
part out of the given 12 parts in one hour.
This implies it covers 30° in 60 minutes So, the correct time is 11 p.m.
i.e. ½° per minute.
This implies that the relative speed of the
2) At what time between 4 and 5 o'clock
minute hand is 6 - ½ = 5 ½ degrees.
will the hands of a watch point in opposite
5)We will use the concept of relative directions?
speed and relative distance while solving
A) 54 past 4
problems on clocks.
B) (53 + 7/11) past 4
6)Every hour, both the hands coincide
once. In 12 hours, they will coincide 11 C) (54 + 8/11) past 4
times. It happens due to only one such
incident between 12 and 1'o clock. D) (54 + 6/11) past 4

7)The hands are in the same straight line Explanation:


when they are coincident or opposite to 4 o'clock, the hands of the watch are 20
each other. min. spaces apart.
8)When the two hands are at a right angle, To be in opposite directions, they must be
they are 15-minute spaces apart. In one 30 min. spaces apart.
hour, they will form two right angles and in
12 hours there are only 22 right angles. It Minute hand will have to gain 50 min.
happens due to right angles formed by the spaces.
minute and hour hand at 3’o clock and 9'o
clock. 55 min. spaces are gained in 60 min

9)When the hands are in opposite 50 min. spaces are gained in (60/55)×50
directions, they are 30-minute spaces min. or 54(6/11)
apart. Required time = 54(6/11) min. past 4.
10)If both the hour hand and minute hand
move at their normal speeds, then both
the hands meet after 65 minutes. 3)Find the angle between the hour hand
and the minute hand of a clock when the
Questions: time is 3.25
1)A clock is set right at 5 a.m. The clock A) 47.5 degrees B) 57.5 degrees
loses 16 minutes in 24 hours. What will be
the true time when the clock indicates 10 C) 45.5 degrees D) 55.5 degrees
p.m. on 4th day?
Explanation:
A) 11pm B) 12pm

pg. 8
10 Seconds

At 3 O'clock, Minute hand is at 12 while C) 5 D) 7


the Hour hand is at 3. Again the minute
hand has to sweep through ( 30 x 5 ) ie Explanation:
150° for reaching the figure 5 to show 25 At 5 o'clock, the hands are 25 min. spaces
mins. apart.
Simultaneously the Hour hand will also To be at right angles and that too between
rotate for 25 mins. Thus starting from the 5.30 and 6, the minute hand has to gain
mark, 3 the hour hand will cover an angle (25 + 15) = 40 min. spaces.
= (25 x 30) / 60 = 12.5°
55 min. spaces are gained in 60 min
Hence, Angle between Hour and the
Minute hand = ( 60 - 12.5 ) = 47.5° 40 min. spaces are gained in (60/55)×40
min

4) An accurate clock shows 8 o'clock in


the morning. Through how may degrees 7) A watch gains 5 seconds in 3 minutes
will the hour hand rotate when the clock and was set right at 8 AM. What time will it
shows 2 o'clock in the afternoon? show at 10 PM on the same day ?
A) 360 B) 180 A) 10 : 27 : 41 AM B) 8 : 51 : 04 AM
C) 90 D) 60 C) 9 : 45 : 15 PM D) 10 : 23 : 20 PM
Explanation: Explanation:
Angle traced by the hour hand in 6 The watch gains 5 seconds in 3 minutes =
hours=(360/12)*6 100 seconds in 1 hour.
From 8 AM to 10 PM on the same day,
time passed is 14 hours.
5) At what time between 9 and 10 o'clock
will the hands of a watch be together? In 14 hours, the watch would have gained
1400 seconds or 23 minutes 20 seconds.
A) (49 + 3/11) min past 11
So, when the correct time is 10 PM, the
B) (49 +1/11) min past 9 watch would show 10 : 23 : 20 PM
C) (45 +1/11) min past 9 8) What time between 4 and 5 o'clock will
D) (45 +7/11) min past 11 the hands of a clock be at rightangle?

Explanation: A) 3 (5/11) B) 4 (5/11)

To be together between 9 and 10 o'clock, C) 5 (5/11) D) 6 (5/11)


the minute hand has to gain 45 min. Explanation:
spaces.
this type of problems the formulae is
55 min. spaces gained in 60 min.
(5*x + or -15)*(12/11)
45 min. spaces are gained in 60/55×45
min or 49(1/11)min Here x is replaced by the first interval of
given time
here i.e 4
6) At what time between 5.30 and 6 will
the hands of a clock be at right angles? Case 1 : (5*x + 15)*(12/11)
A) (43 + 5/11) min past 5 (5*4 +15)*(12/11)
B) (43 + 7/11) min past 5 (20+15)*(12/11)

pg. 9
10 Seconds

35*12/11=420/11=38 2/11 min. The hands are coincident at 16 + 4/11 min


past 3
Therefore they are right angles at 38 2/11
min .past4
Case 2 : (5*x-15)*(12/11) 11) How much does a watch lose per day,
if its hands coincide ever 64 minutes?
(5*4-15)*(12/11)
A) 32 8/11 B) 33 8/11
(20-15)*(12/11)
C) 34 8/11 D) 35 8/11
5*12/11=60/11 min=5 5/11min
Explanation:
Therefore they are right angles at 5 5/11
min.past4. 55min spaces are covered in 60min
60 min. spaces are covered in (60/55 x
60) min= 65+5/11 min.
9) At what time between 3 and 4 o’clock
will the minute hand and the hour hand loss in 64min=(65+5/11)- 64 =16/11
are on the same straight line but facing
opposite directions ? Loss in 24 hrs (16/11 x 1/64 x 24 x 60)
min= 32 8/11.
A) 3:15 2/8 B) 3:49
C) 3:49 1/11 D) 3:51
12) An accurate clock shows 7 o'clock in
Explanation: the morning. Through how may degrees
will the hour hand rotate when the clock
On straight line means 180 degree angle. shows 3 o'clock in the afternoon?
180 = 11/2 min – 30 hrs A) 144º B) 168º
180 = 11/2 m – 30 × 3 C) 180º D) 150º
180 = 11/2 m – 90 Explanation:
(180 + 90) 2 = 11 m Angle traced by the hour hand in 6 hours
m = 540/11 = 49 1/11 minutes. =(360/12)×6 = 180

10) At what time, in minutes, between 3 13) How many degrees will the minute
o'clock and 4 o'clock, both the needles will hand move, in the same time in which the
coincide each other? second hand move 4800 ?

A) 11 4/11 B) 13 4/11 A) 80 deg B) 160 deg

C) 15 4/11 D) 16 4/11 C) 140 deg D) 135 deg

Explanation: Explanation:

At 3 o'clock, the minute hand is 15 min. As minute hand covers, 60 degrees


spaces apart from the hour hand. Minute hand covers 4800/60 = 80°
To be coincident, it must gain 15 min.
spaces.55 min. are gained in 60 min.
14) At 6′o clock a clock ticks 6 times. The
15 min. are gained in time between first and last ticks is 30
= (60/55 x 15)min seconds. How long does it tick at 12′o
clock ?
=16+4/11
A) 66 sec B) 55 sec

pg. 10
10 Seconds

C) 36 sec D) 24 sec Here 15 minutes corresponds to 15 x 6 =


90 degrees
Explanation:
Next, you have to figure out the angle of
For ticking 6 times, there are 5 intervals. the hour hand. Since there are 12 hours in
Each interval has time duration of 30/5 = 6 the entire 360 degrees, each hour
secs corresponds to 30 degrees. But unless the
time is EXACTLY something o'clock, you
At 12 o'clock, there are 11 intervals, have to write the time as a fractional
number of hours rather than as hours and
so total time for 11 intervals = 11 x 6 = 66
minutes.
secs.
Here the time is 9:15 which is (9 + 15/60)
= 37/4 hours. Since each hour
15) What time will it be 18 hours from now corresponds to 30 degrees, we multiply 30
in India, if in the USA it is 4:15 AM? to get (37/4)(30) = 277.5 degrees.

(Hint : IST = US + 9:30 hrs) Since the hour hand is at 277.5 degrees
and the minute hand is at 90 degrees, we
A) 1:45 in the afternoon can subtract to get the angle of
B) 7:45 in the evening separation. 277.5 - 90 = 187.5 =~ 360 -
187.5 = 172.5 degrees.
C) 7:45 in the next morning
D) 1:45 in the next morning
17) How many times are the hands of a
Explanation: clock at right angle in a day?
We know that the time difference between A) 44 B) 54
India and USA is 9 hrs 30 min.
C) 64 D) 22
India is 9:30 hrs ahead of USA.
Explanation:
Time in India = 4:15 + 9:30 = 13:45 PM.
In 12 hours, they are at right angles 22
Required Time after 18 hrs = 13:45 + 18 times.
hrs = 7:45 AM.
In 24 hours, they are at right angles 44
Hence it is 7:45 AM on the next morning. times.

16) What is the angle made by the hour 18) How many degrees will the minute
hand and the minute hand, if the clock hand move, in the same time in which the
shows 9:15 pm ? second hand move 5400 ?
A) 165 degrees B) 172.5 degrees A) 90 degrees B) 85 degrees
C) 112.5 degrees D) 125.5 degrees C) 60 degrees D) 45 degrees
Explanation: Explanation:
The minute hand angle is the easiest Minute hand covers 5400/60 = 90°
since an hour (i.e. 60 minutes)
corresponds to the entire 360 degrees,
each minute must correspond to 6 19) How many times do the hands of a
degrees. So just multiply the number of clock coincide in a day?
minutes in the time by 6 to get the number
of degrees for the minute hand. A) 20 B) 21
C) 22 D) 23

pg. 11
10 Seconds

Explanation:
The hands of a clock coincide 11 times in 22) How many times in a day, are the
every 12 hours (Since between 11 and 1, hands of a clock in straight line but
they opposite in direction?
coincide only once, i.e., at 12 o'clock). A) 11 B) 22
The hands overlap about every 65 C) 33 D) 44
minutes, not every 60 minutes.
Explanation:
The hands coincide 22 times in a day.
The hands of a clock point in opposite
directions (in the same straight line) 11
times in every
20) What is the ratio of 18 minutes to one
hour ? 12 hours. (Because between 5 and 7 they
point in opposite directions at 6 o'clcok
A) 1/5 B) 3/4 only).
C) 1/7 D) 3/10 So, in a day, the hands point in the
Explanation: opposite directions 22 times.

One hour = 60 min


18/60 = 3/10 23) When the time is 5:40, then what is the
angle b/w the hour hand & the minute
hand of a clock?
21) A watch which gains uniformly is 2 A. 70° B. 60°
minutes low at noon on Tuesday and is 4
min 48 sec fast at 2 p.m. on the following C. 74° D. 80°
Tuesday. When was it correct ? Explanation:
A) 12 p.m. on Wednesday Formula used= θ=(11/2)M -30H where, H=
B) 2 p.m. on Thursday hours, M= minutes

C) 3 p.m. on Thursday H= 5 , M= 40

D) 2 p.m. on Wednesday Required angle, θ=(11/2)40 -30×5 = 70°

Explanation:
Time from 12 p.m. on Tuesday to 2 p.m. 24) What when the time is 6:32, then what
on the following Tuesday = 7 days 2 is the angle b/w the hour hand & the
hours. minute hand of a clock?

= 170 hours. A. 2° B. 4° C. 8° D. 12°

The watch gains = (2 + 4 x 4/5) min Explanation:

= 34/5 min. in 170 hrs. Formula used = θ = 30H - (11/2)M where,


H= hours, M= minutes
Now, 34/5 min are gained in 170 hrs.
H= 6 , M= 32
Then, 2 min are gained in (170 x 5/34 x 2)
hrs. Required angle, θ = 30×6 - (11/2)32 = 4°

Watch is correct after 2 days 2 hrs after 12


p.m. on Tuesday, i.e., it will be correct at 2 25) How many times do the hands of a
p.m. on Thursday. clock coincide in a day?

pg. 12
10 Seconds

A. 20 B. 21 28) A clock is set right at 1 pm. If it gains


one minute an hour, what is the true time
C. 22 D. 24 when the clock indicates 6 pm the same
Explanation: day?

The hands of a clock coincide 11 times in A. 5(7/61) min past 5


every 12 hours (since between 11 and 1, B. 55(5/61) min past 5
they coincide only once, i.e. at 12 o’
clock). The hands coincide 22 times in a C. 55(8/61) min past 5
day.
D. 56(5/61) min past 5
Explanation:
26) At what time between 1 and 2 o’ clock
will the hands of a watch makes an angle Clock gains one minute an hour. In 61
of 180° min, it shows 1 min less. In 5 hrs (300
min) it will show 300/61 min less actual
A. 35(5/11) min. past 1 time will be 6 –(300/61) i.e. 55(5/61) min
past 5 pm.
B. 40 min. past 1
C. 50(4/11) min. past 1
29) The minute hand of a clock overtakes
D. 38(2/11) min. past 1 the hour hand at intervals of 64 minutes of
Explanation: correct time. How much does the clock
gain or lose in 12 hours?
Assume that minute hand will be at 7 then
hands make an angle of 180°. it contains A. 16(5/11) min B. 16(4/11) min
7 equal parts. Each part= 30° C. 16(6/11) min D. 16(7/11) min
Total angle= 7 ×30°= 210° Explanation:
11/2° gain in 1 minutes 60 min are gained in (60/55)×60 =
210° gain in 65(5/11) min. But they are together after
=(2/11)×210=420/11=38(2/11) min. past 1 64 min.
Gain in 65 min.= 65(5/11) -64 = (16/11)m
Gain in 12 hrs = (16/11) × (12×60/64) =
27) At what time between 6 and 7 are the 180/11 =16(4/11) min.
hands of a clock 8 minutes apart?
A. 24 min past 6
30) Find the time between 3 and 4 will the
B. 21 min past 6 hands of a watch point in the opposite
direction?
C. 18min past 6
A. 49(1/11) min past 3
D. 20 min past 6
B. 49(3/11) min past 3
Explanation:
C. 49(2/11) min past 3
Between x and (x + 1) O’clock, the 2
hands will be t min apart at (5x ± t)(12/11) D. 49(4/11) min past 3
min past x. Between 6 and 7 O’clock, the
2 hands will be 8 min. apart at (5 6 –8) Explanation:
(12/11) = 264/11 =24 min past 6. Between x and (x + 1) O’clock, the 2
hands are in opposite directions at (5x +
30)(12/11) min past x.

pg. 13
10 Seconds

So, between 3 and 4, 2 hands will be in C) 1/2 D) 7/9


opposite directions at (5x+30)(12/11)
Explanation:
= (45)(12/11) = 540/11 = 49(1/11) min
past 3. Clearly, n(S) = (6 x 6) = 36.
Let E = Event that the sum is a prime
number.
Probability Then E= { (1, 1), (1, 2), (1, 4), (1, 6), (2, 1),
Basics (2, 3), (2, 5), (3, 2), (3, 4), (4, 1), (4,3),(5,
2), (5, 6), (6, 1), (6, 5) }
1)Probability is a measure of how likely an
event is to occur. n(E) = 15.

2)Probability is expressed in the form of P(E) = n(E)/n(S) = 15/36 = 5/12.


fractions = Favorable number of
outcomes/Total number of outcomes
3) In a box, there are 8 red, 7 blue and 6
3)Probability ranges between 0 and 1. green balls. One ball is picked up
randomly. What is the probability that it is
neither red nor green?
A) 1/3 B) 3/5
C) 8/21 D) 7/21

That means in a total of 5 occurrences, 2 Explanation:


times the occurrences are in my favor.
Total number of balls = (8 + 7 + 6) = 21.
4)Favorable outcome is nothing but the
Let E = event that the ball drawn is neither
condition provided to us in the question.
red nor green
= event that the ball drawn is blue.
Questions:
n(E) = 7.
1)Tickets numbered 1 to 20 are mixed up
P(E) = n(E)/n(S) = 7/21 = 1/3.
and then a ticket is drawn at random.
What is the probability that the ticket
drawn has a number which is a multiple of
3 or 5? 4) One card is drawn at random from a
pack of 52 cards. What is the probability
A) 1/2 B) 3/5 that the card drawn is a face card (Jack,
Queen and King only)?
C) 9/20 D) 8/15
A) 3/13 B) 1/13
Explanation:
C) 3/52 D) 9/52
Here, S = {1, 2, 3, 4, ...., 19, 20}.
Explanation: Clearly, there are 52 cards,
Let E = event of getting a multiple of 3 or 5
out of which there are 12 face cards.
= {3, 6 , 9, 12, 15, 18, 5, 10, 20}.
P (getting a face card) = 12/52=3/13.
P(E) = n(E)/n(S) = 9/20.

5) Two cards are drawn together from a


2) Two dice are tossed. The probability
pack of 52 cards. The probability that one
that the total score is a prime number is:
is a spade and one is a heart, is:
A) 5/12 B) 1/6
A) 3/20 B) 29/34

pg. 14
10 Seconds

C) 47/100 D) 13/102 8) Three unbiased coins are tossed. What


is the probability of getting at most two
Explanation: heads?
Let S be the sample space. A) 3/4 B) 7/8
Then, n(S) = 52C2=(52 x 51)/(2 x 1) = C) 1/2 D) 1/4
1326.
Explanation:
Let E = event of getting 1 spade and 1
heart. Here S = {TTT, TTH, THT, HTT, THH,
HTH, HHT, HHH}
n(E)= number of ways of choosing 1
spade out of 13 and 1 heart out of 13 = Let E = event of getting at most two
13C1*13C1 = 169. heads.
P(E) = n(E)/n(S) = 169/1326 = 13/102. Then E = {TTT, TTH, THT, HTT, THH,
HTH, HHT}.
P(E) =n(E)/n(S)=7/8.
6)In a class, 30% of the students offered
English, 20% offered Hindi and 10%
offered both. If a student is selected at
random, what is the probability that he. 9) Three unbiased coins are tossed.What
has offered English or Hindi ? is the probability of getting at least 2
heads?
A) 1/2 B) 3/4
A) 1/4 B) 1/2
C) 4/5 D) 2/5
C) 3/4 D) 1/3
Explanation:
Explanation:
Here S= {TTT, TTH, THT, HTT, THH,
HTH, HHT, HHH}.
Let E = event of getting at least two heads
= {THH, HTH, HHT, HHH}.
P(E) = n(E) / n(S)
= 4/8= 1/2
7) A bag contains 6 black and 8 white
balls. One ball is drawn at random. What
is the probability that the ball drawn is 10) I forgot the last digit of a 7-digit
white? telephone number. If 1 randomly dial the
final 3 digits after correctly dialing the first
A) 3/7 B) 4/7 four, then what is the chance of dialing the
C) 1/8 D) 3/4 correct number?

Explanation: A) 1/999 B) 1/1001

Let number of balls = (6 + 8) = 14. C) 1/1000 D) 4/1000

Number of white balls = 8. Explanation:

P (drawing a white ball) = 8 /14 = 4/7. It is given that last three digits are
randomly dialled. then each of the digit
can be selected out of 10 digits in 10
ways.

pg. 15
10 Seconds

Hence required probability =(1/10)3 = 1 year = 365 days . A leap year has 366
1/1000 days
A year has 52 weeks. Hence there will be
52 Sundays for sure.
11) Two brother X and Y appeared for an
exam. The probability of selection of X is 52 weeks = 52 x 7 = 364days
1/7 and that of B is 2/9. Find the
probability that both of them are selected. 366 – 364 = 2 days

A) 1/63 B) 1/14 In a leap year there will be 52 Sundays


and 2 days will be left.
C) 2/63 D) 1/9
These 2 days can be:
Explanation:
1. Sunday, Monday
Let A be the event that X is selected and B
is the event that Y is selected. 2. Monday, Tuesday

P(A) = 1/7, P(B) = 2/9. 3. Tuesday, Wednesday

Let C be the event that both are selected. 4. Wednesday, Thursday

P(C) = P(A) × P(B) as A and B are 5. Thursday, Friday


independent events: 6. Friday, Saturday
= (1/7) × (2/9) = 2/63 7. Saturday, Sunday
Of these total 7 outcomes, the favourable
12) Two dice are thrown together .What is outcomes are 2.
the probability that the sum of the number Hence the probability of getting 53 days =
on the two faces is divided by 4 or 6. 2/7
A) 7/18 B) 14/35
C) 8/18 D) 7/35 14) There are four hotels in a town. If 3
Explanation: men check into the hotels in a day then
what is the probability that each checks
Clearly, n(S) = 6 x 6 = 36 into a different hotel?
Let E be the event that the sum of the A) 1/2 B) 3/4
numbers on the two faces is divided by 4
or 6. C) 4/7 D) 3/8

Then,E = Explanation:
{(1,3),(1,5),(2,2),(2,4),(2,6),(3,1),(3,3),(3,5) Total cases of checking in the hotels = 4 x
,(4,2),(4,4),(5,1),(5,3),(6,2),(6,6)} 4 x 4 = 64 ways.
n(E) = 14. Cases when 3 men are checking in
Hence, P(E) = n(E)/n(S) = 14/36 = 7/18 different hotels = 4×3×2 = 24 ways.
Required probability =24/64 = 3/8

13)What is the probability of getting 53


Mondays in a leap year? 15) A card is drawn from a pack of 52
A) 1/7 B) 3/7 cards. The probability of getting a queen
of club or a king of heart is:
C) 2/7 D) 1
Explanation:

pg. 16
10 Seconds

A) 2/13 B) 1/13 Therefore, P(E) = n(E)/n(S) = 10/ 21.


C) 1/26 D) 1/52
Explanation: 18)If two letters are taken at random from
the word HOME, what is the probability
Here, n(S) = 52. that none of the letters would be vowels?
Let E = event of getting a queen of club or A) 1/6 B) 1/2
a king of heart.
C) 1/3 D) 1/4
Then, n(E) = 2.
Explanation:
P(E) =n(E)/n(S)=2/52=1/26.
P(first letter is not vowel) = 2/4
P(second letter is not vowel) = 1/3
16)In a simultaneous throw of pair of dice.
Find the probability of getting the total So, probability that none of letters would
more than 7. be vowels is = 2/4×1/3=1/6
A) 1/2 B) 5/12
C) 7/15 D) 3/12 19)What is the probability of getting at
least one six in a single throw of three
Explanation: unbiased dice?
Here n(S) = (6 x 6) = 36 A) 1/36 B) 91/256
Let E = event of getting a total more than C) 13/256 D) 43/256
7
Explanation:
={(2,6),(3,5),(3,6),(4,4),(4,5),(4,6),(5,3),(5, Find the number of cases in which none of
4),(5,5),(5,6),(6,2),(6,3),(6,4),(6,5),(6,6)} the digits show a '6'.
Therefore, P(E) = n(E)/n(S) = 15/36 = i.e. all three dice show a number other
5/12. than '6', 5×5×5=125 cases.
Total possible outcomes when three dice
are thrown = 216.
17)A bag contains 2 red, 3 green and 2
blue balls. Two balls are drawn at random. The number of outcomes in which at least
What is the probability that none of the one die shows a '6' = Total possible
balls drawn is blue? outcomes when three dice are thrown -
Number of outcomes in which none of
A) 10/21 B) 11/21 them show '6'.
C) 1/2 D) 2/7 =216−125=91
Explanation: The required probability = 91/256
Total number of balls = (2 + 3 + 2) = 7.
Let S be the sample space. 20)If two dice are thrown together, the
Then, n(S) = Number of ways of drawing 2 probability of getting an even number on
balls out of 7 =7C2 = 21 one die and an odd number on the other is
?
Let E = Event of drawing 2 balls, none of
which is blue. A) 1 B) 1/2

n(E) = Number of ways of drawing 2 balls C) 0 D) 3/5


out of (2 + 3) balls =5C2 = 10

pg. 17
10 Seconds

Explanation: Required probability of a coin now picked


from the box is 1 rupee = 20C11/30C11 =
The number of exhaustive outcomes is 36. 2/3.
Let E be the event of getting an even
number on one die and an odd number on
the other. Let the event of getting either 23) A fair six-sided die is rolled twice.
both even or both odd then = 18/36 = 1/2 What is the probability of getting 4 on the
first roll and not getting 6 on the second
P(E) = 1 - 1/2 = 1/2. roll ?
A) 1/36 B) 5/36
21) Two dice are thrown simultaneously. C) 1/12 D) 1/9
What is the probability of getting two
numbers whose product is even? Explanation:
A) 3/4 B) 3/8 The two events mentioned are
independent.
C) 5/16 D) 2/7
The first roll of the die is independent of
Explanation: the second roll. Therefore the probabilities
In a simultaneous throw of two dice, we can be directly multiplied.
have n(S) = (6 x 6) = 36. P(getting first 4) = 1/6
Then, E= {(1, 2), (1, 4), (1, 6), (2, 1), (2, 2), P(no second 6) = 5/6
(2, 3), (2, 4), (2, 5), (2, 6), (3, 2), (3,4),(3,
6), (4, 1), (4, 2), (4, 3), (4, 4), (4, 5), (4, 6), Therefore P(getting first 4 and no second
(5, 2), (5, 4), (5, 6), (6, 1),6, 2), (6, 3), (6, 6) = 1/6 x 5/6 = 5/36
4), (6, 5), (6, 6)}
n(E) = 27.
24)If a card is drawn at random from a
P(E) = n(E)/n(S) = 27/36 = 3/4. pack of 52 cards,what is the chance of
getting a spade or ace?
A) 0.25 B) 5/13
22)In a purse there are 30 coins, twenty
one-rupee and remaining 50-paise coins. C) 0.20 D) 4/13
Eleven coins are picked simultaneously at
random and are placed in a box. If a coin Explanation:
is now picked from the box, find the Number of spades in a standard deck of
probability of it being a rupee coin? cards=13
A) 4/7 B) 2/3 Number of aces in a standard deck of
C) 1/2 D) 5/6 cards=4

Explanation: And, one of the aces is a spade.

Total coins 30 So, 13 + 4 - 1 = 16 spades or aces to


choose from.
In that,
Therefore, probabiltiy of getting a spade or
1 rupee coins 20 an ace=16/52=4/13
50 paise coins 10
Probability of total 1 rupee coins = 20C11 25) Three houses are available in a
locality. Three persons apply for the
Probability that 11 coins are picked = houses. Each applies for one house
30C11 without consulting others. The probability

pg. 18
10 Seconds

that all the three apply for the same house 28)Out of first 20 natural numbers, one
is : number is selected at random. The
probability that it is either an even number
A) 2/9 B) 1/9 or a prime number is ?
C) 8/9 D) 7/9 A) 16/19 B) 1
Explanation: C) 3/2 D) 17/20
One person can select one house out of Explanation:
3= 3C1 ways =3.
n(S) = 20
Hence, three persons can select one
house out of 3 in 3 x 3 x 3 =9. n(Even no) = 10 = n(E)
Therefore, probability that all three apply n(Prime no) = 8 = n(P)
for the same house is 1/9
P(E U P) = 10/20 + 8/20 - 1/20 = 17/20

26)What is the probability that a leap year


has 53 Saturdays and 52 Sundays ? 29)If a box contains 10 bulbs,of which just
three are defective. If a random sample of
A) 1/7 B) 2/7 five bulbs is drawn, find the probability that
the sample contains no defective bulbs.
C) 1/2 D) 3/2
A) 5/12 B) 7/12
Explanation:
C) 3/14 D) 1/12
A leap year has 52 weeks and two days
Explanation:
Total number of cases = 7
Total number of elementary events =
Number of favourable cases = 1 10C5
i.e., {Friday, Saturday} Number of ways of selecting no defective
Required Probability = 1/7 bulbs i.e., 5 non-defective bulbs out of 7 is
7C5.
So, required probability =7C5/ 10C5 =
27)A coin is tossed 5 times. What is the 1/12.
probability that head appears an odd
number of times?
A) 1/2 B) 1/3 30)Fifteen persons are sitting around a
circular table facing the centre. What is the
C) 2/3 D) 1 probability that three particular persons sit
together ?
Explanation:
A) 3/91 B) 2/73
The possible outcomes are as follows :
C) 1/91 D) 3/73
(5H, 5T), (H, 4T), (T, 4H), (2H, 3T) (3H,
2T), i.e. 6 outcomes in all. Explanation:
Therefore the probability that head In a circle of n different persons, the total
appears an odd number of times = 3/6 number of arrangements possible = (n -
=1/2 (In only three outcomes out of the six 1)!
outcomes, head appears an odd number
of times). Total number of arrangements = n(S) =
(15 – 1)! = 14 !

pg. 19
10 Seconds

Taking three persons as a unit, total When both the pipes are opened at the
persons = 13 (in 4 units) same time, then the net part of the tank
filled in 1 hr = {(xy) / (y-x)}, provided y>x
Therefore no. of ways for these 13
persons to around the circular table = (13 - 3)If a pipe can fill a tank in x hours and
1)! = 12! can empty the same tank in y hours.
When both the pipes are opened at the
In any given unit, 3 particular person can same time, then the net part of the tank
sit in 3!. Hence total number of ways that filled in 1 hr = {(xy) / (x-y)}, provided x>y
any three person can sit =
4)Net work done = (Sum of work done by
n(E) = 12! X 3! Inlets) – (Sum of work done by Outlets)
Therefore P (E) = probability of three 5)One inlet can fill the tank in x hr and the
persons sitting together = n(E) / n(S) = 12! other inlet can fill the same tank in y hrs, if
X 3 ! / 14! both the inlets are opened at the same
= 12!x3x2 / 14x13x12! = 6/14x13 = 3/91 time, the time taken to fill the whole tank =
{(xy) / (y+x)}
6)If two pipes take x and y hours
Pipes and cisterns respectively to fill a tank of water and a
third pipe is opened which takes z hours to
Basics empty the tank, then the time taken to fill
the tank = {1 / (1/x)+(1/y)+(1/z)} and the
1)Pipes and cistern is another format of
net part of the tank filled in 1 hr =
time and work-based questions. Questions
(1/x)+(1/y)-(1/z)
like the time taken to fill or empty a tank,
the amount of work done for the same and Questions:
similar type of questions may be asked.
There are two major things which a 1)Two type of Pipes ‘A’ and ‘B’ can fill a
candidate needs to know about such tank in 3 hours and 6 hours respectively. If
questions: both the pipes are opened simultaneously,
how much time they will take to fill the
2)Inlet: An inlet is a pipe which is tank?
connected to fill a tank with water. This is
the positive type of work done a)2 hours b)3 hours

3)Outlet: An outlet is a pipe which is c)6 hours d)8 hours


connected to empty the tank of water. This
indicates a negative type of work done. It Solution:
may also be referred to as ‘leak’ in the
question Fraction method
Important Formula on Pipes and 1
Cistern capacity of ‘A’ to do work in one hour =
3

Given below are a few important formulas 1


which shall help you solve the pipes and capacity of ‘B’ to do work in one hour = 6
cistern based questions quicker and more
efficiently: A and B together can complete the work
1 1 1
= 3 +6 = 2
1)If x hours are required to fill up a tank,
then part filled in 1 hr =1/x
Therefore they require 2 hours to
If y hours are required to empty the tank, complete the work
then part emptied in 1 hour = 1/y
2)If a pipe can fill a tank in x hours and LCM method:
can empty the same tank in y hours.

pg. 20
10 Seconds

A → 3 B → 6 Capacity of Pipe B in one minute =


total work 12
time given for B
= 12 = 1
Total Work = LCM of 3 and 6 = 6
Work done by ‘A’ and ‘B’ in 3 minutes = 9
Capacity of Pipe A in one hour =
total work 6
time given for A
=3 = 2 Work remaining = 3

Capacity of Pipe B in one hour = Therefore to complete 3 units ‘B’ requires


total work 6 3
= 6=1 1
= 3 minutes
time given for B

Capacity of A = 2 means, in one hour A


can complete 2 unit of work.
3)Pipe ‘A’ can fill a tank in 5 hours while
Capacity of B = 1 means, in one hour B both ‘A’ and ‘B’ can fill in 2 hours. In how
can complete 1 unit of work. many minutes pipe ‘B’ can fill the tank,

If both A and B are working together then A. 12 minutes B. 14 minutes


they can complete 3 units of work in one
hour. C. 16 minutes D. 18 minutes

Therefore, to complete total 6 units of Solution :


6
work A and B need = 3 = 2 hours
1
Capacity of A = 5 1
NOTE:
1
Capacity of A + B = 2 2
1. Use LCM method when individual
time data is given for at least 2
Substitute 1 in 2 we get,
persons.
2. Use fraction method when 3
individual time data is not given for B = 10 = 0.3 hours or 18 minutes
at least 2 persons.

4)Two type of Pipes ‘A’ and ‘B’ can fill a


tank in 6 hours and 14 hours respectively.
2) ‘A’ and ‘B’ can fill a tank in 6 and 12 min If both the pipes are opened
respectively. Both the taps are opened simultaneously, how much time they will
simultaneously for 1 min. Now ‘A’ is take to fill the tank?
closed. How much time (in minutes) ‘B’ will a)2.2 hours b)3.2 hours
take to fill the remaining tank?
c)4.2 hours d)8.2 hours
a)2 b)3
Solution :
c)4 d)5
Total Work = LCM of 6 and 14 = 84
Solution :
Capacity of Pipe A in one hour =
total work 84
Total Work = LCM of 6 and 12 = 12 time given for A
= 6 = 14

Capacity of Pipe A in one minute = Capacity of Pipe B in one hour =


total work 12 total work 84
time given for A
= 6 =2 = 14 = 6
time given for B

pg. 21
10 Seconds

If both A and B are working together then Capacity of Pipe A in one minute =
they can complete 20 units of work in one total work 96
time given for A
= 24 = 4
hour.

Therefore, to complete total 6 units of Capacity of Pipe B in one minute =


84 total work 96
work A and B need = 20 = 4.2 hours time given for B
= 32 = 3

5)Two pipes ‘A’ and ‘B’ can fill a tank in 36 Here ‘A’ is running is running for 18
hours and 45 hours respectively. If both minutes and completes 72 units.
the pipes are opened simultaneously, how
much time in hours they will take to fill the There ‘B’ is completing 96-72 = 24 units.
tank?
24
To complete 24 units ‘B’ requires 3
=8
a)18 b)20
minutes
c)26 d)25 7) ‘A’ and ‘B’ can fill a tank in 12 and 18
min respectively. Both the taps are
Solution : opened simultaneously for 2 min. Now ‘A’
is closed. How much time (in minutes) ‘B’
Total Work = LCM of 36 and 45 = 180 will take to fill the remaining tank?

Capacity of Pipe A in one hour = a)22 b)13


total work 180
= =5
time given for A 36
c)14 d)15
Capacity of Pipe B in one hour =
total work 180 Solution :
= =4
time given for B 45
Total Work = LCM of 12 and 18 = 36
If both A and B are working together then
they can complete 9 units of work in one Capacity of Pipe A in one minute =
hour. total work 36
time given for A
= 12 = 3
Therefore, to complete total 6 units of
180 Capacity of Pipe B in one minute =
work A and B need = 9 = 20 hours total work 36
time given for B
= 18 = 2

Work done by ‘A’ and ‘B’ in 2 minutes = 10


6)Two Pipes ‘A’ and ‘B’ can fill a tank in 24
and 32 minutes respectively. If both the Work remaining = 26
taps are opened simultaneously, after how
much time ‘B’ should be closed so that Therefore to complete 26 units ‘B’ requires
tank fills in 18 minutes? 26
2
= 13 minutes

a)6 minutes b)8 minutes


8)3 pipes ‘A’, ‘B’, ‘C’ can fill a tank in 7
hours. All the pipes are opened for 2
c)4 minutes d)12 minutes hours. Now ‘C’ is closed. ‘A’ and ‘B’ fill the
remaining tank in 10 hours. Then ‘C’ alone
Solution : can fill complete tank in how many hours?

Total Work = LCM of 24 and 32 = 96 a)22 b)13

pg. 22
10 Seconds

c)14 d)15 10)Pipe ‘A’ is thrice as efficient compared


to Pipe ‘B’, therefore it takes 20 hours less
Solution : to fill the tank compared to Pipe ‘B’. How
many hours pipe ‘A’ and ‘B’ together will
Total work = 7 hours take to fill the tank,

Capacity of A + B + C =
1
1 A. 7 hours B. 7.5 hours
7
C. 8 hours D. 8.5 hours
A, B and C are completing 2 hour work.
Solution :
Time taken by A and B to complete 5 hour
work is 10 hours. Let Capacity of B = 1 and A = 3
Therefore ‘A’ and ‘B’ takes 14 hours to If A takes ‘x’ hours to fill the tank B takes
complete 7 hours work. ‘x + 20’ hours
1
Capacity of A + B = 2 Since work is same, therefore we can
14
write
1
Substituting 2 in 1, we get C = 14
3x = x + 20
Therefore ‘C’ requires 14 hours to fill the
x = 10
tank.
Therefore ‘A’ takes 10 hours and ‘B’ takes
30 hours to fill the tank.
9)One pipe is 3 times as efficient as other.
Total Work = LCM of 10 and 30 = 30
If together two pipes can fill the tank in 36
minutes, then the slower pipe alone will be
able to fill tank in? Capacity of Pipe A in one hour =
total work 30
time given for A
= 10 = 3
a)133 minutes b)144 minutes
Capacity of Pipe B in one hour =
c)155 minutes d)166 minutes total work 30
= 30 = 1
time given for B

Solution :
If both A and B are working together then
Note : Always take least efficient pipe they can complete 4 units of work in one
capacity = 1 hour.

Let Slower pipe capacity = 1 and Faster Therefore, to complete total 6 units of
30
Pipe capacity = 3 work A and B need = = 7.5 hours
4

Both of them are working for 36 minutes to


complete the work.
11)Two pipes ‘A’ and ‘B’ can fill a tank in
Total work done by them = 36 × 4 = 144 10 and 8 hours respectively. A third pipe
‘C’ can empty it in 5 hours. If all the pipes
Therefore slower Pipe takes 144 minutes are opened simultaneously, how much
to fill the tank. time (in hours) they will take to fill the
tank?

pg. 23
10 Seconds

a)120/19 b)120/18 13)Two pipes ‘A’ and ‘B’ can fill a tank in
16 hours and 12 hours respectively. The
c)120/17 d)40 capacity of tank is 240 litres. Both the
pipes are opened simultaneously and
Solution : closed after 2 hours. How much more
water is needed to fill the tank?
Total Work = LCM of 10, 8 and 5 = 40
A. 70 litre B. 170 litre
Capacity of Pipe A in one hour =
total work 40 C. 90 litre D. 190 litre
time given for A
= 10 = 4
Solution :
Capacity of Pipe B in one hour =
total work 40
= =5 Total work = LCM of 16 and 12 = 48
time given for B 8

Capacity of Pipe C in one hour = If 48 units is 240 litres then 1 unit is 5


total work 40 litres.
time given for C
= 5 =8
48
Capacity of A = 16 = 3
If both A, B and C are working together
then they can complete 1 unit of work in 48
one hour. Capacity of B = 12 = 4

Therefore, to complete total 40 units of Work done by A and B in 2 hours = 14 or


40 70 litres
work A, B and C need = = 40 hours
1

Therefore amount of water required is 240


12)A pipe can fill a tank in 9 hours and it
- 70 = 170 litres.
takes 1 hour more when there is a leak at
bottom. How much time the leak will take
to empty the tank if tank is full?

a)80 hours b)90 hours 14)Two pipes can fill a tank in 20 and 24
minutes respectively and a waste pipe can
empty 3 gallons per minute. All the 3 pipes
c)100 hours d)110 hours
working together can fill the tank in 15
minutes. The capacity of tank is
Solution :

1 A. 60 gallons B. 100 gallons


Let Capacity of Leak = 𝑥
C. 120 gallons D. 180 gallons
1
Capacity of Pipe =
9
Solution :
1
Pipe – Leak = 10 Total work = LCM of 20 and 24 = 120

1 1 1 120
Which gives 9 – 𝑥 = 10 Capacity of A = =6
20

Therefore x = 90 hours 120


Capacity of B = 24
=5

All the 3 are working for 15 minutes to fill


120 units.

pg. 24
10 Seconds

Number of units filled by A in 15 minutes = Therefore to complete 60 units they


90 120
require 17 hours

Number of units filled by B in 15 minutes =


75
16)Three pipes A, B, C can fill a tank from
Number of units removed by C in 15
empty to full in 30 minutes , 20 minutes
minutes = 45
and 10 minutes respectively. When the
45
tank is empty all three pipes are opened.
we get Capacity of C = 15 = 3 units A,B,C discharge chemical solutions P,Q,R
respectively, What is the proportion of
Therefore 1 unit is 1 gallon solution ‘R’ in the liquid in the tank after 3
minutes?
Work done by A, B and C in 1 minute = 8
a)6/11 b)5/11
There capacity of tank = 15 × 8 = 120
gallons c)8/11 d)7/11

Solution :

15)Three taps ‘A’, ‘B’ and ‘C’ can fill a tank Total Work = LCM of 30, 20 and 10 = 60
in 12,15 and 20 hours respectively. If ‘A’ is
open all the time and ‘B’ and ‘C’ are open Capacity of Pipe A in one hour =
total work 60
for one hour each alternatively, the tank = 30 = 2
time given for A
will be full in how many hours?
Capacity of Pipe B in one hour =
a)120/17 b)120/18 total work 60
= =3
time given for B 20
c)120/19 d)120
Capacity of Pipe C in one hour =
Solution : total work 60
time given for C
= 10 = 6

Total Work = LCM of 12, 15 and 20 = 60


Total solution filled in 3 minutes by 3 pipes
Capacity of Pipe A in one hour = = 11 × 3 = 33
total work 60
= =5
time given for A 12 Amount of liquid chemical R in total
solution = 6 × 3 = 18
Capacity of Pipe B in one hour =
total work 60
= 15 = 4 Therefore proportion of liquid R in total
time given for B 18 6
solution = 33 = 11
Capacity of Pipe C in one hour =
total work 60
time given for C
= 20 = 3

17)A tap can fill a tank in 6 hours. After


Work done by ‘A’ and ‘B’ during first hour half the tank is filled, three more similar
=9 taps are opened. What is the total time
taken to fill the tank completely?
Work done by ‘A’ and ‘C’ during second
hour = 8 A) 3 hrs 15 min B) 3 hrs 45 min

Work done in 2 hours = 17 C) 4 hrs 15 min D) 4 hrs 10 min

pg. 25
10 Seconds

Solution : and when the waste pipe is open, they can


together fill it in 18 minutes. The waste
Total work of 4 tanks = 6 pipe can empty the full cistern in?

6 A. 7 mins B. 9 mins
Capacity of each tank = 6 = 1

C. 13 mins D. 23 mins
For filling half the tank ( 3 units of work) 1
tap takes 3 hours
Solution :
Now to fill another half 4 taps are running,
Let A = 10 min, B = 15 min and A + B + C
3 = 18 min
They require = 0.75 hours or 45 minutes
4
1
Capacity of A = 1
10
Therefore total time = 3 hours 45 minutes
1
Capacity of B = 15 2

1
18)Two pipes A and B can fill a cistern in Capacity of A + B + C = 18 3
20 and 30 minutes respectively, and a
third pipe C can empty it in 40 minutes.
Substitute 1 and 2 in 3, we get C = 9 mins
How long will it take to fill the cistern if all
the 3 pipes are opened at the same time?

A. 7 1/7 mins. B. 15 1/7 mins.


20)A cistern is normally filled in 8 hours
but takes two hours longer to fill because
C. 17 1/7 mins. D. 19 1/7 mins.
of a leak in its bottom. If the cistern is full,
the leak will empty it in?
Solution :
A. 16 hrs B. 20 hrs
Total Work = LCM of 20, 30 and 40 = 120
C. 25 hrs D. 40 hrs
Capacity of Pipe A in one minute =
total work 120
time given for A
= 20 = 6 Solution :

1
Capacity of Pipe B in one minute = Let Capacity of Leak = 𝑥
total work 120
= =4
time given for B 30
1
Capacity of Pipe = 8
Capacity of Pipe C in one minute =
total work 120 1
time given for C
= 40 = 3 Pipe – Leak = 10

If both A, B and C are working together 1 1 1


Which gives 8 – 𝑥 = 10
then they can complete 7 unit of work in
one minute.
Therefore x = 40 hours
Therefore, to complete total 120 units of
120
work A, B and C need = 7 minutes
21)Two pipes can fill a tank in 18 minutes
19)Two taps can separately fill a cistern and 15 minutes. An outlet pipe can empty
10 minutes and 15 minutes respectively the tank in 45 minutes. If all the pipes are

pg. 26
10 Seconds

opened when the tank is empty, then how Here ‘A’ is running is running for 30
many minutes will it take to fill the tank? minutes and completes 90 units.

A. 9 mins. B. 10 mins. There ‘B’ is removing 90-40 = 42 units.

C. 11 mins. D. 12 mins. 42
To remove 48 units ‘B’ requires 2
= 21
minutes
Solution :

Total Work = LCM of 18, 15 and 45 = 90


23)Two pipes A and B can fill a tank in 15
Capacity of Pipe A in one min = minutes and 20 minutes respectively. Both
total work 90
= =5 the pipes are opened together but after 4
time given for A 18
minutes, pipe A is turned off. What is the
Capacity of Pipe B in one min = total time required to fill the tank?
total work 90
time given for B
= 15 = 6 A) 10 min. 20 sec. B) 11 min. 45 sec.

Capacity of Pipe C in one min = C) 12 min. 30 sec. D) 14 min. 40 sec.


total work 90
= =2
time given for C 45
Solution :
If both A, B and C are working together
Total Work = LCM of 15 and 20 = 60
then they can complete 9 unit of work in
one min.
Capacity of Pipe A in one minute =
total work 60
Therefore, to complete total 90 units of time given for A
= 15 = 4
90
work A, B and C need = = 10 min
1
Capacity of Pipe B in one minute =
total work 60
time given for B
= 20 = 3

22)Pipe A can fill a tank in 16 minutes and Work done by ‘A’ and ‘B’ in 4 minutes = 28
pipe B can empty it in 24 minutes. If both
the pipes are opened together after how
Work remaining = 32
many minutes should pipe B be closed, so
that the tank is filled in 30 minutes?
Therefore to complete 28 units ‘B’ requires
32
A. 19 mins B. 20 mins 3
= 10.66 minutes or 10 minutes 40
seconds.
C. 21 mins D. 22 mins
Therefore total time is 14 minutes 40
Solution : seconds.

Total Work = LCM of 16 and 24 = 48

Capacity of Pipe A in one minute = 24) A water tank is two-fifth full. Pipe A
total work 48 can fill a tank in 10 minutes and pipe B
= 16 = 3
time given for A can empty it in 6 minutes. If both the pipes
are open, how long will it take to empty or
Capacity of Pipe B in one minute = fill the tank completely?
total work 48
time given for B
= 24 = 2
A) 6 min to empty B) 6 min to fill

pg. 27
10 Seconds

C) 9 min to empty D) 9 min to fill If both A, B and C are working together


Solution : then they can complete 5 unit of work in 3
min.
Total Work = LCM of 10 and 6 = 60
Therefore, to complete total 120 units of
120
Capacity of Pipe A in one min = work A, B and C need = 5 = 72 min
total work 60
= =6
time given for A 10

Capacity of Pipe B in one hour =


total work 60 26)Pipe M and N running together can fill
time given for B
= 6 = 10 a cistern in 6 minutes. If M takes 5
minutes less than N to fill the cistern, then
2 the time in which N alone can fill the
Number of units already filled in 5 × 60 =
cistern will be
24
a) 15 min b) 10 min
If both A and B are working together then
they can remove 4 units of in one min. c) 30 min d) 25 min
Therefore, to remove total 24 units A and Explanation:
24
B need = = 6 hours Let pipe M fills the cistern in x minutes.
4
Therefore, pipe N will fill the cistern in
(x+5) minutes.

25)Taps X and Y can fill a tank in 30 and Now, 1/x + 1/(x+5) = 1/6 → x = 10
40 minutes respectively. Tap Z can empty Thus, the pipe M can fill in 10 minutes, so
the filled tank in 60 minutes. If all the three N can fill in 10+5 =15 minutes.
taps are kept open for one minute each
alternatively, how much time will the taps
take to fill the tank?
27)A cistern normally takes 10 hours to be
filled by a tap but because of one open
A) 48min B) 72min
outlet pipe, it takes 5 hours more. In how
many hours will the outlet pipe will empty
C) 24min D) None of these
a full cistern?
Solution : a) 20 hours b) 24 hours
c) 30 hours d) None of these
Total Work = LCM of 30, 40 and 60 = 120
As cistern is filled in 10 hours, therefore in
Capacity of Pipe A in one min = 1 hour, filled part → 1/10th
total work 120
time given for A
= 30 = 4 Now, due to outlet pipe, filled part in 1
hour = 1/15th
Capacity of Pipe B in one min =
total work 120 Part of the cistern emptied, due to leakage
time given for B
= 40 = 3 in 1 hour = 1/10 - 1/15 = 1/30th
Therefore the leak will empty the full
Capacity of Pipe C in one min = cistern in 30 hrs.
total work 120
time given for C
= 60 = 2

28)Two pipes P and Q can fill a cistern in


36 and 48 minutes respectively. Both
pipes are opened together, after how

pg. 28
10 Seconds

many minutes should Q be turned off,so Remaining Part = (1-17/20) = 3/20


that the cistern be fill in 24 minutes?
Now, it is the turn of P and Q and 3/20
a) 6 min b) 16 min part is filled by P and Q in 1 hour.
c) 10 min d) 12 min Therefore, Total time taken to fill the tank
= (6+1) hrs = 7 hrs
Explanation:
P can fill the cistern in 36 minutes, so in 1
min, P can fill the cistern = 1/36th part PERCENTAGE
In 24 min, P can fill the cistern = 24/36 = Basics
2/3rd. Remaining part = 1- 2/3 = 1/3rd
1. Percent implies “for every hundred”.
As Q can fill full cistern in 48 minutes, so it
will fill % is read as percentage and x % is read
as x per cent.
1/3rd part in 16 minutes.
2. To calculate p % of y
(p/100) x y = (p x y)/100
29)A cistern filled in 20 hours by three
pipes A, B and C. The pipe C is twice as p % of y = y % of p
fast as B and B is thrice as fast as A. How 3. To find what percentage of x is y: y/x ×
much time will pipe A alone take to fill the 100
tank?
4. To calculate percentage change in
a) 200 hrs b) 205 hrs value
c) 352 hrs d) Cannot be determined Percentage change = {change/(initial
Explanation: Suppose pipe A alone takes value)} x 100
x hours to fill the tank. 5. Percentage point change = Difference
Then pipes B and C will take x/3 and x/6 of two percentage figures
hours respectively to fill the tank. 6. Increase N by S % = N( 1+ S/100 )
Therefore 1/x + 3/x + 6/x= 1/20 ⇒ 10/x = 7. Decrease N by S % = N (1 – S/100)
1/20 ⇒ x = 200hrs
8. If the value of an item goes up/down by
x%, the percentage reduction/increment to
30)Three taps P,Q and R can fill a tank in be now made to bring it back to the
10,20 and 30hours respectively. If P is original point is 100x/ (100 + x) %.
open all the time and Q and R are open 9. If A is x% more /less than B, then B is
for one hour each alternately, then the 100x/(100 + x) % less/more than A.
tank will be full in :
10. If the price of an item goes up/down by
a) 6 hrs b) 6.5 hrs x %, then the quantity consumed should
c) 7 hrs d) 7.5 hrs be reduced by 100x/ (100 + x)% so that
the total expenditure remains the same.
Explanation: (P+Q)’s 1 hour’s work =
(1/10+1/20) = 3/20 11. Successive Percentage Change

(A+C)’s 1 hour’s work = (1/10+1/30) = If there are successive percentage


2/15 increases of a % and b%, the effective
percentage increase is:
Part filled in 2 hrs = (3/20+2/15) = 17/60
{(a + b + (ab/100)}%
Part filled in 6 hrs = (3×17/60) = 17/20

pg. 29
10 Seconds

12. Percentage – Ratio Equivalence: Pass percentage of female = 20% of 60 =


12
Total pass percentage = 50
38
So, pass percentage of males = 60 × 100
= 63.33%

Questions:
4. Ramesh spent Rs. 38460 on the
1.Evaluate [38% of 455 + 45% of 250] renovation of his home, Rs. 24468 on
buying furniture and remaining 28% of the
a)250 b)285.4 total amount he had as cash with him.
c)253 d)254 What was the total amount?

Solution: a) 87400 b) 92520

38% of 455 + 45% of 250 c) 88470 d) 90150

= 172.9 + 112.5 Solution:

= 285.4 Let X be the total amount with Ramesh


Given 28% of X = Cash remaining

2. If ‘X’ earns 35% more than ‘Y’, how Amount spent = 72% of X = 38460 +
much % does ‘Y’ earn less than ‘X’? 24468 = 62928

a)10% b)20%  72% of X = 62928


 X = 87400
c)25.9% d)40%
Solution: 5.Evaluate [28% of 450 + 45% of 280]
Let’s Y earnings = 100 a)250 b)252
Then X earns 35% more than Y = 135 c)253 d)254
Y earns 35 rupees less than X Solution:
35 28% of 450 + 45% of 280
So, 135 × 100 = 25.9%
= 126 + 126
= 252
3. In recently conducted PWD exam a
total of 10 lakh student appeared. 40% of
them were females while rest were males.
The pass% among females is 20% and 6. What percent of 6.5L is 130ml?
total pass % is 50%. What is pass % of a)1% b)2%
males?
c)3% d)4%
a)10% b)20%
Solution:
c)30% d)63.33%
130
Solution: 6500
× 100 = 2%

Let`s take there are 100 students.


Females = 40 and males = 60 7. 65% of a number is 21 less than four
fifth of that number. What is the number?

pg. 30
10 Seconds

a)150 b)130 After second year height = 144


c)140 d)120 100 → 144
Solution: x → 540
Let the number be x x = 375cm
Then according to the question Therefore, the height before 2 years =
4 375cm.
65% of x = 5 x – 21

x = 140
10. A fruit seller had some apples. 10% of
them were spoiled in transport and he sold
90% of remaining and he was left with 81
8. The population of a town increases at apples. How many apples he had
the rate of 10% every year. If at present originally?
the population is 12100. What was its
population 2 years ago? a)900 b)1000
a)5000 b)7500 c)1200 d)1400
c)12000 d)10000 Solution:
Solution: Let us take there are 100 apples originally.
Let original population = 100 Spoiled = 10
The rate is increasing 10% every year, He sold 90% of the remaining = 90% of
90% = 81
After first year Population = 110
The number of apples left in 100 apples =
After second year Population = 121 9
100 → 121 100 → 9
x → 12100 x → 81
x = 10000 x = 900 apples.
Therefore, the population before 2 years =
10000.
11. Deepika went to a fruit market with a
certain amount of money. With the money
9. A coconut tree was planted 2 years ago. she has she can buy 30 apples or 50
Its height increases at the rate of 20% mangoes. She retains 20% of her money
every year. If at present the height of tree for auto fare. Now she buys 25 mangoes.
is 540cm. What was the height when tree How many more apples she can buy?
was planted? a)9 b)10
a)300cm b)325cm c)12 d)14
c)350cm d)375cm Solution:
Solution: Let Deepika’s total money = 100
Let actual height = 100 The cost of 30 apples or 50 mangoes =
The rate is increasing 20% every year, 100

After first year height = 120 For auto fare = 20


The cost of 25 mangoes = 50

pg. 31
10 Seconds

Remaining amount = 30 Solution:


For 100 rupees she can buy 30 apples so, Use 100 and cross multiplication to solve.
100 → 30 State A State B
30 → x Candidates = 100 Candidates = 100
x = 9 apples. Selected = 6 Selected = 7
Compare to state A , state B has one
candidate extra
12. If ‘A’ earns 25% more than ‘B’, how
much % does ‘B’ earn less than ‘A’? 100 → 1
a)10% b)20% x → 80
c)30% d)40% x = 8000
Solution: Total number of candidates in each state
= 8000.
Let’s B earnings = 100
Then A earns 25% more than B = 125
15. In an election a candidate who got
B earns 25 rupees less than A 84% of votes was elected by a majority of
25 476 votes. What is the total number of
So, 125 × 100 = 20%
votes polled?
a)600 b)700
13. Raman’s salary was decreased by c)800 d)900
50% and subsequently increased by 50%,
how much percent does he lose? Solution:

a)15% b)20% Let total votes = 100

c)25% d)30% Votes got = 84

Solution: Extra votes ( won by majority ) = 100 – 84


= 68
Let Raman’s salary = 100
100 → 68
First his salary was decreased by 50% =
50 x → 476
Again, increased by 50% = 50 + 25 = 75 x = 700 votes
Therefore, loss = 25%
16. In an election between two candidates
20% of the votes were declared invalid. If
14. In a competitive examination in state the total number of votes was 7500, the no
‘A’ 6% candidates got selected from the of valid votes that other candidate got if
total appeared candidates. State ‘B’ had candidate who won got 55% of valid
an equal number of candidates appeared votes?
and 7% candidates got selected with 80
more candidates got selected than ‘A’. a)2100 b)2400
What was the number of candidates
c)2700 d)3000
appeared from each state?
Solution:
a)6000 b)7000
Let total votes = 100
c)8000 d)9000

pg. 32
10 Seconds

Invalid votes = 20
Valid votes = 80 19. Aman gave 40% of amount he had to
45
Rohan. Rohan in turn gave one fourth of
Lost candidate got 45% of 80 = 100 × 80 = what he received from Aman to Sahil.
36 After paying Rs 200 to taxi driver out of
the amount he got from Rohan, Sahil now
100 → 36 has Rs 600 left with him. How much
7500 → x amount did Aman have?

x = 2700 a)6000 b)5000


c)8000 d)7000

17. In recently conducted SSC board Solution:


exam a total of 5 lakh student appeared. Let Aman have 100 rupees
60% of them were females while rest were
males. The pass% among females is Aman to Rohan = 40
33.33% and total pass % is 50%. What is 1
pass % of males? Rohan to Sahil = 4 × 40 = 10

a)10% b)20% If Aman had 100 rupees Sahil gets 10


rupees.
c)30% d)75%
Now Sahil had 800 rupees use cross
Solution: multiplication.
Let`s take there are 100 students. 100 → 10
Females = 60 and males = 40 x → 800
Pass percentage of female = 33.33% of x = 8000
60 = 20
Total pass percentage = 50
30
20. A Student secures 42.5% and failed by
So, pass percentage of males = 40 × 100 5 marks. If he scores 52.5% he would
= 75% pass by 15 more marks. Find the minimum
marks to pass the exam?
a)35 b)60
18. A man spends 35% of his income on
food, 25% on children education and 80% c)80 d)90
of remaining on house rent. What percent
Solution:
of his income he is left with?
42.5% + 5 → pass
a)6% b)6.25%
52.5% - 15 → pass
c)7% d)8%
Both are passing marks so we can equate
Solution:
these two equations
Let salary = 100
42.5% + 5 = 52.5% - 15
For food = 35 and for education = 25
1% = 2
Remaining = 40
So, minimum marks = 42.5 × 2 + 5 = 90
House rent = 80% of 40 = 32
Therefore, minimum marks = 90
Therefore, he saves 8%.

pg. 33
10 Seconds

21. A Student secures 40% and failed by Solution:


10 marks. If he scores 50% he would pass
by 15 more marks. Find the maximum Price Consumption Expenditure
marks of the examination? Initial 100 100 10000
Later 125 x 10000
a)110 b)250
c)200 d)260  125 × x = 10000
 x = 80
Solution:
So, the person should reduce 20%
40% + 10 → pass
consumption.
50% - 15 → pass
Both are passing marks so we can equate
24. 1100 boys and 700 girls are examined
these two equations
in a test; 42% of the boys and 30% of the
40% + 10 = 50% - 15 girls pass. The percentage of the total who
failed is:
1% = 2.5
a) 58 b) 62.66
So, maximum marks = 100 × 2.5 = 250
c) 64 d) 67
Therefore, maximum marks = 250
Solution:
Total number of students = 1100 + 700 =
22. When the price of the product was 1800.
decreased by 10%, the number sold
increased by 30%. What was the effect on Number of students passed = (42% of
total revenue? 1100 + 30% of 700) = (462 + 210) = 672

a) Increase of 11.7% Number of failures = 1800 - 672 = 1128.


1128
b) Increase of 18% Percentage of failure = 1800 × 100 =
c) Decrease of 11.7% 62.66%

d) Decrease of 18%
Solution: 25. Of the 1000 inhabitants of a town, 60
% are males of whom 120 % are literate.
Products Total If, of all the inhabitants, 25% are literate,
Price
revenue then what percent of the females of the
Initial 100 100 10000 town are literate?
Later 90 130 11700
a) 32.5 % b) 43 %
c) 46.6 % d) 53.2 %
So, the effect on total revenue = Increase
of 11.7% Solution:
Number of males = 60% of 1000 = 600
23. In the new budget, the price of Number of females = (1000 - 600) = 400
kerosene is increased by 25%. By how
much % must a person reduce his Number of literates = 25% of 1000 = 250
consumption so that his expenditure does Number of literate males = 20% of 600 =
not increase?
120
a)10% b)20%
Number of literate females = (250 - 120) =
c)25% d)30% 130

pg. 34
10 Seconds

130 50 paise coins removed = Rs. (1200×


Required percentage = ( × 100) % =
400 24
32.5 % )= 288.
100
25 50
Money removed = 72 × ( ) + 288× ( )
100 100
26. In a city, 35% of the population is = 162
composed of migrants, 20% of whom are 162
from rural areas. Of the local population, Required percentage = 750 × 100 =
48% is female while this figure for rural 21.6%
and urban migrants is 30% and 40%
respectively. If the total population of the
city is 728400, what is its female 28. Two students appeared at an
population? examination. One of them secured 9
a) 324138 b) 349680 marks more than the other and his marks
was 56% of the sum of their marks. The
c) 509940 d) None of these marks obtained by them are
Solution: a)39, 30 b)41, 32
Total Population = 728400 c)42, 33 d)43, 34
Migrants = 35 % of 728400 = 254940 Solution:
Local population = (728400 - 254940) = Let their marks be (x + 9) and x.
473460 56
Then, x + 9 = (x + 9 + x)
100
Rural migrants = 20% of 254940 = 50988
25 (x + 9) = 14 (2x + 9)
Urban migrants = (254940 - 50988) =
203952 3x = 9
Female population = 48% of 473460 + x = 33
30% of 50988 + 40% of 203952
So, their marks are 42 and 33.
 227260.8 + 15296.4 + 81580.8
= 324138
29. In an election between two candidates
27. A bag contains 600 coins of 25 paise 70% of the voters cast their votes, out of
denomination and 1200 coins of 50 paise which 2% of the votes were declared
denomination. If 12% of 25 paise coins invalid. A candidate got 7203 votes which
and 24% of 50 paise coins are removed, was 60% of the total valid votes. Find the
the percentage of money removed from total number of voters enrolled in that
the bag is nearly election.

a) 21.6 % B) 15.3 % a) 18050 b) 17500

C) 14.6 % D) 12.5 % c) 17000 d) 7203

Solution: Solution:
25 Let total number of votes enrolled = x
Total money = Rs [600× (100) +
50 No. of votes casted = 70% of x = 0.7x
1200 × (100)] = Rs. 750.
12 Valid votes = 98% of 0.7x
25 paise coins removed = Rs. (600× 100) =
 60% of total valid votes = 7203 =
72.
60% of 98% of 0.7x
 0.6 × 0.98 × 0.7x = 7203
 x = 17500

pg. 35
10 Seconds

total ages of Amulya, Bhavana and Chitra


is 27, then how old is Bhavana?
30. A Student secures 50% and failed by
20 marks. If he scores 60% he would pass a)10 years b)15years
by 25 more marks. Find the maximum
marks of the examination? c)20 years d)25 years

a) 110 b) 450 Solution:

c) 200 d) 260 Here Chitra is the youngest person so,


Chitras’s age = x
Solution:
Bhavana’s age = 2 and Amulya’s age = 2x
50% + 20 → pass +2
60% - 25 → pass Then x + 2x + 2x + 2 = 27
Both are passing marks so we can equate  X = 5, therefore, Bhavana = 2x =
these two equations 10 years.
50% + 20 = 60% - 25
1% = 4.5 2. The age of John is 4 times of Robert.
Five years ago, John was nine times old
So, maximum marks = 100 × 4.5 = 450 as Robert was at that time. The present
Therefore, maximum marks = 450 age of John is?
a) 30 b)32

AGES c)28 d)40

Basics Solution:

Given below are a few formulas related to Present age Ago or


the problems on ages which may help to before (5
answer the questions quicker and also get years)
a better idea of the concept: John 4x 4x – 5
Robert x x–5
1)If you are assuming the current age to So, according to the question we get the
be x, then the age after n years will be equation, 4x – 5 = 9(x – 5)
(x+n) years.
Therefore, x = 8
2)If you are assuming the current age to
be x, then the age before n years will be The present age of John = 4x = 32
(x-n) years.
3)If the age is given in the form of a ratio,
3. A man is 24 years older than his
for example, p:q, then the age shall be
daughter. In two years his age will be
considered as qx and px
twice the age of his daughter. The present
4)If you are assuming the current age to age of his daughter is
be x, then n times the current age will be
a)14 b)18
(x×n) years
c)20 d)22
5)If you are assuming the current age to
be x, then 1/n of the age shall be equal to Solution:
(x/n) years
Present age After or
Questions: hence (2
1.‘Amulya’ is two years older than years)
‘Bhavana’ who is twice as old as ‘Chitra’. If Father x + 24 x + 26

pg. 36
10 Seconds

Daughter x x+2 6. The sum of ages of 5 children born at


the intervals of 3 years each is 50 years.
What is the age of youngest child?
So, according to the question we get the
equation, x + 26 = 2(x + 2) a)4 b)8
Therefore, x = 22 c)10 d)12
The present age of the daughter = 22 Solution:
years.
Here fifth born child is the youngest one
So, 5th born child’s age = x
4. At present, the ratio between the ages
4th born child’s age = x + 3
of Sushma and Nithin is 4 : 3. After 6
years, Sushma’s age will be 26 years. 3rd born child’s age = x + 6
What is the age of Nithin at present?
2nd born child’s age = x + 9
a)12 b)15
1st born child’s age = x + 12
c)19 and half d)21
Therefore, x + x + 3 + x + 6 + x + 9 + x +
Solution: 12 = 50
Sushma : Nithin = 4 : 3 Age of youngest child is x = 4
Present age After or
hence (6
years) 7. The total age of A and B is 12 years
Sushma 4x 4x + 6 more than the total age of B and C. C is
Nithin 3x 3x + 6 how many years younger than A?
a)12 b)24
So, according to the question we get the c)C is elder than A
equation, 4x + 6 = 26
d)Data inadequate
Therefore, x = 5
Solution:
The present age of the Nithin = 3x = 15
years. So according to the question
(A + B) - (B + C) = 12
5. ‘A’ is two years older than ‘B’ who is A - C = 12
twice as old as ‘c’. If total ages of A, B and
C is 27, then how old is B?
a)10 years b)15years 8.The sum of the present ages of a father
and his son is 60 years. Six years ago,
c)20 years d)25 years
Father’s age was five times the age of the
Solution: son. After 6 years, son’s age will be:
Here C is the youngest person so, C’s age a)12 b)14
=x
c)18 d)20
B’s age = 2x and A’s age = 2x + 2
Solution:
Then x + 2x + 2x + 2 = 27
Present age Ago or
X = 5, therefore, B = 2x = 10 years. before (6
years)

pg. 37
10 Seconds

Father 60 – x 54 – x So, according to the question we get the


Son x x–6 equation, 4x – 5 = 9(x – 5)
Therefore, x = 8
So, according to the question we get the
The present age of a man = 4x = 32
equation 54 – x = 5(x – 6)
So, x = 14
11. A man is 24 years older than his son.
After 6 years son’s age will be 14 + 6 = 20
In two years his age will be twice the age
years.
of his son. The present age of his son is
a)14 b)18
9. Nisha is 15 years elder to Romi. If 5
c)20 d)22
years ago, Nisha was 3 times as old as
Romi, then find Nisha’s present age. Solution:
a)32.5 years b)27.5 years After or hence (2
Present years)
c)25 years d)24.9 years
age
Solution: x + 24 x + 26
Father
Present age Ago or
before (5 X x+2
years) Son
Nisha x + 15 x + 10
Romi x x–5
So, according to the question we get the
equation, x + 26 = 2(x + 2)
So, according to the question we get the Therefore, x = 22
equation, x + 10 = 3(x – 5)
The present age of the son = 22 years.
Therefore, x = 12.5
Nisha’s present age = x + 15 = 27.5 years.
12. The sum of the ages of a son and
father is 56 years after four years the age
10. The age of a man is 4 times of his son. of the father will be three times that of the
Five years ago, the man was nine times son. Their ages in years respectively are:
old as his son was at that time. The a)12, 44 b)16, 42
present age of man is?
c)16, 48 d)18, 36
a) 30 b)32
Solution:
c)28 d)40
Present age After or
Solution:
hence (4
Present Ago or years)
age before (5 Father 56 - x 60 - x
years) Son x x+4
4x 4x – So, according to the question we get the
Man 5 equation, 60 - x = 3(x + 4)
X x–
Therefore, x = 12
Son 5
The present age of the son = 12 years.
The present age of the father = 44 years.

pg. 38
10 Seconds

The present age of the Anand = 4x = 24


years.
13. Mother’s age today is thrice as her
daughter’s. After 10 years it would be just
double. How old is the daughter today?
15. At present, the ratio between the ages
a)8 years b)9 years of Arun and Deepak is 4 : 3. After 6 years,
Arun’s age will be 26 years. What is the
c)10 years d)11 years age of Deepak at present?
Solution: a)12 b)15
Present age After or c)19 and half d)21
hence (10
years) Solution:
Mother 3x 3x + 10
Arun : Deepak = 4 : 3
Daughter x x + 10
After or hence
Present (6 years)
So, according to the question we get the age
equation, 3x + 10 = 2(x + 10) 4x + 6
Therefore, x = 10 Arun 4x
3x + 6
The present age of the daughter = 10 Deepak 3x
years.
So, according to the question we get the
equation,4x + 6 = 26
14. Present ages of Sameer and Anand
are in the ratio of 5 : 4 respectively. Three Therefore, x = 5
years hence, the ratio of ages will become
11 : 9 respectively. What is Anand’s The present age of the Deepak = 3x = 15
present age in years? years.

a)24 b)27
c)40 d)50 16. Sachin is younger than Rahul by 7
years. If their ages are in the respective
Solution: ratio of 7:9, how old is Sachin?
Sameer : Anand = 5 : 4 a)16 b)18
Then, Sameer`s present age = 5x and c)28 d)24.5
Anand`s present age = 4x
Solution:
After or hence
Present (3 years) Sachin`s present age = x
age Then, Rahul`s present age = x + 7
5x + 3
𝑥 7
Sameer 5x Their ages are in the ratio, 𝑥+7 = 9
4x + 3
Anand 4x Therefore, x = 24.5

So, according to the question we get the


17. What is Aman's present age, if after 20
ratio, 5x + 3 : 4x + 3 = 11 : 9
years his age will be 10 times his age 10
Therefore, x = 6 years back?
a) 6.2 years b) 7.7 years

pg. 39
10 Seconds

c) 13.3 years d) 10 years younger to her was born. What is the


difference between the ages of her
Solution: parents?
Present Before After a)2 b)4
(10 (20
years) years) c)6 d)8
Aman X x-10 x-20
Solution:
Mother's age when Ayesha's brother was
We are given that, Aman's age after 20
born = 36 years.
years (x + 20) is 10 times his age 10 years
back (x – 10) Father's age when Ayesha's brother was
born = (38 + 4) years = 42 years.
Therefore, (x + 20) = 10 (x – 10)
Required difference = (42 - 36) years = 6
Solving the equation, we get x + 20 = 10x
years.
– 100
Therefore, x = 13.3 years
20. A person was asked to state his age in
years. His reply was “Take my age 3 years
18. The age of father 10 years ago was hence, multiply it by 3 and then subtract 3
thrice the age of his son. Ten years hence, times my age 3 years ago and you will
father's age will be twice that of his son. know how old I am”. What as the age of
The ratio of their present ages is: the person?
a)5 : 2 b)7 : 3 a)14 years b)16 years
c)9 : 2 d)13 : 4 c)18years d)20years

Solution: Solution:

Ago After Let the present age of the person be x


Present (10 (10 years
years) years)
His age after 3 years = (x + 3) years
3x + 10
Father 3x 3x + 10 His age 3 years ago = (x − 3) years
x + 10 x + 20
Son x Then according to given condition, his
present age will be 3 (x + 3) – 3 (x − 3)

According to the given after 10 years = (3x + 9 − 3x + 9) = 18 years.


father`s age (3x +10) is twice the age of
his son (x + 20)
21. A father said to his son, 'I was as old
Therefore, (3x + 10) = 2(x + 10) as you are at present at the time of your
Solving the equation, we get x = 20 birth.' If the father's age is 38 years now,
the son's age five years back was:
So, the ratio of present age of father and
son is 3x + 10 = 70 and x + 10 = 30 a)14 years b)19 years

Therefore, their ratios = 70 : 30 = 7 : 3 c)33 years d)38 years


Solution:

19. Ayesha’s father was 38 years of age Let the son's present age be x years, then
when she was born while her mother was father’s present age be 38-x
36 years old when her brother four years We can write, (38 - x) = x

pg. 40
10 Seconds

2x = 38. Solving the equation, we get x = 24


And x = 19. 4
Therefore, the present age of Ajay = x +
3
Son's age 5 years back (19 - 5) = 14 8 = 40 years.
years.

24. If two times of the daughter’s age in


22. The ages of Krish and Vaibhav are in years is included to the mother’s age, the
the proportion of 3 : 5. After 9 years, the total is 70 and if two times of the mother’s
proportion of their ages will be 3 : 4. Then age is included to the daughter’s age, the
the current age of Vaibhav is total is 95. So, the Mother’s age is

a)10 b)13 a)30 b)38

c)15 d)18 c)40 d)41

Solution: Solution:

Krish : Vaibhav = 3 : 5 Let daughter’s age = A and mother’s age


=B
Present age After or
hence (9 Given that 2A + B = 70 and A + 2B = 95
years)
Solving for B, we will get B = 40.
Krish 3x 3x + 9
Vaibhav 5x 5x + 9 Therefore, mother`s age = 40 years.

So, according to the question their ages


3𝑥+9 3 25. The ages of Anushka, Kajal and Charu
after 9 years are in the ratio, 5𝑥+9 = 4 latha together are 57 years. Kajal is thrice
as old as Anushka and Charu latha is 12
Therefore, x = 3
years older than Anushka. Then, the
Vaibhav`s present age = 5x = 15 years. respective age of Anushka, Kajal and
Charu latha is?
a)27, 9, 21 b)9, 21, 27
4
23. Eight years ago, Ajay's age was 3
c)21, 27, 9 d)9, 27, 21
times that of Vijay. Eight years hence,
6 Solution:
Ajay's age will be 5 times that of Vijay.
What is the present age of Ajay? Let Anushka’s age = x
a)41 years b)40 years Charu latha is 12 years older than
Anushka, so Charu latha’s age = x+12
c)37 years d)33 years
Kajal is thrice as old as Anushka, so
Solution:
Kajal`s age = 3x
Present Ago (8 After (8
The ages of Anushka, Kajal and Charu
years) years)
4 4 4 latha together are 57 years
Ajay x +8 x x+ 16
3 3 3
x + (x + 12) + 3x = 57
Vijay x+8 x x + 16
x=9
According to the given after 8 years Ajay`s Anushka’s age = 9
6
age will be 5 times that of Ajay
Kajal age’s = 3x = 27
4 6
Therefore, 3
x + 16 = (x + 16)
5
Charu latha’s age = x + 12 = 21

pg. 41
10 Seconds

Age of Ashok = 3x
26. Today is Varun's birthday. One year, Age of Deepak = 5x
from today he will be twice as old as he
was 12 years ago. How old is Varun Age of Ramu = 6x
today? Given that 6x - 5x = 5 => x = 5
a)25 years b)24 years Therefore, age of Ashok = 3x = 15 years.
c)23 years d)22 years
Solution: 29. The age of Manisha and Sudeshna
Let Varun's age = x years. are in the ratio of 5 : 6 respectively. After 8
years the ratio of their ages will be 7 : 8.
From the given conditions, What is the difference of their ages ?
x + 1 = 2(x - 12) a)6 b)4
x = 25 years. c)8 d)10
Therefore, Varun`s age = 25 years. Solution:
Present After (8
years)
27. Sony is one-fifth the age of her mother
Manisha 5x 5x + 8
15 years ago and Sony’s brother is three-
Sudeshna 6x 6x + 8
fifth the age of his mother 10 years ago. If
the sum of Sony and her brother’s ages is
31 then how old is Sony's mother? After 8 years, their ages are in the ratio,
5𝑥+8 7
a)40 b)50 6𝑥+8
=8

c)60 d)70 x=4


Solution: Therefore, difference = 4 years.
Sony and her brother's age sum is 31
Let Sony's mother's age be 'x'. 30. The difference between present ages
of Ravali and Swarna is 9 years. After 7
Now the conditions given for both of their years, Ravali’s age is twice of Swarna’s
ages related to their mom is age. What will be Ravali’s age after 4
1 3 years?
So, 5 (x – 15) + 5 (x – 10) = 31
a)15 years b)16 years
x = 50
c)20 years d)21 years
So, mother's age = 50.
Solution:
Present After (7
28. The Age of Ashok, Deepak and Ramu years)
are in the proportion of 3 : 5 : 6. If the Ravali X x+7
difference between the ages of Deepak Swarna x–9 x-2
and Ramu is 5 years . What is Ashok's
age in years?
From the given data,
a)15 b)10
x + 7 = 2(x - 2)
c)20 d)30
x = 11 years
Solution:

pg. 42
10 Seconds

Required Ravali's age after 4 years = 11 + for 5 consecutive months. How much sale
4 = 15 years. must he have in the sixth month so that he
gets an average sale of Rs, 6500?
A) 4991 B) 5467
AVERAGES
C) 5987 D) 6453
Basics
Solution:
Average is mainly defined as the sum of
observations divided by the number of Total sale for 5 months = Rs. (6435 +
observations. 6927 + 6855 + 7230 + 6562) = Rs. 34009.
Formula: Required sale = Rs. [(6500 x 6) - 34009]
Average= (Sum of observations)/(Number = Rs. (39000 - 34009)
of observations)
= Rs. 4991.
Important Points:
1)When a person replaces another person
then: 2. The average of 7 consecutive numbers
is 20. The largest of these numbers is :
If the average is increased, then
A) 21 B) 22
Age of new person= Age of person who
left + (Increase in average * total number C) 23 D) 24
of persons) Solution:
If the average is decreased, then Let the numbers be x, x + 1, x + 2, x + 3,
Age of new person= Age of person who x + 4, x + 5 and x + 6,
left - (Decrease in average * total number Then, average =
of persons) (x + (x + 1) + (x + 2) + (x + 3) + (x + 4) + (x + 5) + (x + 6))
7
2)When a person joins the group: = 20
In case of an increase in average,  x =17.
Age of new member= Previous average+ Largest number = x + 6 = 23.
(Increase in average * Number of
members including new member)
In case of decrease in average, 3. A pupil's marks were wrongly entered
as 83 instead of 63. Due to that the
Age of new member= Previous average- average marks for the class got increased
(Decrease in average * Number of by half. The number of pupils in the class
members including new member) is :
3)In the Arithmetic Progression there are A) 45 B) 40
two cases:
C) 39 D) 37
When the number of terms is odd - the
average will be the middle term. Solution:

When number of terms is even - the Let there are x pupils in the class.
average will be the average of two middle 1 𝑥
terms. Total increase in marks = (𝑥 × 2 ) = 2
𝑥
Questions:  2
= (83 - 63)
𝑥
1. A grocer has a sale of Rs 6435, Rs.  2
= 20
6927, Rs. 6855, Rs. 7230 and Rs. 6562  x = 40.

pg. 43
10 Seconds

When N = 3, sum = 3 × 30 = 90kg


4. The average age of a husband and his When N = 2, sum = 2 × 35 = 70kg
wife was 23 years at the time of their
marriage. After five years they have a one Therefore, the weight of the excluded
- year old child. The average age of the person = 90 – 70 = 20kg.
family now is : Alternatively,
A) 25 B) 23 Here one person is excluding and the
C) 19 D) 18 average is increasing, so the person`s
weight is less than the original average 30
Solution: and difference in the average = 5,
Sum of the present ages of husband, wife Number of students after excluding = 2
and child = (23 × 2 + 5 × 2) + 1 = 57
years. So, 30 - 5 × 2 = 20kg.
57
Required average = 3
= 19 years.
7. The average weight of 3 students in a
class is 30kg. If one person is excluded
the average becomes 30kg. The weight of
5. The average weight of 3 students in a
the excluded person is
class is 30kg. If one person is excluded
the average becomes 25kg. The weight of a)40kg b)30kg
the excluded person is
c)20kg d)10kg
a)40kg b)30kg
Solution:
c)20kg d)10kg
When N = 3, sum = 3 × 30 = 90kg
Solution:
When N = 2, sum = 2 × 30 = 60kg
When N = 3, sum = 3 × 30 = 90kg
Therefore, the weight of the excluded
When N = 2, sum = 2 × 25 = 50kg person = 90 – 60 = 30kg.
Therefore, the weight of the excluded Alternatively,
person = 90 – 50 = 40kg.
Here, the average is not changing it
Alternatively, means the weight of the excluded person
is also 30 kg.
Here one person is excluding and the
average is decreasing, so the person`s
weight is more than the original average
30 and difference in the average = 5, 8. The average of 5 numbers is 27. If one
number is excluded average becomes 25.
Number of students after excluding = 2 The excluded number is
So, 30 + 5 × 2 = 40kg. a)40 b)35
c)20 d)10
6. The average weight of 3 students in a Solution:
class is 30kg. If one person is excluded
the average becomes 35kg. The weight of When N = 5, sum = 5 × 27 = 135kg
the excluded person is
When N = 4, sum = 4 × 25 = 100kg
a)40kg b)30kg
Therefore, the weight of the excluded
c)20kg d)10kg person = 135 – 100 = 35kg.

Solution: Alternatively,

pg. 44
10 Seconds

Here one person is excluding and the Here one match is including and the
average is decreasing, so the excluding average is increasing by 5 runs so he
number is greater than the original should score more than the original
average 27 and difference in the average average 35 and difference in average = 5.
=2
Number of matches = 12
After excluding one number, N = 4.
So, 35 + 5 × 12 = 95kg.
So, 27 + 2 × 4 = 35kg.

11.The average of 13 numbers is 15. The


9. The average weight of 20 students in a average of first 6 numbers is 12 and last 6
class is 32kg. If weight of teacher is numbers is 17. Then the middle number is
included the average becomes 34kg.
What is the weight of teacher? a)21 b)31

a)40kg b)35kg c)41 d)12

c)74kg d)84kg Solution:

Solution: Let middle number be x

When N = 20, sum = 20 × 32 = 640kg Sum = average × number of observations

When N = 21, sum = 21 × 34 = 714kg The sum of 13 numbers = 13 × 15 = 195

Therefore, the weight of the included The average of first 6 numbers = 6 × 12 =


person = 714 – 640 = 74kg. 72

Alternatively, The average of last 6 numbers = 6 × 17 =


102
Here one person is including and the
average is increasing, so the person`s So, 72 + x + 102 = 195
weight is more than the original average We get x = 21
32 and difference in the average = 2,
Number of people after including = 21
12. The average of all workers in a
So, 32 + 2 × 21 = 74kg. workshop is Rs 5000. The average salary
of first 6 workers is Rs 7000 and
remaining workers is Rs 4000. How many
10. The average score of a cricketer in 11 workers are there?
matches was 35. How many runs must he
make in next innings so as to increase his a)18 b)20
average by 5 runs? c)22 d)12
a)40 b)95 Solution:
c)20 d)10 Let number of workers be x
Solution: Sum = average × number of observations.
When N = 11, sum = 11 × 35 = 385 runs The sum of x workers = 5000x
When N = 12, sum = 12 × 40 = 480 runs The sum of first 6 workers = 6 × 7000 =
Therefore, the cricketer should score = 42000
(480 – 385) = 95 runs. The sum of remaining workers = x –
Alternatively, 6(4000) = 4000x - 24000
So, 42000 + 4000x – 24000 = 5000x

pg. 45
10 Seconds

 x = 18 15. The average weight of A, B and C is


50kg. The average weight of ‘A’ and ‘B’ is
45kg. The average weight of ‘B’ and ‘C’ is
13. The average age of 10 men is 47kg. What is the weight of ‘B’?
increased by 3 years when two of them
whose ages are 22 and 24 are replaced a)60kg b)34kg
by 2 new men. What is the average age of
2 new men? c)40kg d)50kg

a)38 b)40 Solution:

c)42 d)44 The sum of A + B + C = 3 × 50 = 150 → 1

Solution: The sum of A + B = 90 → 2

Let the original average be x The sum of B + C = 94 → 3

S=A×N Substitute 2 in 1

Sum = 10x Then C = 60

After replacement, sum = 10 (x + 3) Substitute the value of C in 3, we get B =


34 kg
Let p and q be the newly added members,
Then according to the question,10x – 22 –
24 + p + q = 10 (x + 3) 16. A family consists of Grandparents,
Parents and 3 children. The average age
p + q = 76 of grandparents is 50 years, parents is 35
76 years and grandchildren is 7 years. Find
Average = 2
= 38. the average age of family?
a)191/7 b)197/7
14. The average weight of 40 students in a c)195/7 d)130/7.
class is 45kg. 10 of them whose average
weight is 50kg leaves the class. Another Solution:
10 students whose average weight is Sum of grandparents = 2 × 50 = 100
46kgs join the class. Average weight of
new class is? Sum of parents = 2 × 35 = 70

a)46 b)45 Sum of children = 3 × 7 = 21


100+70+21 191
c)47 d)44 Average = =
7 7
Solution:
The sum of 40 students = 40 × 45 = 1800 17. In an examination the student average
The sum of 10 students leaving the class marks were 50. If he had obtained 15
= 10 × 50 = 500 more marks in English and 5 more marks
in Hindi paper, his average would be 52.
The sum of 10 students joining the class = How many papers were there in exam?
10 × 46 = 460
a)6 b)8
Therefore, the new sum of the class =
1800 – 500 + 460 = 1760 c)10 d)12
1760 Solution:
The average weight of new class = 40
=
44 Let number of papers = x
Sum = 50x

pg. 46
10 Seconds

Then, according to the question 50x + 15 A) 8 B) 9


+ 5 = 52x
C) 10 D) 11
 x = 10
Solution:

18. Ian has 14 boxes of paper and divides Let the number of papers be x.
them evenly between 4 co-workers. How Then sum = 63x
many whole boxes did each co-worker
get? 63x + 20 + 2 = 65x
A) 2 B) 2.5 2x = 22
C) 3 D) 3.5 x = 11.
Solution: There are 11 papers in the exam.
Given number of boxes = 14
Number of workers = 4 21. The average price of three items of
furniture is Rs. 15000. If their prices are in
Now, number of whole boxes per worker = the ratio 3 : 5 : 7, the price of the cheapest
14
= 3.5 item is :
4

Hence, number of whole boxes per each A) 6000 B) 7000


co-worker = 3 C) 8888 D) 9000
Solution:
19. Out of 9 persons, 8 persons spent Rs. Let their prices be 3x, 5x and 7x.
30 each for their meals. The ninth one
spent Rs. 20 more than the average Then, 3x + 5x + 7x = 15000
expenditure of all the nine. The total
money spent by all of them was :  x = 3000.

A) Rs. 292.50 B) Rs. 297.50 Cost of cheapest item = 3x = Rs. 9000.

C) Rs. 298 D) Rs. 298.50


Solution: 22. The average weight of a group of boys
is 30 kg. After a boy of weight 35 kg joins
Let the average expenditure be Rs. x the group, the average weight of the group
goes up by 1 kg. Find the number of boys
Then, sum = 9x in the group originally?
=> 9x = [ (8×30) + (x + 20)] A) 6 B) 8
=> x =32.50 C) 4 D) 5
Total money spent = 9x = Rs. (9 x 32.50) Solution:
= Rs 292. 50
Let the number of boys in the group
originally be x.
20. In an examination, a pupil's average Total weight of the boys = 30x
marks were 63 per paper. If he had
obtained 20 more marks for his After the boy weighing 35 kg joins the
Geography paper and 2 more marks for group, total weight of boys = 30x + 35
his History paper, his average per paper
would have been 65. How many papers So, 30x + 35 = 31(x + 1)
were there in the examination?  x = 4.

pg. 47
10 Seconds

23. 12 boys decided to contribute Rs. 750 Total of 100 observations = 40 x 100 =
each to an Orphanage. Suddenly few of 4000
them boys dropped out and consequently
the rest had to pay Rs. 150 more. Then => 84 is misread as 48
the number of boys who dropped out? => Difference = 84 - 48 = 36
A) 4 B) 6 => Now, new total of 100 observations =
C) 2 D) 3 4000 + 36 = 4036
4036
Solution: Correct Mean = 100
= 40.36

Total money decided to contribute = 750 x


12 = 9000
26. The average of marks in 3 subjects is
Let 'b' boys dropped 224. The first subject marks is twice the
second and the second subject marks is
The rest paid 150 rupees more
twice the third. Find the second subject
=> (12 - b) x 900 = 9000 marks?

=> b = 2 A) 384 B) 96

Hence, the number of boys who dropped C) 192 D) 206


out is 2.
Solution:
Let the third subject marks be 'x'
24. 19 friends went to a restaurant for a
Second subject marks = 2x
combined weekend dinner party. 13 of
them spent Rs. 79 each on their dinner Third subject marks = 4x
and the rest spent Rs. 4 more than the
average expenditure of all the 19. What Given average = 224
was the total money spent by them? x + 2x + 4x = 224 x 3
A) Rs. 1398.96 B) Rs. 1457.09 7x = 224 x 3
C) Rs. 1662.35 D) Rs. 1536.07 => x = 96
Solution: Hence, Second subject marks = 2x = 2 x
Let average expenditure was Rs. x 96 = 192.

13 x 79 + 6(x + 4) = 19x
=> x= Rs. 80.84 27. The average age of the district level
hockey team of eleven is 22 years. The
Total money = 19 x 80.84 = Rs. 1536.07. average age gets increased by 1 year
when the coach age is also included.
What is the age of the coach?
25. The mean of 100 observations is 40. It
A) 33 years B) 34 years
is found that an observation 84 was
misread as 48. Then the correct mean is? C) 35 years D) 36 years
A) 40.36 B) 41.24 Solution:
C) 41.92 D) 42.05 When N = 11, sum = 11 × 22 = 242 years.
Solution: When N = 12, sum = 12 × 23 = 276 years.
Given, mean of 100 observations = 40 Therefore, the weight of the included
person = 276 – 242 = 34 years.

pg. 48
10 Seconds

Alternatively, Latest number = x + 6 = 23.


Here one person is including and the
average is increasing, so the person`s age
is more than the original average 22 and 30. The average of 20 numbers is zero. Of
difference in the average = 1. them, at the most, how many may be
greater than zero?
Number of players after including = 12
A) 0 B) -1
So, 22 + 1 × 12 = 34 years.
C) 1 D) none of these
Solution:
28. The average of runs of a cricket player
of 10 innings was 32. How many runs A) 0
must he make in his next innings so as to Explanation:
increase his average of runs by 4?
Average of 20 numbers = 0.
A) 76 B) 79
Sum of 20 numbers = (0 * 20) = 0.
C) 85 D) 87
It is quite possible that 19 of these
Solution: numbers may be positive and if their sum
A) 76 is a, then 20th number is (- a).

Explanation:
Average = total runs / no.of innings = 32 CODING AND DECODING

So, total = Average x no.of innings = 32 x Basics


10 = 320. A code is ‘a system of signals’. Coding is,
Now increase in avg = 4runs. So, new avg therefore, a method of transmitting a
= 32+4 = 36runs message between sender and receiver
which cannot be understood or
Total runs = new avg x new no. of innings comprehended by a third person. The
= 36 x 11 = 396 Coding and Decoding test is mainly to
judge the test-taker's ability to decipher a
Runs made in the 11th inning = 396 - 320 particular word/message by breaking the
= 76 code or decoding the same.
Questions:
29. The average of 7 consecutive numbers 1. If Z = 52 and ACT = 48, then BAT will
is 20. The largest of these numbers is : be equal to
A) 21 B) 22 a) 39 b) 41
C) 23 D) 24 c) 44 d) 46
Solution: Solution:
C) 23 In the given code, A = 2, B = 4, C = 6,.... ,
Explanation: Z = 52.

Let the numbers be x, x + 1, x + 2, x + 3, So, ACT = 2 + 6 + 40 = 48 and


x + 4, x + 5 and x + 6, BAT = 4 + 2 + 40 = 46
Then (x + (x + 1) + (x + 2) + (x + 3) + (x +
4) + (x + 5) + (x + 6)) / 7 = 20.
or 7x + 21 = 140 or 7x = 119 or x =17.

pg. 49
10 Seconds

2. If FRIEND is coded as HUMJTK, how is Solution:


CANDLE written in that code?
Code : Alternate letters forward and
a) EDRIRL b) DCQHQK backward.
c) ESJFME d) DEQJQM Therefore, BROWSER = CQPVTDS
Solution:
Code :The first, second, third, fourth, fifth 6. In a certain language ‘ITNIETAM’ is
and sixth letters of the word are written as ‘INTIMATE’. How would
respectively moved two, three, four, five, ‘TREVNIETARBI’ be written in the same
six and seven steps forward to obtain the language?
corresponding letters of the code.
a) INVERTIBRATE b) INVERTBARTE
c) INVERTBTREA d) INTERBRATE
3. In a certain code, MONKEY is written
as XDJMNL. How is TIGER written in that Solution:
code? Code : Reverse first half letters and last
a) SHFDQ b) HFDSQ half letters.

c) RSAFD d) QDFHS Therefore, TREVNIETARBI =


INVERTIBRATE
Solution:
Code :The letter of the word are written in
a reverse order and then each letter is 7. In a certain language ‘GIVE’ is written
moved one step backward to obtain the as ‘VIEG’. How would ‘DISK’ be written in
code. the same language?
a) SIDK b) KISD

4. If train is called bus, bus is called c) KDSI d) SIKD


tractor, tractor is called car, car is called Solution:
scooter, scooter is called bicycle, bicycle
is called moped, which is used to plough a Code : First letter => last place, second
field? letter => no change
a) Train b) Bus Third letter => first place and
fourth letter => third place
c) Tractor d) Car
Therefore, DISK = SIKD
Solution:
'Tractor' is used to plough a field.
8. In a certain language ‘BAT’ is written as
But a 'tractor' is called 'car'. ‘YZG’. How would ‘SICK’ be written in the
So, a 'car' will be used to plough the field. same language?
a) HRYV b) HRZP

5. In a certain language ‘PICTURE’ is c) HRXP d) RHPX


written as ‘QHDSVQF’. How would Solution:
‘BROWSER’ be written in the same
language? BAT = YZG
a) CQVVTDS b) CQPVTDS Code : B + Y = 2 + 25 = 27, A+Z=1+
26 = 27
c) CQPUTDS d) CQVPPDS
and T + G = 20 + 7 = 27

pg. 50
10 Seconds

Similarly, SICK = HRXP W = 4, O = 12, R = 9, K = 16


S + H = 19 +8 = 27 This is in the form of backward numbering,
I + R = 9 + 18 = 27 Therefore, WOMAN = 4-12-14-26-13
C + X = 3 + 24 = 27
K+ P = 11 + 16 = 27 12. If ‘A’ = 1 , ‘ACE’ = 9, then ‘ART’ = ?
a) 14 b) 24
9. In a certain code language, ‘STARK’ is c) 39 d) 15
written as ‘LBFMG’ and ‘MOBILE’ is
written as ‘TNRSPJ’. How will ‘BLAME’ be Solution:
written If A = 1 and ACE = 9 => A + C + E = 1 + 3
a) TSFRJ b) RPFTJ +5=9

c) NJFTP d) TSFGJ Then, ART = 1 + 18 + 20 = 39

Solution:
STARK = LBFMG and MOBILE = 13. If ‘CAT’ = 12, then ‘MAN’ = ?
TNRSPJ a)14 b) 24
So, S = L, T = B, A = F, R = M, K = G c)16 d)15
And M = T, O = N, B = R, I = S, L = P, E = Solution:
J
If CAT = 12 => C + A + T = 3 + 1 + 20 =
Therefore, BLAME = RPFTJ 24
24 = = 12
2

So, MAN = M + A + N = 13 + 1 + 14 = 28
10. In a certain code language, ‘SHEEP’ is 28
= 2 = 14
written as ‘GAXXR’ and ‘BLEAT’ is written
as ‘HPXTN’. How will ‘SLATE’ be written
a) GPTNX b) GPTXN 14.In a certain code language, ‘it pit sit’
c) GPXNT d) PTGXN means ‘I am boy’ and ‘it nit sit’ means ‘I
am girl’. Then which word represents
Solution: ‘girl’?
SHEEP = GAXXR and BLEAT = HPXTN a) it b) pit
So, S = G, H = A, E = X, E= X, P = R c) sit d) nit
And B = H, L = P, E = X, A = T, T = N Solution:
Therefore, SLATE = GPTNX It pit sit = I am boy →1
It nit sit = I am girl →2
11. If ‘WORK’ is coded as ‘4-12-9-16’, From 1 and 2
then how will you code ‘WOMAN’?
I means it and sit means i and am, boy
a)4-12-14-26-13 b) 4-12-14-26-12 means pit, and girl means nit
c) 4-12-14-26-11 d) 4-12-14-26-10
Solution: 15. In a certain code language,
WORK = 4-12-9-16 ‘7 8 6’ means ‘Study very hard’

pg. 51
10 Seconds

‘9 5 8’ means ‘hard work pays’ ‘orange’, then what would be colour of


human blood?
‘6 4 5’ means ‘Study and Work’
a) Red b) Green
Then which number represents ‘Very’?
c) Yellow d) Violet
a) 8 b) 6
Solution:
c) 7 d) 5
The colour of human blood is red.
Solution:
But here red is called yellow so, the
‘7 8 6’ means ‘Study very hard’ → 1 answer is yellow.
‘9 5 8’ means ‘hard work pays’ → 2
‘6 4 5’ means ‘Study and Work’ 18. If ‘parrot’ is ‘peacock’, ‘peacock’ is
→3 ‘swallow’, ‘pigeon’ is ‘sparrow’, then what
Compare 1 and 2 => 6 means study, is the name of Indian National Bird?

Compare 2 and 1 => 8 means hard, a) Parrot b) Peacock

Therefore, 7 means ‘very’. c) Swallow d) Sparrow


Solution:

16)In a certain code language , Indian national bird is peacock.

‘po ki top ma’ means ‘Usha is playing But here peacock is called swallow , so
cards’ the answer is swallow.

‘kop ja ki ma’ means ‘Asha is playing


tennis’ 19. If ‘orange’ is called ‘butter’, ‘butter’ is
‘ki top sop ho’ means ‘they are playing called ‘soap’, ‘soap’ is called ‘ink’, ‘ink’ is
football’ called ‘honey’ and ‘honey’ is called
‘orange’, then which is used for washing
‘po sur kop’ means ‘cards and tennis’ clothes?
Then which word represents ‘Asha’? a) soap b) Ink
a)ja b)ma c) honey d) orange
c)kop d)top Solution:
Solution : For washing clothes we use soap.
Here ‘Asha’ is present in second But here soap is called ink, so the answer
statement. is ink.
Comparing 2 and 4 we get kop = tennis
Comparing 2 and 3 we get ki = playing 20. If E = 5 and READ is coded as 7, then
what is the code of 'DEAR'?
Comparing 2 and 1 we get ma = is
a) 6 b) 7
Therefore ‘Asha’ means ‘ja’
c) 8 d) 9
Solution:
17. If ‘white’ is called ‘blue’, ‘blue’ is called
‘red’, ‘red’ is called ‘yellow’, ‘yellow’ is 5
Here, E = 5 = 1 = 5
called ‘green’, ‘green’ is called ‘black’,
‘black’ is called ‘violet’ and ‘violet’ is called

pg. 52
10 Seconds

28 (B) ‘dar tok pa’ means ‘good and bad’


=>READ = 18 + 5 + 1 + 4 = =7
4
28 (C) ‘tim na tok’ means ‘they are bad’
=>DEAR = 4 + 5 + 1 + 18 = =7
4
In that language, which word stands for
'they' ?
21. In a certain code 'Black' means a) na b) tok
Orange', 'Orange' means Violet','Violet'
means 'Green', 'Green' means 'White' c) tim d) pit
,'White', means 'Yellow' and 'Yellow' Solution:
means 'Sky blue', what is the colour of
coal? Code : In the first and third statements, the
common word is 'na' and the common
a) Orange b) Sky Blue word is 'are'.
c) Violet d) Yellow So, 'na' means 'are'.
Solution: In the second and third statements, the
Code : The colour of coal is Black. In the common code word is 'tok' and the
code language, black is called orange. So, common word is 'bad'.
the colour of coal is Orange. So, 'tok' means 'bad'.
Thus, in the third statements, 'tim' stands
22. If LBAEHC is the code for BLEACH for 'they'.
then which of the following is coded as
NBOLZKMH?
25. In a certain code language,
a) OBNKZLHM b) LOBNHMKZ
'134' means 'good and tasty';
c) OCPMALNI d) BNLOKZHM
'478' means 'see good pictures' and
Solution:
'729' means 'pictures are faint'.
Code : The word if formed into pairs of
letters and the letters in each pair are Which of the following digits stands for
reversed. 'see'?
a) 9 b) 2
23. If in a certain language FASHION is c) 1 d) 8
coded as FOIHSAN, how is PROBLEM
coded in that language? Solution:

a) PELBORM b) PRBOELM Code : In the first and second statements,


the common code digit is '4' and the
c) RPBOELM d) RPBOELM common word is 'good'. So, '4' stands for
'good'.
Solution:
In the second and third statements, the
Code :The first and the last letters of the common code digit is '7' and the common
word remain as such and the remaining word is 'pictures'. So, '7' means 'pictures'.
letters are written in a reverse order, to
obtain the code. Thus, in the second statements, '8' means
'see'.

24. In a certain code language,


(A) ‘pit dar na’ means ‘you are good’

pg. 53
10 Seconds

26. In certain code 'TIGER' is written as Man would sleep on BED,


'QDFHS'. How is 'FISH' written in that
code? But here, BED is called WINDOW.

a) GERH b) GRHE Therefore, the answer is WINDOW.

c) GREH d) GHRE
Solution: 30. In a certain code, '247' means 'spread
red carpet' ; '256' means 'dust one carpet'
Code : Reverse the word and move each and '234' means 'one red carpet'. Which
letter by –1. Reverse of FISH is HSIF digit in that code means 'dust'?
subtract 1 from each letter of HSIF. So
code of FISH become GRHE. A) 2 B) 3
C) 5 D) 6

27. In a certain code 'ROAR' is written as Solution:


'URDU'. How is 'URDU' written in that C) 5
code?
Explanation:
a) V X D Q b) XUGX
In the first and second statements, the
c) ROAR d) VSOV common code digit is '2' and the common
Solution: word is 'carpet'.

Code : Each letter moves +3. Add 3 to So, '2' means 'carpet'. In the second and
each letter of URDU, so code of URDU third statements, the common code digit is
will be XUGX. '6' and the common word is 'one'.
So, '6' means 'one'.

28. If DELHI is coded as 73541 and Therefore, in the second statement, '5'
CALCUTTA as 82589662, how can means 'dust'.
CALICUT be coded?
a) 5279431 b)5978213
PARTNERSHIP
c)8251896 d)8543691
Basics
Solution:
1) Partnership: It refers to a business
Code : Direct substitution association between two or more than two
persons who run a business together and
DELHI = 73541 and CALCUTTA =
share the total profit at an agreed
82589662
proportion. The persons who enter into a
Therefore, CALICUT = 8251896 partnership are called partners.
2) Working partner: A partner who is
actively involved in the business and
29. If ROOM is called BED, BED is called manages the business is known as an
WINDOW, WINDOW is called FLOWER active partner.
and FLOWER is called COOLER, on what
would a man sleep? 3) Sleeping partner: A partner who invests
money but does not involve or look after
a) WINDOW b) BED the business is known as a sleeping
c) FLOWER d) COOLER partner.

Solution:

pg. 54
10 Seconds

4) Simple partnership: It refers to a = 15 : 16 : 31


partnership in which each partner invests
capital for the same period.
5) Compound partnership: It refers to a 3. In a business, ‘M’ and ‘N’ invested
partnership in which the partners invest amounts in the ratio 1 : 2, whereas ratio of
capital for different periods. amounts invested by ‘M’ and ‘P’ is 1 : 3. If
Rs 57300 was their profit, how much
Questions: amount did ‘N’ receive?
1. ’X’, ‘Y’ and ‘Z’ started a business by a)12100 b)34000
investing Rs 130000, Rs 140000 and Rs
150000 respectively for one year. Find the c)19100 d)50000
share of ‘X’ out of total annual profit of Rs Solution:
56700.
M:N=1:2 and M : P= 1 : 3
a)17010 b)17550
M:N:P=1:2:3
c)20000 d)15000
2
N's share = 6 × 57300 = 19100Rs.
Solution:
Here we can use the formula, profit =
investment × time 4. ’X’, Y’, ‘Z’ invests Rs 50000 for a
X : Y : Z = 130000 : 140000 : 150000 = 13 business. ‘X’ invests Rs 4000 more than
: 14 : 15 ‘Y’ and ‘Y’ invests Rs 5000 more than ‘Z’.
Out of total profit of Rs 35000, ‘X’
13
Share of X = 56700 × 42 = 17550 receives?
a)14700 b)15700
14
Share of Y = 56700 × 42
= 18900
c)17700 d)44000
15
Share of Z = 56700 × = 20250 Solution:
42
Let Z invests = c.
Therefore, the share of X is 17550.
Then, Y = c + 5000 and
X = c + 5000 + 4000 = c + 9000.
2. John, Thomas and William enter a
partnership. They invest Rs 50000, Rs So, c + c + 5000 + c + 9000 = 50000
80000 and Rs 130000 respectively. At the
end of 1st year ‘Thomas’ withdraws Rs 3c = 36000
40000 while at the end of 2nd year ‘William’
c = 12000
withdraws Rs 80000. In what ratio will the
profit be shared at the end of 3 years? X : Y : Z = 21000 : 17000 : 12000 = 21 :
17 : 12.
a)12 : 14 : 16 b)11 : 21 : 41
21
c)15 : 16 : 31 d)20 : 31 : 40 X's share = Rs. (35000 × 50
) = Rs. 14,700.

Solution:
J : T : W = (50000 × 36) : (80000 × 12 + 5. ’A’, ‘B’ and ‘C’ started a business by
40000 × 24) : (130000 × 24 + 50000 × 12) investing Rs 120000, Rs 135000 and Rs
150000 respectively for one year. Find the
= 1800000 : 1920000 : 3720000 share of ‘A’ out of total annual profit of Rs
= 180 : 192 : 372 56700.

= 30 : 32 : 62 a)17010 b)16800

pg. 55
10 Seconds

c)20000 d)15000 c)7500 d)10000


Solution: Solution:
Here we can use the formula, profit = A's share = 20000 × 24 = 480000
investment × time
A : B : C = 120000 : 135000 : 150000 B's share = 15000 × 24 = 360000
= 120 : 135 : 150 C's share = 20000 × 18 = 360000
= 8 : 9 : 10 Ratio of A, B, C 's share = 480000 :
8 360000 : 360000
Share of A = 56700 × 27 = 16800
9
Share of B = 56700 × = 18900 = 48 : 36 : 36
27
10
Share of C = 56700 × 27
= 21000 =4:3:3

Therefore, the share of A is 16800. Total ratio = 4 + 3 + 3 = 10


3
B's share = 10 × 2500 = 7500Rs
6. Alfred started a business investing Rs
45000. After 3 months Peter joined him
with a capital of Rs 60000. After another 6 8. ’A’, ‘B’ and ‘C’ started a business each
months Ronald joined them with a capital investing Rs 20000. After 5 months ‘A’
of Rs 90000. At the end of the year, they withdrew Rs 5000, ‘B’ withdrew Rs 4000
made a profit of Rs 16500. Find the share and ‘C’ invests Rs 6000 more. At the end
of Peter. of the year a total profit of Rs 69900 was
recorded. Find the share of ‘A’?
a)6600 b)6900
a)21200 b)20500
c)3300 d)4400
c)28200 d)16000
Solution:
Solution:
Here we can use the formula, profit =
investment × time Ratio of the capitals of A, B and C
Ratio of the share of Alfred, Peter and = 20000 × 5 + 15000 × 7 : 20000 × 5 +
Ronald is 16000 × 7 : 20000 × 5 + 26000 × 7
45000 × 12 : 60000 × 9 : 27000 × 3 = 2 : 2 = 205000 : 212000 : 282000
:1
= 205 : 212 : 282
2
Share of Alfred = 5
× 16500 = 6600 205
A's share = Rs. (69900 × 699
) = Rs.20500
2
Share of Peter = × 16500 = 6600
5 212
B's share = Rs. (69900 × 699
) = Rs.21200
Share of Ronald = 3300
282
Therefore, the share of Peter = 6600. C's share = Rs. (69900 × 699
) = Rs.28200

7. A’ and ‘B’ started a business investing


9. A, B and C enter a partnership. They
Rs. 20000 and Rs. 15000 respectively.
invest Rs 40000, Rs 80000 and Rs
After 6 months ‘C’ joined them with Rs.
120000 respectively. At the end of 1st year
20000. What will be B’s share in total profit
‘B’ withdraws Rs 40000 while at the end of
of Rs.25000 earned at the end of 2 years
2nd year ‘C’ withdraws Rs 80000. In what
from starting of business?
ratio will the profit be shared at the end of
a)25000 b)5000 3 years?

pg. 56
10 Seconds

a)1 : 1 : 1 b)1 : 2 : 4 = 36 : 60 : 30
c)3 : 4 : 7 d)2 : 3 : 4 = 6 : 10 : 5
Solution:
A : B : C = (40000 × 36) : (80000 × 12 + 12. ’A’, ‘B’ and ‘C’ enter into a partnership
40000 × 24) : (120000 × 24 + 40000 × 12) 1 1
and their investments are in the ratio 2 : 3 :
1
= 1440000 : 1920000 : 3360000 . After 2 months, ‘A’ withdraws half of his
4
= 144 : 192 : 336 capital and after 10 months a profit of Rs
378 is divided among them. What is ‘B’s
= 12 : 16 : 28 share?
=3:4:7 a)100 b)122
c)144 d)none
10. In a business, ‘A’ and ‘C’ invested Solution:
amounts in the ratio 2:1, whereas ratio of 1 1 1
amounts invested by ‘A’ and ‘B’ is 3:2. If Ratio of initial investments = : : = 6 :
2 3 4
Rs 157300 was their profit, how much 4 : 3.
amount did ‘B’ receive? Let their initial investments be 6x, 2x and
3x respectively.
a)12100 b)34000
A : B : C = (6x × 2 + 3x × 10) : (4x × 12) :
c)48400 d)50000 (3x × 12)
= 42 : 48 : 36
Solution: = 7: 8 : 6.
8
A : B = 3 : 2 = 6 : 4 and A : C = 2 : 1 = 6 : B's share = Rs. (378 × ) = Rs. 144.
21
3
A:B:C=6:4:3
13. ’A’ began a business with Rs 85000.
B's share =
4
× 157300 = 48400Rs. He was joined afterwards by ‘B’ with Rs
13 42500. For how much period did ‘B’ join if
the profits at the end of year are divided in
the ratio of 3:1?
11. ’A’ and ‘B’ started a partnership a)6 months b)2 months
investing some amount in the ratio of 3:5.
‘C’ joined them after 6 months with an c)8months d)none
amount equal to that of ‘B’. In what ratio Solution:
should the profit be distributed at the end
of year? Suppose B joined for x months.
a)2 : 3 : 4 b)2 : 6 : 7 85000×12 3
Then, 42500×𝑥
= 1
c)6 : 10 : 5 d)none
So, x = 8
Solution:
So, B joined for 8months.
Let the initial investment of A is 3x and B
is 5x, then C investment is also 5x, but
most important to note in this question is 14. ’A’, ‘B’, ‘C’ enters into a partnership. ‘A’
the time duration of investment. invests 3 times as much as ‘B’ invests and
‘B’ invests 2/3rd of what ‘C’ invests. At the
Like, A invested for 12 months, B invested
end of the year the profit earned was Rs
for 12 months and C invested for 6
6600. What is the share of ‘B’?
months.
a)1000 b)1200
A : B : C = (3x × 12) : (5x × 12) : (5x × 6)

pg. 57
10 Seconds

c)1400 d)1600 Let 4A = 6B = 10C = k.


Solution: 𝑘 𝑘 𝑘
Then, A = 4, B = 6 and C = 10 .
Let C invest x Rs
𝑘 𝑘 𝑘
2 A:B:C= : : = 15 : 10 : 6
Then according to the question B invest = 3x 4 6 10
Rs 6
Hence, C's share (4650 × 31) = Rs 900.
2
A invest = 3x × 3 = 2x Rs.
2
Then x + x + 2x = 6600 17. Four milkman rented a pasture. ‘A’
3
grazed 24 cows for 3 months, ‘B’ grazed
 3x + 2x + 6x = 19800 10 cows for 5 months, ‘C’ grazed 35 cows
 11x = 19800 for 4 months and ‘D’ grazed 21 cows for 3
 x = 1800 months. If ‘A’s share of rent is Rs 720, find
 Then B's share = 32 × 1800 = the total rent?
1200Rs.
a)2250 b)3250
c)4250 d)5250
15. ’A’, B’, ‘C’ invests Rs 50000 for a
Solution:
business. ‘A’ invests Rs 4000 more than
‘B’ and ‘B’ invests Rs 5000 more than ‘C’. Ratio of A, B, C, D 's share = (24 × 3) : (10
Out of total profit of Rs 35000, ‘A’ × 5) : (35 × 4) : (21 × 3)
receives?
= 72 : 50 : 140 : 63
a)14700 b)15700
= 72 + 50 + 140 + 63 = 325
c)17700 d)44000
Let total rent is x
Solution:
72𝑥
Then A's share = 325
Let C = x.
72𝑥
Then, B = x + 5000 and Therefore, = 720
325

A = x + 5000 + 4000 = x + 9000. x = 3250


So, x + x + 5000 + x + 9000 = 50000 Total rent = 3250
3x = 36000
X = 12000 18. Three milkman rented a pasture. ‘A’
grazed 10 cows for 7 months, ‘B’ grazed
A : B : C = 21000 : 17000 : 12000 = 21 :
12 cows for 5 months and ‘C’ grazed 15
17 : 12.
cows for 3 months. If the rent of the
A's share = Rs. (35000 ×
21
) = Rs. 14,700. pasture is Rs 175, what is the share of ‘C’
50 in the rent?
a)41 b)52
16. If 4 times of ‘A’ capital is equal to ‘6’ c)63 d)74
times of ‘B’s capital and 10 times of ‘C’s
capital. Then out of a profit of Rs 4650, ‘C’ Solution:
receives?
Ratio of A, B, C share = (10 × 7) : (12 × 5)
a)700 b)900 : (15 × 3)
c)500 d)440 = 70 : 60 : 45
Solution: = 14 : 12 : 9

pg. 58
10 Seconds

= 14 + 12 + 9 = 25 Munna invested Rs.60000 for 6 months,


whereas Raju invested his amount for the
The total rent = 175 whole year, what was the amount invested
175 by Raju?
Then C`s share is 25
= 7
A. Rs. 12000 B. Rs. 15000
Therefore, C`s rent = 9 × 7 = 63
C. Rs. 18000 D. Rs. 50000
Solution:
19. Joey started a business and he
invested in 38000, After some month, Suppose Raju invested Rs. x. Then, Ratio
Amar came to join with him and invest of their investments will be:
28500.The end of the year the total profit
Munna : Raju = 60000 × 6 : x × 12.
was divided among them into ratio form
16: 8. Find after how many months did 360000 : 12x = 3000 : 1500
Amar join.
x = 15000
A. 4 B. 6
Therefore, Raju invested 15000 rupees.
C. 8 D. 5
Solution:
22. Rs.1400 is divided among Amla, Bimla
We can assume that Amar joined into and Simla so that Amla receives half as
business after x months. So, Amar`s much as Bimla and Bimla half as much as
money was invested for (12 – x) months. Simla. Then Simla's share is
38000 x 12 : 28500 x (12 – x) = 2 : 1 A. Rs. 200 B. Rs. 300
456000 : 28500 (12 – x) = 2 : 1 C. Rs. 800 D. Rs. 600
Therefore, x = 4 Solution:
After 4 months Amar join the business. Let Amla's share = Rs. X
Bimla's share = Rs.2x
20. A, B and C started a business by Simla's share = Rs.4x
investing Rs. 250000 Rs. 300000 and
Rs.350000 respectively. Find the share of Amla:Bimla:Simla = x : 2x : 4x = 1 : 2 : 4
B, out of an annual profit of Rs.187200. 4
Hence, Simla's share = 1400 × 7
= Rs.800
A. Rs. 65400 B. Rs. 62400
C. Rs. 63400 D. Rs. 66200
23. A and B invest in a business in the
Solution: ratio 3 : 2. If 5% of the total profit goes to
Ratio of shares of A, B and C = Ratio of charity and A's share is Rs. 855, the total
their investment profit is:

A : B : C = 250000 : 300000 : 350000 = 5 : A.Rs. 1425 B.Rs. 1500


6:7 C.Rs. 1537.50 D.Rs. 1576
6
Share of B = Rs. [187200 x 18
] = 62400. Solution:
Let the total profit be Rs. 100.

21. Munna received Rs. 3000 as his share After paying to charity, A's share = Rs.95
3
out of the total profit of Rs. 4500 which he × 5 = 57
and Raju earned at the end of one year. If

pg. 59
10 Seconds

If A's share is Rs. 57, total profit = Rs. 26. A, B, C rent a pasture. A puts 10 oxen
100. for 7 months, B puts 12 oxen for 5 months
100
and C puts 15 oxen for 3 months for
If A's share is Rs. 855, total profit = x grazing. If the rent of the pasture is Rs.
57
855 = 1500. 175, how much must C pay as his share of
rent?
A. Rs. 45 B. Rs. 50
24. A, B and C jointly thought of engaging
themselves in a business venture. It was C. Rs. 55 D. Rs. 60
agreed that A would invest Rs. 6500 for 6
Solution:
months, B, Rs. 8400 for 5 months and C,
Rs. 10,000 for 3 months. A want to be the A : B : C = (10 × 7) : (12 × 5) : (15 × 3)
working member for which, he was to
receive 5% of the profits. The profit earned = 70 : 60 : 45
was Rs. 7400. Calculate the share of B in = 14 : 12 : 9
the profit.
9
A.Rs. 1900 B.Rs. 2660 C's rent = Rs. (175 × 35) = Rs. 45.

C.Rs. 2800 D.Rs. 2840


Solution: 27. If 4 (A's capital) = 6 (B's capital) = 10
(C's capital), then out of a profit of Rs.
For managing, A received = 5% of Rs. 4650, C will receive?
7400 = Rs. 370.
A) Rs.700 B) Rs.800
Balance = Rs. (7400 - 370) = Rs. 7030.
C) Rs.900 D) Rs.1000
Ratio of their investments = (6500 x 6) :
(8400 x 5) : (10000 x 3) Solution:
𝑘
= 39000 : 42000 : 30000 Let 4A = 6B = 10C = k. Then, A = 4, B =
𝑘 𝑘
= 13 : 14 : 10 , and C =
6 10
14 𝑘 𝑘 𝑘
B's share = Rs.7030 x 37 = Rs. 2660. A:B:C= : :
4 6 10

= 15 : 10 : 6
25. Arun, Kamal and Vinay invested Rs. 6
Hence, C's share = (4650 × 31) = Rs, 900.
8000, Rs. 4000 and Rs. 8000 respectively
in a business. Arun left after six months. If
after eight months, there was a gain of Rs.
4005, then what will be the share of 28. A and B began business with Rs.3000
Kamal? and Rs.4000 after 8 months, A withdraws
Rs.1000 and B advances Rs.1000 more.
A.Rs. 890 B.Rs. 1335 At the end of the year, their profits
C.Rs. 1602 D.Rs. 1780 amounted to Rs.630 find the share of A?

Solution: A) Rs. 240 B) Rs. 75

Arun : Kamal : Vinay = (8,000 x 6) : (4,000 C) Rs. 125 D) Rs. 354


x 8) : (8,000 x 8) Solution:
= 48 : 32 : 64 The ratio of A & B investments = (3 x 8 + 2
= 3 : 2 : 4. x 4) : (4 x 8 + 5 x 4) = 8 :13
8
2
Kamal's share = Rs.4005 x 9 = Rs. 890. A’s share = 21 x 630 = 240.

pg. 60
10 Seconds

Therefore, the share of A is 240. Rakesh's share = Rs.(2540 x 42/127) =


Rs.840

29. Ram and Raj start a business with


investments of Rs. 2500 and Rs. 2250 30. A and B invests Rs.10000 each, A
respectively. After 2 months, Ram takes investing for 8 months and B investing for
out Rs.1250 from his capital. After 1 more all the 12 months in the year. If the total
month, Raj takes out Rs.750 of his capital profit at the end of the year is Rs.25000,
while Rakesh joins them with a capital of find their shares?
Rs. 3500. At the end of 6 months, they
earn a profit of Rs. 2540. Which of the A) 10000 and 15000 B) 15000 and 10000
following is the share of each member in C) 5000 and 20000 D) 20000 and 5000
the profit?
Solution:
A) Ram - Rs. 800, Raj - Rs. 900, Rakesh
- Rs. 840 A) 10000 and 15000
B) Ram - Rs. 900, Raj - Rs. 800, Rakesh - Explanation:
Rs. 840
As both A and B invest the same amounts,
C) Ram - Rs. 840, Raj - Rs. 920, Rakesh - the ratio of their profits at the end of the
Rs. 840 year is equal to the ratio of the time
periods for which they have invested.
D) Ram - Rs. 800, Raj - Rs. 900, Rakesh -
Rs. 940 Thus, the required ratio of their profits = A
: B = 8 : 12 = 2 : 3.
Solution:
Hence, share of A in the total profit = 2 x
Initial investment of Ram = Rs.2500. 25000/5 = Rs.10000
After 2 months he withdraw Rs.1250 from Similarly, share of B in the total profit = 3 x
his capital. 25000/5 = Rs.15000
Therefore, we have, Ram invested
Rs.2500 for 2 months and Rs.(2500-
1250=) 1250 for 4 months. PROFIT AND LOSS
Raj invested Rs. 2250 for 3 months and Formulas
Rs.(2250-750=) 1500 for 3 months.
1)Profit, P = SP – CP; SP>CP
And, Rakesh invested Rs.3500 for 3
2)Loss, L = CP – SP; CP>SP
months;
3)P% = (P/CP) x 100
Their investing ratio:
4)L% = (L/CP) x 100
Ram:Raj:Rakesh = (2500x2 +
1250x4):(2250x3 + 1500x3):(3500x3) 5)SP = {(100 + P%)/100} x CP
= (10,000):(11,250):(10,500) = 6)SP = {(100 – L%)/100} x CP
1000:1125:1050 = 40:45:42
7)CP = {100/(100 + P%)} x SP
Total profit for 6 months = Rs.2540
8)CP = {100/(100 – L%)} x SP
Therefore, Ram's share = Rs.(2540 x
40/(40+45+42)) = Rs.(2540 x 40/127) = 9)Discount = MP – SP
Rs.800 10)SP = MP -Discount
Raj's share = Rs.(2540 x 45/127) = 11)For false weight, profit percentage will
Rs.900 be P% = [(True weight – false weight)/
false weight] x 100.

pg. 61
10 Seconds

12)When there are two successful profits, a)360 b)425


say m% and n%, then the net percentage
profit equals to [m+n+(mn/100)] c)450 d)475
13)When the profit is m%, and loss is n%,
then the net % profit or loss will be: [m-n- Solution :
(mn/100)]
Cost price of Karan = Rs 300
14)If a product is sold at m% profit and
then again sold at n% profit then the Cost price of Rohan = Rs 300 + 300 × 0.2
actual cost price of the product will be: CP = Rs 360
= [100 x 100 x P/(100+m)(100+n)]. In case
of loss, CP = [100 x 100 x L/(100-m)(100- Cost price of Karan = Rs 360 + 360 × 0.25
n)] = Rs 450
15)If P% and L% are equal then, P = L
and %loss = P2/100
Questions: 4)3 successive discount of 8% amount to
a single discount of
1)By selling an article for Rs.80 a man
gains Rs.20 . Find his profit% a)20.35% b)22.13%

a)10% b)15% c)18.67% d)19%

c)33.33% d)25% Solution :

Solution : Let price of the article be 100

Here S.P = 80 and P = 20 which gives First discount of 8% = 8


C.P = 60
Second discount of 8% = 7.36
P
We know that Profit % = C.P × 100
Third discount of 8% = 6.77
20
 Profit % = 60
× 100 = 33.33 %
It amounts to a single discount of 22.13%

2)A shopkeeper sold a article at Rs 81 at


10% discount. What is the marked price of
5)By selling an article for Rs.120 a man
the article?
gains Rs.20 . Find his profit%
a)85 b)90
a)10% b)15%
c)95 d)100
c)20% d)25%
Solution :
Solution :
90% of M.P is 81, therefore 100% of M.P
Here S.P = 120 and P = 20 which gives
is 90.
C.P = 100
3)Aman bought a cycle for Rs 300. He P
sold to Rohan at 20% profit and Rohan in We know that Profit % = C.P × 100
turn sold the cycle at 25% to Karan. What
is the cost price of Karan?

pg. 62
10 Seconds

20 P
 Profit % = × 100 = 20% We know that Profit % = × 100
100 C.P

1
 Profit % = 3 × 100
6) Find C.P when profit=25 and gain
 Profit % = 33.33 %
percentage is 50%.

a)50 b)100
9)A man bought a machine for Rs.70000
and spent Rs.4000 on repair and Rs.1000
c)150 d)75 on transport and sold at a profit of 30%.
Find selling price
Solution :
a)97600 b)97500
Here P = 25 and P% = 50
c)98000 d)100000
P
We know that Profit % = C.P × 100
Solution :
25
 50 = × 100
C.P If C.P = 100, S.P = 130 at 30% profit.
 C.P = 50
Using cross multiplication if C.P is 75000,
S.P = 97500
7)The ratio of CP and SP is 5:4. Find
profit% or loss%

a)10 b)20
10) The cash difference between selling
price of an article at a profit of 4% and 6%
c)45 d)50 is Rs 3. Find the cost price
Solution : a)100 b)150
Here C.P = 5 , S.P = 4 and L = 1 c)175 d)200
L
We know that Loss % = × 100 Solution :
C.P

1 If C.P = 100, Difference between S.P of an


 Loss % = 5 × 100
article at 4% and 6% profit is 2.
 Loss % = 20%
If the difference is 3, then the C.P is 150.
8)If selling price of an article is 4/3rd of its
cost price, the profit % in the transaction is
11)A shopkeeper takes 20% profit on his
a)33.33% b)20%
goods. He lost 30% of goods during theft.
Find his profit or loss%.
c)25% d)50%
a)12 b)14
Solution :

S.P 4
c)16 d)21
Given C.P = 3
Solution :
Here C.P = 3 , S.P = 4 and P = 1

pg. 63
10 Seconds

Let us take C.P = 100, then remaining C.P c)60 d)70


after theft is 70.
Solution :
S.P at 20 % profit = 84.
Let C.P = 100, S.P = 136 at 15% discount
Therefore there is a loss of 16%.
85% of M.P is 136 and 100% of M.P is
160.

12)A trader marks his goods 40% above Therefore 60% is profit when no discount
CP and allowed a discount of 30% on is allowed.
marked price. Find profit or loss%

a)1 b)2
15)By selling 48 oranges a man loses SP
c)3 d)4 of 12 oranges. Find his loss%.

Solution : a)10% b)20%

Let C.P = 100 which gives M.P = 140. c)30% d)None

Discount = 30% of 140 = 42 Solution :

Therefore S.P = 98 and Loss is 2% Let 1 orange is 1 Re.

S.P = 48, L = 12 and C.P = 60

13)A shopkeeper marks his goods above L


We know that Loss % = × 100
C.P
cost price by 25%. What % of discount
allowed to get a profit of 5%? 12
 Loss % = × 100
60
a)14% b)16%  Loss % = 20%

c)18% d)20%
16)The C.P of 16 pens is equal to S.P of
Solution : 12 pens. Find its profit or loss%

Let C.P = 100 which gives M.P = 125 and a)25% b)50%
S.P = 105.
c)33.33% d)None
Here discount = 20
Solution :
20
Discount % = 125
× 100 = 16%
16 × C.P = 12 × S.P

C.P 12
 S.P
= 16
14)A shopkeeper sold an article at a
discount of 15% and got profit of 36%. Here C.P = 12 and S.P = 16 which gives
What is the profit% he earns if no Loss = 4
discount is allowed?
L
We know that Loss % = C.P × 100
a)40 b)50

pg. 64
10 Seconds

4 a)24% b)28%
 Loss % = × 100
12
 Loss % = 33.33 %
c)33% d)36%

17)A man sold a flat for Rs.22000 and loss Solution :


is 20%. At what price he have to sell the
flat to get a profit of 20%. Let price of the article be 100

a)33000 b)44000 1st discount of 10% is 10

c)55000 d)66000 2nd discount of 12% is approximately 11

Solution : 3rd discount of 15% is approximately 12

For C.P = 100, S.P = 80 at 20% loss and Therefore total single discount = 33%
S.P = 120 at 20% profit.

80  120
20) The difference b/w a discount of 35%
22000  ? and two successive discounts of 20% on a
certain bill was 22. Find the amount of bill
Therefore he should sell at 33000.
a)2200 b)2400

c)2600 d)2800
18)By selling 70 lemons for Rs.60 a man
loses 25%. How many should he sell for Solution :
Rs.40 to get profit of 25%?
Let price of the article be 100.
a)24 b)28
Single discount of 35% = 65 and two
c)32 d)36 successive discount of 20% is 64.

Solution : For price 100, the difference is 1. The


difference to become 22 price should be
For C.P = 100, S.P = 75 at 25% loss and 2200.
S.P = 125 at 25% profit.

75  125
21)’A’ sells a bicycle to ‘B’ at a profit of
60  ? 25%. ‘B’ sells it to ‘C’ at a profit of 30%
and ‘C’ pays Rs.260 for it. What is the CP
for ‘A’?
He should sell 70 lemons at 100 at 25%
profit.
a)150 b)160
Therefore for Rs 40 he should sell 28
lemons at 25% profit. c)170 d)180

Solution :

19)Successive discount of 10%, 12% and Let C.P of A is 100 which gives C.P of B is
15% amount to a single discount of 125 and C.P of C is 162.5

pg. 65
10 Seconds

If cost of A is 100, C pays 162.5 23)If selling price of 16 items is same as


the cost price of 20 items, then there is a
Since C is paying Rs 260, C.P of A is 160.
a)Loss of 20% b)Loss of 25%

c)Gain of 20% d)Gain of 25%


22)A man sells two flats at the rate of
150000 each. On one flat he gains 25% Solution :
and on other he loses 25%. Find his profit
or loss%. 16 × S.P = 20 × C.P

a)5% loss b)6.25% profit C.P 16


 S.P
= 20
c)5% Profit d)6.25% loss
Here C.P = 16 and S.P = 20 which gives
Gain = 4
Solution :
P
Let C.P of 1st flat = 100, then S.P of 1st flat We know that Gain % = C.P × 100
becomes 125.
4
 Gain % = 16 × 100
In the question S.P is 150000, therefore
 Gain % = 25 %
by cross multiplication we get C.P of 1st
flat = Rs. 120000
24)Marked price of a Doll is 35% above
Let C.P of 2nd flat = 100, then S.P of 2nd
the cost price. If he gives a discount of
flat becomes 75.
15%, how much he gains on the deal?
In the question S.P is 150000, therefore
a)11.75% b)12.75%
by cross multiplication we get C.P of 2nd
flat = Rs. 200000
c)13.75% d)14.75%
Total C.P = 320000
Solution :
Total S.P = 300000
Let C.P = 100 which gives M.P = 135
Loss = 20000
Discount = 15% of 135 = 20.25
Loss % = 6.25%
S.P = 135 - 20.25 = 114.75
Alternatively,
Therefore profit = 14.75%
In this type of question, the person will
always be at loss.

(Loss % ×Profit %) 25)Raj sold an item for Rs. 6,384/- and


Overall Loss % = 100 incurred a loss of 30%. At what price
should he have sold the item to have
(25 ×25) gained a profit of 30%?
Overall Loss % = 100
= 6.25%

a)11434 b)11856

c)12234 d)12967

pg. 66
10 Seconds

Solution : 8598
S.P of 1 grinder = = 191.06
45

For C.P = 100, S.P = 70 at 30% loss and


S.P = 130 at 30% profit.
28)If a boy sells a book for Rs. 450 he
70  130
gets a loss of 10 %. To gain 10%, what
should be the selling price?
6384  ?
a)500 b)600
Therefore he should sell at 11856.
c)550 d)525

Solution :
26)If on an item a company gives 25%
discount, they earn 25% profit. If they give
Let C.P = 100, then S.P = 90 at 10% loss
10% discount then what is the profit
and S.P = 110 at 10% profit
percentage?
90  110
a)20% b)30%
450  ?
c)40% d)50%
Therefore S.P required = Rs 550
Solution :

Let C.P = 100, then S.P at 25% discount =


125
29)A merchant sells 30 metres of cloth
and gains selling price of 10 metres. Find
That is 75% of M.P is 125
the gain percent.
100% of M.P is 166.67
a) 15 % b) 25 %
S.P at 10% discount is 150
c)50 % d)75%

Solution :
27)Sandeep bought 45 grinders for Rs.
Let 1 metre cloth is 1 Re.
2215, while carrying he incurred
transporting cost of Rs 2190. Again, Rs.
2760 were spend on packing these S.P = 30, G = 10 and C.P = 20
grinders into boxes. Find SP of each piece P
to make profit of 20%. We know that Gain % = C.P × 100

a)123 b)134 10
 Gain % = 20 × 100
d)172 d)191  Gain % = 50%

Solution :
30)A man purchased two plots for Rs.
5,00,000. On one he gains 15 % while on
Total cost of 45 grinders is 2215 + 2190 +
the other he losses 15%. Find how much
2760 = 7165
does he gain or lose in the transaction.
a)1.5 % b)2 %
S.P at 20% profit = 7165 + 0.2 × 7165 =
8598

pg. 67
10 Seconds

c)2.25 % d)2.50 % Questions:


1. The ratio of Pens and Pencils in a shop
Solution : is 3 : 2 respectively. The average number
of Pens and Pencils is 180. What is the
(15 ×15)
Overall Loss % = = 2.25% number of Pencils in the shop?
100
A) 444 B) 344
C) 244 D) 144
RATIO AND PROPORTIONS Solution:

Formulas: Given the ratio of pens and pencils = 3 : 2


Number of Pens = 3x
1)If x : y and z : a, then it can be solved as
(x*z)/(y*a). Number of Pencils = 2x
3x+2x
Average number of pencils & Pens = 2
2)If x/y=z/a=b/c, then each of these ratios
is equal to (x+z+e) ⁄(y+a+f) = 180
x = 72
3)If x/y=z/a, then y/x=a/z (Invertenao)
Hence, the number of pencils = 2x = 72 x
4)If x/y=z/a, then x/z=y/a (Alterenao) 2 = 144.

5)If x/y=z/a, then (x+y)/y=(z+a)/a


(Componendo) 2. Total number of students in 3 classes of
a school is 333. The number of students in
6)If x/y=z/a, then (x-y)/y=(z-a)/a class 1 and 2 are in 3 : 5 ratio and 2 and 3
(Dividenao) class are 7 : 11 ratio. What is the strength
of class that has highest number of
students?
7)If x/y=z/a, then (x+y)/(x-y)=(z+a)/(z-a)
(Componendo and Dividendo) A) 125 B) 155
C) 135 D) 165
8)Four numbers x, y, z ana a are said to
be in proportion if x : y = z : a. If on the Solution:
other hand, x : y = y : z = z : a, then the
four numbers are said to be in continued Ratio of C1 : C2 = 3 : 5 and C2 : C3 = 7 :
proportion. 11
So, C1 : C2 : C3 = 21 : 35 : 55
9)Let us consider the ratios, x : y = y : z.
Here y is called the mean proportional and Let the strength of three classes are 21x,
is equal to the square root of the product 35x and 55x respectively.
of x and z i.e. y2 = x *z ⇒ y = √xz Then, 21x + 35x + 55x = 333
10)If the three ratios, x : y, y : z, z : a is x=3
known, we can find x : a by the multiplying
So, strength of the class with highest
these three ratios x/a = x/y * y/z * z/a
number of students = 55x = 55 x 3 = 165.
11)If x, y, z, and a are four terms and the
ratios x : y, y : z, z : a are known, then one
3. Two solutions have milk & water in the
can find the ratio x : y : z : a.
ratio 7 : 5 and 6 : 11. Find the proportion in
which these two solutions should be mixed
so that the resulting solution has 1 part
milk and 2 parts water?

pg. 68
10 Seconds

A) 7 : 12 B) 8 : 13 6. If a : b = 3 : 4 and b : c = 8 : 9 then a : c
is
C) 9 : 4 D) 2 : 5
a)1 : 1 b)1 : 2
Solution:
c)2 : 3 d)3 : 4
To get the solution that contains 1 part of
milk and two parts of water, Solution:
they must be mixed in the ratio as a 3
= →1
b 4
7x + 6x : 5y + 11y = 1 : 2 b 8
c
=9 →2
26x = 16y
a 2
x : y = 8 : 13 Multiply 1 and 2 we get b
=3

4. Rs. 7444 is divided among 5 ladies, 3 7. If 2A = 3B = 4C, then A : B : C is


gents and 3 girls. The ratio of share of one
a)2 : 4 : 6 b)6 : 4 : 3
lady, one gents and one girl is 7 : 4 : 3.
What is the share of gents? c)5 : 3 : 4 d)1 : 2 : 3
A) Rs. 1595.14 B) Rs. 1793.4 Solution:
C) Rs. 595.14 D) Rs. 1551.5 Given 2A = 3B = 4C
Let 2A = 3B = 4C = k
Solution: k k k
We get A = 2 , B = 3 and C = 4
Ratio of the share of lady, gents and girl is k k k
7:4:3 Now, A: B: C = 2 : 3 : 4

No of ladies, gents and girls are 5, 3, 3 Therefore, A: B: C: = 6: 4: 3


Thus effective ratio of ladies, gents and
1 3 1 5
girls is 7 x 5 : 4 x 3 : 3 x 3 = 35 : 12 : 9 8. If A : B = : , B : C = : and C : D =
2 8 3 9
12 5 10
So, share of gents = 56 x 7444 = Rs. : then A : B : C : D is
6 9
1595.14.
a) 4 : 3 : 9 : 8 b) 3 : 4 : 5 : 8
c) 8 : 4 : 2 :1 d) 8 : 6 : 10 : 9
5. If a : b = 5 : 9 and b : c = 4 : 7, find a : b
Solution:
:c
𝟏 𝟑
a)20 : 64 : 28 b)20 : 65 : 78 A:B= : =4:3
𝟐 𝟖
𝟏 𝟓
c)20 : 36 : 63 d)20 : 66 : 50 B:C= : =3:5
𝟑 𝟗
𝟓 𝟏𝟎 𝟗
Solution: C:D= : = 10 : 9 = 5 : 𝟐
𝟔 𝟗
𝟗
 A:B:C:D = 4:3:5:𝟐
a:b=5:9 → multiply by 4
 A : B : C : D = 8 : 6 : 10 : 9
b:c=4:7 → multiply by 9

a : b = 20 : 36 9. The fourth proportional to 4, 9, 12 is


b : c = 36 : 63 a) 21 b)20

Therefore, a : b : c = 20 : 36 : 63 c)28 d)27


Solution:

pg. 69
10 Seconds

Here a = 4, b = 9, c = 12 𝐱+𝟕 𝟑
=
𝟐𝐱+𝟕 𝟓
We need to find fourth proportional d We get, x =14
a c Therefore, the smallest number is 14.
Therefore, =
b d
4 12
 9
= d
13. A sum of money is to be distributed
among A, B, C, D in the ratio of 8 : 10 : 7 :
Therefore, d = 27 5. If ‘A’ gets Rs 1500 more than ‘D’. What
is the share of ‘C’?

10. The third proportional to 16 and 36 is a)2500 b)3500

a) 45 b)66 c)4500 d)5500

c)81 d)99 Solution:

Solution: A : B : C : D = 8 : 10 : 7 : 5

Here a = 16 and b = 36 A gets 1500 more than D

We need to find third proportional c And also, A gets 3 shares more than D
a b For 3 shares Rs 1500 is extra
So, =
b c
Therefore, the value of 1 share is Rs 500
16 36
 =
36 c C is getting 7 shares so, C gets 7 × 500 =
Therefore, c = 81 3500.

11. The mean proportional to 0.08 and 14. A bag contains 50 paise, 10 paise, 5
0.18 is paise coins in the ratio 2 : 5 : 10
respectively. How many 5 paise coins are
a) 0.45 b)0.66 there in Rs 20?
c)0.12 d)0.16 a)100 b)120
Solution: c)150 d)none
Here a = 0.08 and b = 0.18 Solution:
The formula for mean proportional is √ab 50paise: 10paise: 5paise = 2 : 5 : 10
= √0.08 × 0.18 = 0.12 Taking the same number of coins as given
Therefore, the mean proportional for a and in ratio, we get
b = 0.12 50 Paise × 2 = 1rupee
10 Paise × 5 = 0.5rupee
12. Two numbers are in the ratio 1 : 2. If 7 5 Paise × 10 = 0.5rupee
is added to both, the ratio becomes 3 : 5.
The lowest number is So, total amount in the ratio is 2 rupees.
a)10 b)12 For 2 rupees → 10 five paise coins
are there
c)14 d)none
For 20 rupees → ?
Solution:
Therefore, 100 five paise coins are
Let the numbers are x and 2x
present in 20 rupees.
Given that when 7 is added to both numbers,
ratio changes to 3 : 5

pg. 70
10 Seconds

15. The prices of scooter and T.V are in are in the ratio = 3: 2
the ratio 7 : 5. If the scooter costs Rs 8000 Therefore, the marks obtained by Vijay,
more than T.V set, then price of T.V set is Amit and Abhishek are in the ratio = 12 :
15 : 10
a)20000 b)40000 Therefore, the marks obtained by Vijay
c)30000 d)none and Abhishek are in the ratio = 12 : 10 = 6
:5
Solution:
As the ratio is 7 : 5,
18. Sushil got thrice as many marks in
Let us assume the prices of scooter be 7x English as in Science. His total marks in
and tv set be 5x English, Science and maths are 162. If the
ratio of his marks in English and maths is
The difference between the two prices is 3: 5, find his marks in science?
7x-5x = 2x, it is given as 8000.
a)18 b)24
2x = 8000 implies x = 4000.
c)30 d)28
So, price of scooter is 7x = 28000
Solution:
And price of tv set is 5x = 20000
Let marks in science be x
Then, Marks in English will be 3x and
16. A sum is divided among X, Y, and Z in marks in Maths will be 5x
such a way that for each rupee X gets, Y
gets 45 paisa and Z gets 30 paisa. If the Therefore, x + 3x + 5x = 162
share of Y gets 45 paisa and Z gets 30  9x = 162
paisa. If the share of Y is Rs. 27, what is  x = 18
the total amount?
Therefore, the marks of Sushil in science
a)90 b)105 = 18.
c)96 d)120
Solution: 19. Rs. 1170 is divided so that 4 times the
first share, thrice the 2nd share and twice
X : Y : Z = 1 : 0.45 : 0.3 = 100 : 45 : 30 the third share amount to the same. What
= 20 : 9 : 6 is the value of the third share?
Given Y share is 27
9 Parts → 27 a)260 b)270
35 Parts → ? c)360 d)540
Therefore, the total amount is 105. Solution:
Let the first, second and third amounts are
17. The marks obtained by Vijay and Amit A, B and C respectively.
are in the ratio 4 : 5 and those obtained by A + B + C = 1170
Amit and Abhishek in the ratio of 3 : 2. The
marks obtained by Vijay and Abhishek are 4 times the first share, thrice the 2nd
in the ratio of? share and twice the third share amount to
the same.
a)2 : 1 b)5 : 3
Let x is the amount which is same.
c)6 : 5 d)5 : 6
4A = 3B = 2C = x
Solution: The ratio of A, B and C are:
The marks obtained by Vijay and Amit are 1 1 1
A : B : C: = : ∶ =3:4:6
in the ratio = 4: 5 4 3 2
The marks obtained by Amit and Abhishek

pg. 71
10 Seconds

Therefore, the value of 3rd share is : C. 5 : 3 D. 7 : 3


6
3+4+6
× 1170 = 540 Solution:
2
Let 40% of A = 3 B
20. Seats for Mathematics, Physics and
40 2
Biology in a school are in the ratio 5 : 7 : 8. Then, 100 A = B
3
There is a proposal to increase these
seats by 40%, 50% and 75% respectively. A:B=5:3
What will be the ratio of increased seats?
A. 2 : 3 : 4 B. 6 : 7 : 8
24. Two number are in the ratio 3 : 5. If 9
C.6 : 8 : 9 D. None of these is subtracted from each, the new numbers
are in the ratio 12 : 23. The smaller
Solution: number is:
Originally, let the number of seats for A. 27 B. 33
Mathematics, Physics and Biology be 5x,
7x and 8x respectively. C. 49 D. 55
Number of increased seats are (140% of Solution:
5x), (150% of 7x) and (175% of 8x).
Let the numbers be 3x and 5x.
21x
= 7x, and 14x. Then, 3x – 9 : 5x – 9 = 12 : 23
2

=14x : 21x : 28x 23(3x - 9) = 12(5x - 9)


Therefore, the ratio of increased seats = 2 9x = 99
:3:4
x = 11.
The smaller number = (3 x 11) = 33.
21. The sum of three numbers is 98. If the
ratio of the first to second is 2 : 3 and that
of the second to the third is 5 : 8, then the 25. Salaries of Ravi and Sumit are in the
second number is ratio 2 : 3. If the salary of each is
A.20 B.30 increased by Rs. 4000, the new ratio
becomes 40 : 57. What is Sumit's salary?
C.48 D.58
A) 38000 B) 46800
Solution:
C) 36700 D) 50000
Let the three numbers be A, B, C. Then,
Solution:
A:B=2:3
Let the original salaries of Ravi and Sumit
3 3 24 be Rs. 2x and Rs. 3x respectively.
and B : C = 5 : 8 = 5 x : 8 x = 3 :
5 5 5
Then,
A : B : C = 10 : 15 : 24
15 (2x + 4000) : (3x + 4000) = 40 : 57
 B = 98 x 49
 B = 30 ⇒ 57 × (2x + 4000) = 40 × (3x + 4000)
⇒ 6x = 68,000
23. If 40% of a number is equal to two- ⇒ 3x = 34,000
third of another number, what is the ratio
of first number to the second number? Sumit's present salary = (3x + 4000) = Rs.
(34000 + 4000) = Rs. 38,000
A. 2 : 5 B. 3 : 7

pg. 72
10 Seconds

26. A mixture contains alcohol and water Solution:


in the ratio 4 : 3. If 5 liters of water is
added to the mixture, the ratio becomes 4 Suppose there are all the pigeons then
: 5. Find the quantity of alcohol in the total no of heads are 340 and total no of
given mixture. legs are 680.

A) 10 B) 12 Now, since 380 (1060 - 680) legs are


extra, it means there will be 190 (380/2)
C) 15 D) 18 rabbits. As we know a rabbit has two extra
legs than that of a pigeon.
Solution:
Therefore, number of rabbits =190 and
Let the quantity of alcohol and water be 4x number of pigeons = 340 - 190 = 150.
litres and 3x litres respectively.
4x : (3x + 5) = 4 : 5
29. Three cats are roaming in a zoo in
20x = 4(3x + 5) such a way that when cat A takes 5 steps,
8x = 20 B takes 6 steps and C takes 7 steps. But
the 6 steps of A are equal to the 7 steps of
x = 2.5 B and 8 steps of C. What is the ratio of
Quantity of alcohol = (4 x 2.5) litres = 10 their speeds?
litres. A) 140 : 144 : 147 B) 40 : 44 : 47
C) 15 : 21 : 28 D) 252 : 245 : 240
27. The salaries of A, B, and C are in the Solution:
ratio of 1 : 2 : 3. The salary of B and C
together is Rs. 6000. By what percent is Frequency of step of A : B : C = 5 : 6 : 7
the salary of C more than that of A? But in terms of size of step, 6A = 7B = 8C
A) 100 % B) 200% Therefore, Ratio of speeds of A, B and C
5 6 7
C) 300% D) 600% = 6 : 7 : 8 = 140 : 144 : 147
Solution:
Let the salaries of A, B, C be x, 2x and 3x 30. The ratio of the incomes of Pavan and
respectively. Amar is 4 : 3 and the ratio of their
Then, 2x + 3x = 6000 expenditures are 3 : 2. If each person
saves Rs.1889, then find the income of
 x = 1200. Pavan?
A's salary = Rs. 1200, B's salary = Rs. A) 6548 B) 5667
2400, and Cs salary = Rs. 3600.
C) 7556 D) 8457
2400
Excess of C's salary over A's= [ 1200 x
Solution:
100] = 200%.
Let ratio of the incomes of Pavan and
Amar be 4x and 3x
28. In a zoo, there are rabbits and And Ratio of their expenditures be 3y and
pigeons. If heads are counted, there are 2y
340 heads and if legs are counted there
are 1060 legs. How many pigeons are 4x - 3y = 1889 → 1
there?
3x - 2y = 1889 → 2
A) 120 B) 150
From 1 and 2
C) 170 D) 180
y = 1889 and x = 1889

pg. 73
10 Seconds

Pavan's income = 3x = 7556 1)The difference between simple interest


and compound interest for 2 years at the
rate of 10% per annum on a certain sum
SIMPLE AND COMPOUND INTEREST of money is Rs 12. Find sum.

Simple Interest a)1200 b)1300

Simple interest is a percentage of the c)1400 d)1500


principal added to the loan amount
Solution :
regularly, such as by month, quarter or
year. It can be expressed as a formula: For P =1000, SI = 1200 and CI = 1210
after 2 years.
I=pxr
The interest (I) is the sum of the principal Difference = 10
(p) multiplied by the interest rate (r) and is 1000  10
the amount to be added to the principal
every accrual period—for example, every ?  12
year. If you want to know how much
Therefore Principal = 1200
interest will be added over the life of a
loan, you would multiply that interest by
the time period:
2) Find the amount to be paid after 2 years
I=pxrxt on the sum of Rs 600 compounded
In that formula, t is the duration of the annually at the rate of 20%?
loan. a)122 b)633
Compound Interest c)864 d)976
Compound interest is a percentage of the Solution :
principal amount including all previously
accrued interest. In other words, for each For P =1000, CI = 1440 after 2 years.
interest-accruing period, the amount of
1000  1440
interest added to the principal is calculated
based on the principal plus the interest 600  ?
added in the previous period. As a
borrower, compound interest can work Therefore Amount = Rs 864
against you as interest accrues faster than
you can pay down the principal.
3) Find the amount to be paid after 1 year
To calculate the amount of compound
on the sum of Rs 800 compounded half
interest you would accrue every year, you
yearly at the rate of 20%?
can use the following formula:
a)722 b)633
I = p x (1 + r)t - p
In that formula, p is the principal amount, r c)968 d)976
is the interest rate and t is the number of Solution :
accrual or compounding periods in a year.
If the number of compounding periods per Here RoI = 10%
year is more than one, you need to adjust
Amount after first quarter = 880
the formula:
Amount after second quarter = 968
I = p x (1 + r/t)t x y - p
In this version of the formula, y is the
number of years. 4)The difference between simple interest
and compound interest for 2 years at the

pg. 74
10 Seconds

rate of 4% per annum on a certain sum of 7)What will be the compound interest on a
money is Rs 1. Find sum. sum of Rs 25000 after 3 years at the rate
of 12% per annum?
a)625 b)300
a)9000 b)10123
c)400 d)500
c)11123 d)12123
Solution :
Solution :
For P =1000, SI = 1080 and CI = 1081.6
after 2 years. For P =1000, CI = 1404.92 after 3 years.
Difference = 1.6 1000  404.92
1000  1.6 25000  ?
?  1 Therefore Amount = 10123
Therefore Principal = 625
8)How much time will it take for an amount
of Rs 450 to yield to Rs 81 as interest at
5)The difference between simple interest 4.5% per annum of simple interest?
and compound interest for 2 years at the
rate of 10% per annum on a certain sum a)3 years b)2 years
of money is Rs 40. Find sum.
c)3.5 years d)4 years
a)2500 b)3000
Solution :
c)4000 d)5000
SI for 1 year is 20.25
Solution :
For the SI to become 81 it takes 4 years
For P =1000, SI = 1200 and CI = 1210
after 2 years.
Difference = 10 9) A man took a loan from a bank at 12%
p.a simple interest. After 3 years he had to
1000  10 pay Rs 5400 as interest. The principal
amount borrowed is?
?  40
a)14000 b)15000
Therefore Principal = 4000
c)16000 d)10000
Solution :
6) Find the amount to be paid after 2 years
on the sum of Rs 10000 compounded For P = 1000, Interest after 3 years = 360
annually at the rate of 20%?
1000  360
a)12220 b)13300
?  5400
c)14400 d)15500
Therefore Principal amount is Rs 15000
Solution :
For P =1000, CI = 1440 after 2 years.
10)The compound interest on Rs 30000 at
1000  1440 7% per annum is Rs 4347.The period in
year is
10000  ?
a)2 years b)2.5 years
Therefore Amount = 14400
c)3 years d)4 years

pg. 75
10 Seconds

Solution : Solution :
Interest after 1st year is 2100 First year SI = 10 Rs, Principal = 200
If checked logically option B, C, D is not For 2nd year, Principal = 100 and SI = 5 Rs
possible. Therefore Option A is correct.
Therefore in order to clear his dues he
needs to pay 115 Rs.
11)There is 60% increase in an amount in
6 years at S.I. What will be the compound
interest of Rs 12000 after 3 years at same 14)The effective annual rate of interest
rate? corresponding to a nominal rate of 6% per
annum payable half yearly is
a)2972 b)3972
a)10% b)6.06%
c)4972 d)5972
c)6.08% d)6.09%
Solution :
Solution :
For Principal = 1000, Amount = 1600 for 6
years. Therefore Rate of Interest is 10%. Here RoI = 3%

For Principal = 1000, RoI = 10%, CI = For 1000, CI = 1060.9


1331 Therefore option D is the answer.
1000  331
12000  ? 15) A bank offers 5% compound interest
Therefore CI is Rs 3972 calculated on half yearly basis. A
customer deposits Rs 1600 each on 1st
Jan and 1st July of a year. At the end of
year, the amount he would have gained by
12)If the simple interest on a sum of way of interest is,
money for 2 years at 5% per annum is Rs
50, what is the compound interest on a)100 b)121
same sum of money at same rate and for
same time? c)151 d)175

a)48 b)50 Solution :

c)51.25 d)49 Here RoI = 2.5%

Solution : For the amount deposited on Jan, CI = 81

For P =1000, SI = 1100 at 5% RoI for 2 For the amount deposited on Jan, CI = 40
years. Therefore CI = 121
For 1000, SI =100; If SI is 50, P = 500
Therefore CI at 5% after 2 years is 51.25 16) A sum of money lent at compound
interest for 2 years at 20% per annum
would fetch Rs 482 more if the interest
13)A person takes a loan of Rs.200 at 5% was payable half yearly than if it was
S.I. He returns Rs.100 at end of 1st year. payable annually. The sum is
In order to clear his dues at the end of 2
years, he would pay? a)Rs 10000 b)Rs 20000

a)100 b)115 c)Rs 40000 d)Rs 50000

c)112 d)130 Solution :

pg. 76
10 Seconds

For 1000, CI if payable yearly is 440 and a)3 years b)4 years
Half yearly is 464 after 2 years.
c)5 years d)6 years
Difference is 24
Solution :
1000  24
For P =1000, CI = 2073.6 after 4 years
?  482
Therefore Sum is 20000.
20)’A’ lent Rs 5000 to ‘B’ for 2 years and
Rs. 3000 to ‘C’ for 4 years on simple
interest at the same rate of interest and
17)The amount to be paid on a certain received Rs 2200 in total as interest. The
sum of money for 2 years is 3600 (SI) and rate of interest per annum is
for years is Rs 4500 (SI). Find the sum
and rate of interest. a)5% b)7%
a)2000 and 10% b)3000 and 20% c)8% d)10%
c)2000 and 20% d)3000 and 10% Solution :
Solution : Let us use options to solve the question
For 3 years SI = 900 and for 1 years is Option A
300.
Interest from B is 500 and C is 600
Therefore Principal is 3600 – 600 = 3000
and RoI = 10% Total 1100 (Not correct)
For 5% 1100 means 10% it is 2200

18)A man invested 1/3rd of his capital at


7%, 1/4th at 8% and remainder at 10%. If 21)A person borrowed Rs 500 at 3% per
his total annual income is 561 through annum simple interest and Rs 600 at 4.5%
interest. The capital is per annum on the agreement that whole
a)5000 b)6600 sum will be returned only when total
interest becomes Rs 126. The no of years,
c)8000 d)4000 after which the borrowed money will be
retained is
Solution :
a)2 b)3
Let Principal = 120
c)4 d)5
Rs 40 at 7% is 2.8
Solution :
Rs 30 at 8% is 2.4
For 1 year, Interest = 15 + 27 = 42
Rs 50 at 10% is 5
Therefore to become 126, it takes 3 years.
120  10.8
?  561
22) How long will it take a sum of money
Therefore Sum is 6600. invested at 5% per annum simple interest
to increase its value by 40%?

19)The least number of complete years in a)5 years b)7 years


which a sum of money put out at 20% c)6 years d)8 years
compound interest will be more than
doubled is Solution :

pg. 77
10 Seconds

For 1000, 1 year interest is 50. For the ?  930


amount to become 1400, it takes 8 years.
Therefore Principal = 30000

23)Consider the following statements : If a


sum of money is lent at simple interest 25)The simple interest on a sum of money
then in 5 years at 12 % per annum is Rs. 400
less than the simple interest accrued on
a)Money gets doubled in 5 years if the the same sum in 7 years at 10 % per
rate of interest is 15%. annum. Find the sum.
b)Money gets doubled in 5 years if the A) 3500 B) 4000
rate of interest is 20%.
C) 4500 D) 2500
c)Money becomes four times in 10 years if
it gets doubled in 5 years. Solution :

1)a and c correct Let P = 1000

2)b alone is correct SI at 12% per annum for 5 years = Rs 600

3)c alone is correct SI at 10% per annum for 10 years = Rs


700
4)b and c are correct
Difference is Rs 100
Solution :
1000  100
Let P =1000
?  400
At 15%, SI =150. So it will not be doubled
in 5 years. Therefore sum is 4000.

At 20%, SI = 200 and it will be doubled in


5 years. 26)A sum of Rs. 12,500 amounts to Rs.
After 10 years money becomes Rs 3000. 15,500 in 4 years at the rate of simple
(3 times of principal) interest. What is the rate of interest?

Therefore Option 2 is correct A) 3% B) 4%


C) 5% D) 6%

24)The difference between Compound Solution :


Interest and Simple Interest on a certain In 4 years there is a increase of 3000.
sum of money at 10% per annum for 3
years is Rs. 930. Find the principal if it is Every year there is a increase of 750.
known that the interest is compounded 750
annually. Therefore RoI = 12500 × 100 = 6%

A) 30000 B) 35000
C) 40000 D) 45000 27)A sum of Rs. 1000 was lent to two
people, one at the rate of 5 % and other at
Solution : the rate of 8 %. If the simple interest after
For P =1000, SI = 1300 and CI = 1331 one year is Rs. 68, find the sum lent at
after 3 years. each rate.

Difference = 31 A) 400, 600 B) 800, 1200

1000  31 C) 300, 500 D) 700, 900


Solution :

pg. 78
10 Seconds

Let us solve this by using options Let us solve this using options
Option A Option A
400 at 5% is 20 and 600 at 8% is 48. SI for 1 year = 84, For 6 years it is 504
(Not correct)
Total is 68 (correct)
Option B
SI for 1 year = 112, For 8 years it is 896
28)Find the compound interest on Rs. (Not correct)
5000 for 9 months at 6% per annum, if the
interest is reckoned quarterly. Option C
a. Rs. 218.98 b. Rs. 228.39 SI for 1 year = 98, For 7 years it is 686
(correct)
c. Rs. 250.69 d. Rs. 356.50
Solution :
Solution :
Amount after 1 year = Rs 7800
Here rate of interest is 6/4 = 1.5%
Amount after 2 year = Rs 8112
Amount after 3 months at 1.5% rate =
5075 Therefore CI = Rs 612
Amount after 6 months at 1.5% rate =
5151.125
TIME,SPEED AND DISTANCE
Amount after 9 months at 1.5% rate =
5228.39 Average Speed Formula

Therefore interest is 228.39 Average speed = Total Distance covered/


Total Time Taken
When the distance is constant: Average
29)A sum of money at simple interest speed = 2xy/x+y; Where, x and y are the
amounts to Rs. 815 in 3 years and to two speeds at which the same distance
Rs.854 in 4 years. What is the sum? has been covered.
A) Rs.650 B) Rs.700 When time taken is constant: Average
speed = (x + y)/2; Where, x and y are the
C) Rs.698 D) Rs.690 two speeds at which we traveled for the
Solution : same time.

Interest in one year = 854 – 815 = 39 Time and Distance Formula

Since SI is same for all the years, P = 815 Distance = Speed × Time. Using this
– 3 × 39 = Rs 698 formula, all basic problems can be
handled. However, you need to make sure
about the correct usage of units while
using the above formulas.
30)Arun took a loan of Rs.1400 with
simple interest for as many years as the Speed is inversely proportional to the time
rate of interest. If he paid Rs.686 as taken when distance travelled is constant.
interest at the end of the loan period, what So when speed increases, time decreases
was the rate of interest? and vice versa.
A) 6% B) 8% Relative Speed:
C) 7% D) 4% Relative speed is defined as the speed of
a moving object with respect to another.
Solution :
When two objects are moving in the same

pg. 79
10 Seconds

direction, relative speed is calculated as 𝑇𝑜𝑡𝑎𝑙 𝐷𝑖𝑠𝑡𝑎𝑛𝑐𝑒 738


Average speed = = = 82
their difference. When the two objects are 𝑇𝑜𝑡𝑎𝑙 𝑇𝑖𝑚𝑒 9
mph
moving in opposite directions, relative
speed is computed by adding the two
speeds.
4)Mac, Jack and Ray are on a trip by car.
Questions: ‘Mac’ drives during the first hour at an
average speed of 50kmph. ‘Jack’ drives
1. A train is running with a speed of
during next 2 hours at an average speed
108kmph. What is its speed in m/s?
of 48kmph. ‘Ray’ drives for next 3 hours at
a)20 b)25 an average speed of 52kmph. They
reached the destination after exactly 6
c)30 d)35 hours. Their mean speed was (kmph)
Solution: a)300 b)302
5
To convert 108kmph into m/s multiply c)300/6 d)302/6
18
5 Solution :
 108× = 30m/s
18
Note : 50kmph speed means “in one hour
a person covers 50km at this speed”.
Person Speed Time Distance
2. A man is walking with a speed of 15m/s. (kmph) (Hours) (km)
What is his speed in kmph? Mac 50 1 50
a)25 b)54 Jack 48 2 96
Ray 52 3 156
c)53 d)55 Total 6 302
Solution:
𝑇𝑜𝑡𝑎𝑙 𝐷𝑖𝑠𝑡𝑎𝑛𝑐𝑒 302
18 Average speed = =
To convert 15m/s into kmph multiply 5
𝑇𝑜𝑡𝑎𝑙 𝑇𝑖𝑚𝑒 6

18
 15× 5
= 54 m/s

5)A man on tour travels first 88km at


3)Sharath travels from ‘A’ to ‘B’ a distance 64kmph and next 88km at 80kmph. The
of 369 miles in 4 and half hours. He average speed of 176km journey is
returns to ‘A’ in 3 hours 30 min. His (kmph)
average speed in mph is a)60 b)71.1
a)50 b)130 c)80 d)89
c)82 d)96 Solution :
Solution: Here we can make use of the formula
2𝑋𝑌
The distance between ‘A’ to ‘B’ is 369 Average Speed = 𝑋+𝑌
miles.
where X = 64 and Y = 80
Similarly distance between ‘B’ to ‘A’ also
2 ×64 ×80
360 miles Average Speed = = 71.1 kmph
64 + 80
Total distance travelled = 369 + 369 = 738
miles
Total time taken = 4.5 + 3.5 = 9 hours

pg. 80
10 Seconds

6)A man travels 352km by train at


88kmph, 275km by ship at 55kmph,
800km by aeroplane at 400kmph and 8)A leopard can cover a certain distance
128km by car at 32kmph. What is the at the speed of 32kmph in 5 hours. To
average speed for entire distance in cover the same distance in 5/3 hours it
kmph? must travel at a speed of (kmph)

a)100 b)103.66 a)22 b)66

c)132 d)221 c)96 d)47

Solution : Solution :

Vehicle Spee Distan Time (Hours) Formula for Distance = Time × Speed
d ce = Here Distance travelled is same, therefore
(kmp (km) Distance/Sp we can take D1 = D2
h) eed
Train 88 352 4  Time1 × Speed1 = Time2 × Speed2
5
Ship 55 275 5  5 × 32 = 3 × S2
Aeropla 400 800 2  S2 = 96kmph
ne
Car 32 128 4
Total 1555 15 9)Ajay starts daily from his home to office
at 7 am and reaches the office at exact
time always. One Tuesday he walked at a
𝑇𝑜𝑡𝑎𝑙 𝐷𝑖𝑠𝑡𝑎𝑛𝑐𝑒 1555
Average speed = = = speed of 5kmph and reached the office 40
𝑇𝑜𝑡𝑎𝑙 𝑇𝑖𝑚𝑒 15
103.66 kmph minutes late than the actual time. He
walked at the speed of 8kmph on
Wednesday and reached the office 50
minutes early than the actual time. What is
7)A car travels first 1/3rd of a certain the actual time required to reach the office
distance with a speed of 10kmph, the next at his normal speed?
1/3rd distance with a speed of 20kmph and
last 1/3rd distance with a speed of a)200 minutes b)250 minutes
60kmph. The average speed for whole
journey is (kmph) c)300 minutes d)400 minutes

a)10 b)12 Solution :

c)16 d)18 Formula for Distance = Time × Speed

Solution : Here Distance travelled is same, therefore


we can take D1 = D2
Let the distance travelled each time is ‘X’
 Time1 × Speed1 = Time2 × Speed2
Journe Spee Distanc Time (Hours)
y d e (km) = Let us take ‘x’ hours for him to reach the
(kmp Distance/Spe office from home.
h) ed 40 50
 (X + 60 ) × 5 = (X - )×8
First 10 X X/10 60
Secon 20 X X/20
d  X = 200 minutes
Third 60 X X/60
Total 3X X/6 10)In the race between 3 cars, all cars
travelled equal distances with speed of
𝑇𝑜𝑡𝑎𝑙 𝐷𝑖𝑠𝑡𝑎𝑛𝑐𝑒 3𝑋 3kmph, 4kmph and 5kmph. They took a
Average speed = 𝑇𝑜𝑡𝑎𝑙 𝑇𝑖𝑚𝑒
= 𝑋/6 = 18 total time of 47 minutes. The total distance
kmph (in km) covered by them is

pg. 81
10 Seconds

a)2 b)3 13)Rohith is travelling in a train. As he was


having the window seat he started
c)4 d)5 counting the telephone posts outside. He
Solution : noticed that he can count 21 telephone
posts in one minute. If they are known to
Let the distance travelled by each car be be 50m apart, then at what speed is train
‘X’ travelling?
Given, Total time = 47 minutes a)55kmph b)57kmph
47
Time1 + Time2 + Time3 = 60 c)60kmph d)63kmph
𝐷𝑖𝑠𝑡𝑎𝑛𝑐𝑒 Solution :
Using the formula Time = , we can
𝑆𝑝𝑒𝑒𝑑
write Time given is one minute (60 seconds)
and the total distance of 21 telephone
𝑋 𝑋 𝑋 47 posts is 1000m (As there are 20 intervals
3
+ 4
+ 5
= 60
of 50m).
Solving the equation we get X = 1km 𝐷𝑖𝑠𝑡𝑎𝑛𝑐𝑒
Speed = 𝑇𝑖𝑚𝑒
Since total distance is asked, answer is 3
times of X = 3km 1000 18
Speed = × (Converting into kmph)
60 5

Speed = 63 kmph
11) Rohith walks at 14kmph instead of
10kmph and he found that he can walk
20km more in a particular time interval. 14) Excluding stoppages, the speed of a
The actual distance travelled by him is, bus is 54kmph and including stoppages it
is 45kmph. For how many minutes does
a)40km b)45km
the bus stop per hour?
c)50km d)55km
a)9 b)10
Solution :
c)12 d)20
Let us take the actual distance travelled by
Solution :
him is ‘X’ km.
The bus is travelling 9km less because of
Here the time period is same for both the
stoppages or Bus is using the time
speed i.e, Time1 = Time2
required to travel 9km for stops.
𝐷𝑖𝑠𝑡𝑎𝑛𝑐𝑒
Time = 𝐷𝑖𝑠𝑡𝑎𝑛𝑐𝑒
𝑆𝑝𝑒𝑒𝑑 Speed = 𝑇𝑖𝑚𝑒
𝑋 𝑋+20
 10
= 14
9
54 = 𝑇𝑖𝑚𝑒

 X = 50km 1
Time = 6 hours or 10 minutes

12)Which of the following trains is fastest?


15)A thief steals a car at 2:30pm and
a)25m/s b)1500m/min
drives at 60kmph. The theft is discovered
c)90kmph d)none at 3pm and owner sets off in another car
and drives at 75kmph. When will he
Solution : overtake the thief?
Convert all the options to m/s, you can see a)4pm b)5pm
that all the speeds are 25m/s.
c)6pm d)7pm

pg. 82
10 Seconds

Solution : To travel 100km more both trains must


travel for 10 hours each.
Thief will lead by 30km in half an hour.
Distance travelled by first train in 10 hours
Since owner can cover 15km extra every = 500km
hour he can catch the thief in 2 hours.
Distance travelled by second train in 10
Therefore he will overtake the thief at hours = 400km
5pm.
Therefore, total distance between two
cities = 900km
16)A passenger train is travelling from
Udupi to Bengaluru. The distance from
Udupi to Bengaluru is 600km. it is running 18)The distance between two cities ‘A’
with a speed of 75kmph stopping 3 and ‘B’ is 330km. A train starts from ‘A’ at
minutes after every 75km. How long will it 8am and travel towards ‘B’ at 60kmph and
take to reach a destination of 600km from another train starts from ‘B’ at 9am and
starting point? travel towards ‘A’ at 75kmph. When will
they meet?
a)8 hours 24min b)8 hours 21min
a)10am b)11am
c)8hours 27 min d)8 hours c)12pm d)1pm
Solution : Solution :
𝐷𝑖𝑠𝑡𝑎𝑛𝑐𝑒 After 1 hour (at 9 am) the gap between
Time required to travel 600km = 𝑆𝑝𝑒𝑒𝑑
=
600
two trains will be reduced to 270km.
= 8 hours
75
Every hour both trains cover 60 + 75 =
600 135 km (As they are coming in opposite
Total No of stops = =8
75 direction)
But the last stop is after reaching the To cover 270km they require 2 hours.
destination. Therefore it is stopping 7
times in between. Therefore they meet at 11 am.
Stoppage time = 7 × 3 = 21 minutes
Therefore total journey time is 8 hours 21 19)During the onward journey from
minutes Bombay to Pune, Deccan Queen travels
at an average speed of 80 kmph, while on
the return journey, the train is able to
17)Two trains ‘P’ and ‘Q’ started from two average a speed of 100 kmph. What is the
cities ‘A’ and ‘B’ respectively and travelled average speed of the train on its entire
towards each other at a speed of 50kmph journey?
and 40kmph respectively. By the time they A) 88(8/9)km/hr B) 89(8/9)km/hr
meet, the first train has travelled 100km
more than second. What is the distance C) 900/ 7 km/hr D) None
between two cities in km?
Solution :
a)600 b)700 2𝑋𝑌 2 ×80 ×100 800
Average speed =𝑋+𝑌 = 80+100
= 9
= 88
c)800 d)900 8
9
km/hr
Solution :
Every hour first train travels 10km more
than the second train. 20) A man travelled a distance of 61 km
in 9 hours. He travelled partly on foot

pg. 83
10 Seconds

at 4 km/hr and partly on bicycle at 9 km/hr. Solution :


What is the distance travelled on foot?
Relative speed of the thief and policeman
A) 14 km B) 16 km = (11 – 10) km/hr = 1 km/hr
Distance covered in 6 minutes = (1/60) x
C) 18 km D) 12 km 6 km = 1/10 km = 100 m
Solution : Therefore, Distance between the thief and
policeman = (200 – 100) m = 100 m.
Let the time in which he travelled on
foot =x hr
Then the time in which he travelled on 23)A truck covers a distance of 376 km at
bicycle =(9−x) hr a certain speed in 8 hours. How much
distance = speed × time time would a car take at an average speed
⇒ 4x+9(9−x)=61 which is 18 kmph more than that of the
⇒ 4x+81−9x=61 speed of the truck to cover a distance
⇒ 5x=20 which is 14 km more than that travelled by
⇒ x=4 the truck ?
Therefore distance travelled on foot = 16 A) 6 hours B) 9 hours
km
C) 8 hours D) 5 hours
Solution :
21)The speed of a car increases by 2 kms
after every one hour. If the distance Speed of the truck = Distance/time
travelling in the first one hour was 35 kms. = 376/8 = 47 kmph
what was the total distance travelled in 12 Now, speed of car = (speed of truck + 18)
hours? kmph
= (47 + 18) = 65 kmph
A) 456 kms B) 552 kms Distance travelled by car = 376 + 14 = 390
C) 482 kms D) 556 kms km
Time taken by car = Distance/Speed
Solution : = 390/65 = 6 hours
Total distance travelled in 12 hours =
(35+37+39+.....upto 12 terms)
This is an A.P with first term, a= 35, 24)In covering a distance of 30 km, Abhay
number of terms, takes 2 hours more than Sameer. If Abhay
n= 12,d=2. doubles his speed, then he would take 1
Required distance = 12/2 [2 x 35+ {12-1) hour less than Sameer. Abhay's speed is:
x 2] A) 5 kmph B) 6 kmph
= 6(70+23)
= 552 kms C) 7.5 kmph D) 6.25 kmph
Solution :
22)A thief is noticed by a policeman from a Let Abhay’s speed be ‘x’
distance of 200 m. The thief starts running 30 30
and the policeman chases him. The thief  𝑥
− 2𝑥
=3
and the policeman run at the rate of 10 km
and 11 km per hour respectively. What is  x = 5kmph
the distance between them after 6
minutes?
25)Ramesh travels 760 km to his home,
A) 100 m B) 150 m partly by train and partly by car He takes 8
C) 190 m D) 200 m hours, if he travels 160 km by train and the
rest by car. He takes 12 minutes more, if

pg. 84
10 Seconds

he travels 240 km by train and the rest by A.12 B.24


car. What are the speeds of the train and
of the car? C.30 D.40

Speed of car = 90 km/h, speed of train = Solution :


60 km/h Time taken at 3 km/hr = Distance/speed =
2. Speed of car = 100 km/h, speed 10/3
of train = 80 km/h Actual time is obtained by subtracting the
late time
3. Speed of car = 80 km/h, speed of So, Actual time = 10/3 – 1/3 = 9/3 = 3
train = 70 km/h hour
Time taken at 4 km/hr = 10/4 hr
4. Speed of car = 100 km/h, speed of Time difference = Actual time – time taken
train = 90 km/h at 4 km/hr
Solution : = 3 – 10/4
= 1/2 hour
Let speeds be x and y for train and car Hence, he will be early by 30 minutes.
respectively.
Then 8 = (160/8) + (600/y) .....(1)
And 8(1/5) = (240/x) + ((760-240)/y) .....(2) 28)A man drives at the rate of 18 km/hr,
Solving for x and y, we get 100 and 80 but stops at red light for 6 minutes at the
km/hr. end of every 7 km. The time that he will
You can also use the option straightway take to cover a distance of 90 km is :
for such Qs.
a)6 hour 12 minutes
b)6 hour 16 minutes
26)By travelling at 40 kmph, a person
reaches his destination on time. He c)6 hour 20 minutes
covered two-third the total distance in one-
third of the total time. What speed should d)6 hour 24 minutes
he maintain for the remaining distance to Solution:
reach his destination on time? (Accenture
09-01-2018) Total Red light at the end of 90 km = 90/7
A. 15 kmph B. 20 kmph = 12 Red light + 6 km
Time taken in 12 stops= 12 x 6 = 72
C. 25 kmph D. 30 kmph minutes
Solution : Time taken by the man to cover the 90 km
with 18 km/hr without stops = 90/18 = 5
Let the time taken to reach the destination hours
be 3x hours. Total time to cover total distance = 5 hour
+ 1 hour 12 minute
Total distance = 40 * 3x = 120x km = 6 hour 12 minutes
He covered 2/3 * 120x = 80x km in 1/3 *
3x = x hours
So, the remaining 40x km, he has to cover
in 2x hours 29)Two city A and B are 27 km away. Two
buses start from A and B in the same
The required speed = 40x / 2x = 20 kmph direction with speed 24 km/hr and 18
km/hr respectively. Both meet at point C
beyond B. Find the distance BC.
27)If an employee walks 10 km at a speed A)81km B)85km
of 3 km/hr, he will be late by 20 minutes. If
he walks at 4 km/hr, how early (in C)76km D)91km
minutes) from the fixed time he will reach
? Solution :

pg. 85
10 Seconds

Relative speed = 24 – 18 = 6 km/hr and after crossing each other, they took t1
Time required by faster bus to overtake and t2 time in reaching y and x
the slower bus = Distance/time = 27/6 hr respectively, then the ratio between the
Distance between B and C= 18*(27/6)= 81 speed of two trains = √t2 : √t1
km
5)If two trains leave x and y stations at
time t1 and t2 respectively and travel with
speed L and M respectively, then
30. The speed of a car increases by 2 kms distanced from x, where two trains meet is
after every one hour. If the distance = (t2 – t1) × {(product of speed) /
travelling in the first one hour was 35 kms. (difference in speed)}
what was the total distance travelled in 12
hours? 6)The average speed of a train without
any stoppage is x, and with the stoppage,
A) 456 kms B) 482 kms it covers the same distance at an average
C) 552 kms D) 556 kms speed of y, then Rest Time per hour =
(Difference in average speed) / (Speed
Solution: without stoppage)
C) 552 kms 7)If two trains of equal lengths and
different speeds take t1 and t2 time to
Explanation:
cross a pole, then the time taken by them
Total distance travelled in 12 hours to cross each other if the train is moving in
=(35+37+39+.....upto 12 terms) opposite direction = (2×t1×t2) / (t2+t1)

This is an A.P with first term, a=35, 8)If two trains of equal lengths and
number of terms, different speeds take t1 and t2 time to
cross a pole, then the time taken by them
n= 12,d=2. to cross each other if the train is moving in
Required distance = 12/2[2 x 35+{12-1) the same direction = (2×t1×t2) / (t2-t1)
x 2]
=6(70+23) Questions:
= 552 kms. 1)A train of length 250m runs at a speed
of 70 km/hr. What will be the time taken to
cross any stationary object standing at the
TRAINS railway station?
A. 20 sec B. 17.23 sec
Formulas
C. 12.86 sec D. 9.5 sec
1)Speed of the Train = Total distance
covered by the train / Time taken Solution :
L
2)If the length of two trains is given, say a Here we can use the formula S = T
and b, and the trains are moving in
5 250
opposite directions with speeds of x and y  70 × 18 = T
respectively, then the time taken by trains  T = 12.86 sec
to cross each other = {(a+b) / (x+y)}
3)If the length of two trains is given, say a 2)Chandigarh express of 100m runs at a
and b, and they are moving in the same speed of 60 km/hr. What will be the time
direction, with speeds x and y taken to cross a platform of 150 meters
respectively, then the time is taken to long?
cross each other = {(a+b) / (x-y)} A. 11.00 sec B. 12.50 sec
4)When the starting time of two trains is C. 15.00 sec D. 15.23 sec
the same from x and y towards each other

pg. 86
10 Seconds

Solution : Solution :
L+B L
Here we can use the formula S = T
Here we can use the formula S = T
5 100 + 150 5 L
 60 × 18 = T
 80 × 18 = 27
 T = 15 seconds  L = 600m

6)Rahul was standing in a platform waiting


3)The Chennai Express of 200 m runs at a for the train. The train was having an
speed of 62 km/hr and a person runs on engine of 20m in length and 8
the platform at a speed of 20 km/hr in the compartments of 22.5m each. If the train
direction opposite to that of train. Find the crosses the man in 10 seconds, then the
time taken by the train to cross the running speed of train in kmph is
person?
A. 8.77 sec B. 9.77 sec a)4.44kmph b)5.55kmph
C. 12.77 sec D. 13.00 sec c)6.66kmph d)7.77kmph
Solution : Solution :
Here we can use the formula S1 + S2 = Length of train = 20 + 22.5 × 8 = 200m
L1+L2
T L
Here we can use the formula S = T
5 200+0
 (62 + 20) × 18 = 18 200
T  S × 5 = 10
 T = 8.77 seconds
 S = 5.55 kmph

4)Two trains P and Q move in same


direction with a speed of 85 km/hr and 70 7)How long (in seconds) does a train
km/hr respectively. If train P is 120 m long 150m long running at speed of 90kmph
and train Q is 240 m, then find taken by take to cross a bridge 200m in length?
train P to cross the train Q? a)20 b)14
A. 24 sec B. 48 sec
c)30 d)35
C. 84.5 sec D. 86.4 sec
Solution :
Solution :
L+B
Here we can use the formula S =
Here we can use the formula S1 - S2 = T
L1+L2
5 150+200
T  90 × 18 = T
 (85 – 70) × 18 =
5 120 + 240  T = 14 seconds
T
 T = 86.4 sec

8)A Bullet train whose length is 175m


crossed a bridge of 325m length in 50
5) A train running at speed of 80kmph seconds. By how much value it should
crosses a pole in 27 seconds. What is the increase its speed (in m/s) to cross the
length of train? same bridge in 25 seconds.

a)200m b)400m a)20 m/s b)10 m/s

c)600m d)800m c)15m/s d)25 m/s

pg. 87
10 Seconds

Solution : 11)Two trains were going from Bangalore


L+B
to Delhi. First train started with a speed of
Here we can use the formula S = T 36kmph and second train started with a
speed of 72kmph. They crossed each
175+325
 S= 50
= 10 m/s other in 42 seconds. What is the length of
the shortest train if their lengths are in the
To cross the same bridge in 25 seconds, ratio 3:1 ?
L+B
S= T
a)101m b)103m
175+325
 S= 25
= 20 m/s c)105m d)110m

Therefore increase in speed required = 10 Solution :


m/s Here we can use the formula S1 - S2 =
L1+L2
T
9) A train 150m long is running with a 5 L1+L2
 (72 - 36) × 18 =
speed of 80kmph. In what time (in 42
seconds) it pass a man who is running in  L1 + L2 = 420 m
opposite direction at 10kmph? Their lengths are in the ratio 3 : 1
a)20 b)14 Therefore length of smallest train = 420 ×
1
c)6 d)35 = 105m
4
Solution :
Here we can use the formula S1 + S2 = 12)Train from Bangalore to Ahmedabad
L1+L2
T
crossed a man moving in same direction
in 9 seconds and car moving in same
5 150+0
 (80 + 10) × 18 = T
direction in 18 seconds. If Speed of train
 T = 6 seconds and man is 24m/s and 6m/s respectively,
then what is the speed of car in kmph?
a)15 b)24

10)Two trains are running in opposite c)34 d)54


directions with same speed. If length of Solution :
each train is 160m and they cross each
other in 16s, what is the speed of each For Train and Man
train in kmph?
Here we can use the formula S1 - S2 =
L1+L2
a)10 b)14
T
c)36 d)35 L1+0
 (24 - 6) = 9
Solution :  L1 = Length of Train = 162m
Here we can use the formula S1 + S2 = For Train and Car
L1+L2
T Here we can use the formula S1 - S2 =
L1+L2
S1 = S2 and L1 = L2 = 160m
T
5 160+160
 (S1 + S2) × 18 = 16  (24 – S2) =
162+0
18
 S1 = S2 = 10 m/s  S2 = 15m/s
Converting into kmph we get 36kmph Therefore speed in kmph is 54

pg. 88
10 Seconds

length in opposite direction in 16 seconds.


Speed of second train in m/s is,
13)A train 300m long passed a pole in 25
seconds. How long (in seconds) will it take a)10 b)15
to pass a platform 600m long?
c)36 d)35
a)25 b)54
Solution :
c)53 d)75
For Train and Man
Solution : L
Here we can use the formula S = T
For Train and Pole
200
L  S = 10
Here we can use the formula S = T
 S = 10 m/s
300
 S = 12 For Trains in opposite directions
 S = 12 m/s
Here we can use the formula S1 + S2 =
For Train and Platform L1 + L2
T
L+B
Here we can use the formula S = 200+200
T  10 + S2 = 16
300+600
 12 = T Therefore B = 15 m/s
Therefore T = 75 Seconds
16)A jogger running at 9 km/hr along side
a railway track is 240m ahead of the
14)A train passes a station platform in 48 engine of a 120m long train running at 45
seconds and a man standing in platform in km/hr in the same direction. In how much
30 seconds. If speed of train is 60kmph. time will the train pass the jogger?
What is the length of platform?
A. 3.6 sec B. 18 sec
a)200m b)300m
C. 36 sec D. 72 sec
c)600m d)800m
Solution :
Solution :
Here we can use the formula (S1 - S2) =
For Train and Man L1+L2
L T
Here we can use the formula S = T
Since Jogger is 240m ahead, the total
5 L distance the train has to cover L1 + L2=
 60 × = 18 30 240 + 120 = 360m
 L = 500m
5 360
 (45 - 9) × 18 =
For Train and Platform T
 T = 36 sec
L+B
Here we can use the formula S = T
5 500+B 17)Two trains running in opposite
 60 × 18 =
48 directions cross a man standing on the
platform in 27 seconds and 17 seconds
Therefore B = 300m
respectively and they cross each other in
23 seconds. The ratio of their speeds is:

15)A train 200m long passes a km stone A. 1 : 3 B. 3 : 2


in 20 seconds and another train of same
C. 3 : 4 D. None of these

pg. 89
10 Seconds

Solution : 20)The length of a train and that of a


L
platform are equal. If with a speed of 90
Using the formula S = T , we get L1 = S1 km/hr, the train crosses the platform in
× 27 and L2 = S2 × 17 one minute, then the length of the train (in
meters) is:
Substituting the values in (S1 + S2) =
L1+L2 A. 850 B. 525
T
S1 ×27 + S2 ×17
C. 550 D. 750
 (S1 + S2) = 23 Solution :
 S1 : S2 = 3:2
L+B
Here we can use the formula S = T
5 L+L
18)A train 125 m long passes a man,  90 × 18
=
60
running at 5 km/hr in the same direction in
which the train is going, in 10 seconds. Therefore L = 750m
The speed of the train is:
A. 45 kmph B. 50 kmph
21)A train 110 meters long is running with
C. 54 kmph D. 55 kmph a speed of 60 kmph. In what time (in
seconds) will it pass a man who is running
Solution :
at 6 kmph in the direction opposite to that
L1+L2 in which the train is going?
Using the formula (S1 - S2) = T
5 125 + 0
A. 4 B. 6
 (S1 - 5) × 18 = 10 C. 5 D. 9
 S1 = 50 kmph
Solution :
Here we can use the formula S1 + S2 =
19)A 300 meter long train crosses a L1+L2
platform in 39 seconds while it crosses a T

signal pole in 18 seconds. What is the 5


 (60 + 6) × 18 =
110+0
length of the platform? T
 T = 6 seconds
A. 150 m B. 200 m
C. 350 m D. 400 m 22)A train crosses a platform of 120 m in
15 sec, same train crosses another
Solution :
platform of length 180 m in 18 sec. then
For Train and Man find the length of the train?

Here we can use the formula S = T


L A. 175 m B. 180 m

300
C. 185 m D. 170 m
 S = 18
Solution :
 L = 16.66 m/s
L+B
For Train and Platform Here we can use the formula S = T
L+B Since speed is same, we can write
Here we can use the formula S = T
L+B1 L+B2
300+B  = T
 16.66 = 39
T
L+120 L+180
 15
= 18
Therefore B = 350m  L = 180m

pg. 90
10 Seconds

23)The two trains of lengths 400m, 600m 26)Two trains each 100m long moving in
respectively, running at same directions. opposite directions cross each other in 8
The faster train can cross the slower train seconds. If one is moving twice as fast the
in 180 sec, the speed of the slower train is other, then speed of faster train in kmph is
48kmph, then find the speed of the faster :
train?
a)30 b)60
A. 58 kmph B. 68 kmph
c)80 d)120
C. 78 kmph D. 55 kmph
Solution :
Solution :
Here we can use the formula S1 + S2 =
Here we can use the formula S1 - S2 = L1 + L2
L1 + L2 T
T
Let us take S1 = 2S2
5 400+600
 (S1 – 48)× = 5 100+100
18 180  (2S2 + S1) × 18 = 8
Therefore S1 = 68 kmph  S2 = 30 kmph
Therefore faster trainer = S1 = 2S2 =
60kmph
24)How many seconds will a 500 meter
long train take to cross a man walking with
a speed of 3 km/hr in the direction of the
moving train if the speed of the train is 63 27)Two trains 140m and 160m long run at
km/hr? the speed of 60kmph and 40kmph
respectively in opposite directions on
A. 25 sec B. 28 sec parallel tracks. The time (in seconds)
which they take to cross each other is,
C. 30 sec D. 35 sec
a)15 b)20
Solution :
c)10.80 d)12
Here we can use the formula S1 - S2 =
L1 + L2 Solution :
T
5 500+0 Here we can use the formula S1 + S2 =
 (63 – 3)× 18
=
T
L1 + L2
T
Therefore T = 30 Seconds 5 140+160
 (60 + 40) × 18 = T
 T = 10.8 Seconds
25)A train 800 m long is running at a
speed of 78 km/hr. If it crosses a tunnel in
1 min, then the length of the tunnel is?
28. A man sitting in a train which is
A. 130 m B. 360 m
traveling at 50 kmph observes that a
C. 500 m D. 540 m goods train, traveling in opposite direction,
takes 9 seconds to pass him. If the goods
Solution : train is 280 m long, find its speed.?
L+B
Here we can use the formula S = A) 60 B) 62
T
5 800+B C) 64 D) 65
 78 × 18 = 60
 B = 500m Solution:
B) 62

pg. 91
10 Seconds

Explanation: BOATS AND STREAMS


Relative speed =280/9 m / sec = Basic Concepts
(280/9*18/5) kmph = 112 kmph.
Upstream:
Speed of goods train = (112 - 50) kmph =
62 kmph. When the boat moves against the current
of the river (i.e. in opposite direction), then
the relative speed of the boat is the
difference in the speed of the boat and
29. A man's speed with the current is 15 stream. It is known as upstream speed.
km/hr and the speed of the current is 2.5
km/hr. The man's speed against the Remember it with UP as going up the hill
current is : means against the direction of the force
(speed) of the river.
A) 9.5 km/hr B) 10 km/hr
If the speed of boat or swimmer is x km/h
C) 10.5 km/hr D) 11 km/hr and the speed of the stream is y km/h
Solution: then,

B) 10 km/hr Speed of boat upstream = (x − y) km/h

Explanation: Downstream:

Man's rate in still water = (15 - 2.5) km/hr When the boat moves with the current of
= 12.5 km/hr. the river (i.e. in the same direction), then
the relative speed of the boat is the sum of
Therefore, man's rate against the current the speed of the boat and stream. It is
= (12.5 - 2.5) = 10 km/hr. known as downstream speed.
Remember it with DOWN as going down
the hill means towards the direction of the
30. A train 125 m long passes a man,
force (speed) of the river.
running at 5 km/hr in the same direction in
which the train is going, in 10 seconds. If speed of boat or swimmer is x km/h and
The speed of the train is? the speed of the stream is y km/h then,
A) 45 kmph B) 25 kmph Speed of boat downstream = (x + y) km/h
C) 30 kmph D) 50 kmph Important Points
Solution: When the speed of the boat is given then
it means speed in the still water, unless it
D) 50 kmph
is stated otherwise.
Explanation:
Some Basic Formulas
Speed of the train relative to man =
1)Speed of boat in still water is
(125/10) m/sec = (25/2) m/sec.
= ½ (Downstream Speed + Upstream
[(25/2) x (18/5)] km/hr = 45 km/hr.
Speed)
Let the speed of the train be 'S' km/hr.
2)Speed of stream is
Then, relative speed = (S - 5) km/hr.
= ½ (Downstream Speed – Upstream
S - 5 = 45 => S = 50 km/hr. Speed)
Questions:
1. A man’s speed with the current is
15kmph and speed of current is 2.5kmph.

pg. 92
10 Seconds

The man’s speed against current in kmph takes 4 hours. What is the speed of boat
is in still water?
a)10 b)20 a)8kmph b)6kmph
c)30 d)40 c)10kmph d)12kmph
Solution : Solution :
Given DS = 15 kmph and RS = 2.5 kmph Here instead of US and DS, time and
distance is given.
As DS = BS + RS, we get BS = 12.5kmph
We can find Speed using the formula
We need US = BS – RS = 12.5 – 1.5 = 𝐷𝑖𝑠𝑡𝑎𝑛𝑐𝑒
10kmph Speed = 𝑇𝑖𝑚𝑒

We get DS = 8kmph and US = 4kmph


1 1
2. A boat can travel with a speed of We know that BS = 2 × (DS + US) = 2 × (8
13kmph in still water. If speed of stream is + 4) = 6 kmph
4kmph. Find the time taken by boat to go
68km downstream?
a)4 hours b)2 hours 5. The speed of the boat in still water is
15kmph and rate of current is 3kmph. The
c)3 hours d)1 hour distance travelled downstream in 12
Solution : minutes is,

Given RS = 13 kmph and RS = 4kmph a)3km b)3.6km

DS = BS + RS = 17 kmph c)4km d)4.6km


𝐷𝑖𝑠𝑡𝑎𝑛𝑐𝑒 Solution :
We know that Speed = 𝑇𝑖𝑚𝑒
Given BS = 15kmph and RS = 3 kmph
Therefore time required to 68km
𝐷𝑖𝑠𝑡𝑎𝑛𝑐𝑒
downstream = 𝑆𝑝𝑒𝑒𝑑 DS = 15 + 3 = 18 kmph
𝐷𝑖𝑠𝑡𝑎𝑛𝑐𝑒
68 We know that Speed = 𝑇𝑖𝑚𝑒
Time = 17
= 4 hours
Therefore the distance travelled in 12
minutes can be found by using the formula
3. In one hour, a boat goes 11kmph along Distance = Speed × Time
the stream and 5kmph against the stream. 12
Distance = 18 × 60
The speed of boat in still water is,
a)8kmph b)9kmph Distance = 3.6 km

c)10kmph d)12kmph
Solution : 6. If a boat goes 7km upstream in 42
minutes and speed of stream is 3kmph,
Given DS = 11 Kmph and US = 5 Kmph then speed of boat in still water is
1 1
We know that BS = 2 × (DS + US) = 2 × a)13kmph b)14kmph
(11 + 5) = 8kmph c)15kmph d)16kmph
Solution :
4. A boat running downstream covers a 𝐷𝑖𝑠𝑡𝑎𝑛𝑐𝑒
We know that Speed =
distance of 16km in 2 hours while for 𝑇𝑖𝑚𝑒
covering the same distance upstream it

pg. 93
10 Seconds

7 36 36 3
Therefore US = = 10 kmph  + = (As Time =
42/60 10 − 𝑥 10+ 𝑥 2
𝐷𝑖𝑠𝑡𝑎𝑛𝑐𝑒
)
We know that US = BS – RS 𝑆𝑝𝑒𝑒𝑑

 10 = BS – 3 Using the equation (a + b) (a – b) = a2 – b2


 BS = 13 kmph
We get X = 2 mph

7. A motorboat whose speed is 15kmph in


still water goes 30km downstream and 9. A man can row at 5kmph in still water. If
comes back in a total of 4 hours 30 velocity of current is 1kmph and it takes
minutes. The speed of stream in kmph is, him 1 hour to row to a place and come
back. How far the place is?
a)8kmph b)6kmph
a)2.4km b)2.6km
c)5kmph d)12kmph
c)2.8km d)3km
Solution :
Solution :
Given BS = 15 kmph, Distance travelled in
Downstream and Upstream = 30km Given BS = 5 kmph, RS = 1 kmph, Total
time = 1 hour
Let us take RS = X
Let Distance travelled in Downstream and
We get DS = 15 + X and US = 15 – X
Upstream direction each = ‘X’ km
We can write Downstream time +
We get DS = 6 kmph and US = 4 kmph
Upstream time = 4.5 hours
30 30 9 𝐷𝑖𝑠𝑡𝑎𝑛𝑐𝑒 We can write Downstream time +
 15+𝑥
+ 15− 𝑥 = 2 (As Time = 𝑆𝑝𝑒𝑒𝑑 Upstream time = 1 hour
) 𝑥 𝑥 𝐷𝑖𝑠𝑡𝑎𝑛𝑐𝑒
 6
+4=1 (As Time = 𝑆𝑝𝑒𝑒𝑑
Using the equation (a + b) (a – b) = a – b 2 2
)
We get X = 5 kmph  X = 2.4 km

8. If A boat takes 90 minutes less to travel 10. A boat covers a certain distance
36 miles downstream than to travel same downstream in 1 hour, while it comes back
distance upstream. peed of boat in still in 1 and half hours. If speed of stream is
water is 10 mile per hour, speed of stream 3kmph. What is the speed of boat in still
is water?
a)2mph b)6mph a)15kmph b)12kmph
c)5mph d)12mph c)11kmph d)10kmph
Solution : Solution :
Given BS = 10 mph, Distance travelled in Given Downstream time – 1 hour and
Downstream and Upstream = 36 miles Upstream time = 1.5 hours
Let us take RS = X RS = 3 kmph
We get DS = 10 + X and US = 10 - X Let BS = X
We can write Upstream time - DS = X + 3 and US = X - 3
Downstream time = 1.5 hours
We can write the equation Downstream
Distance = Upstream Distance

pg. 94
10 Seconds

3 We know that Average speed of boat =


 (X+3 )×1 =(X−3) ×
2 𝑈𝑆 ×𝐷𝑆
(Distance = Speed × Time) 𝐵𝑆
 X = 15 kmph 8 ×16
 Average speed = 12
= 10.66
kmph
11. Two were moving in opposite direction
Since Average speed of cycle > Average
with the same speed in still water. One
speed of Boat, Cyclist will return to Place
boat covered the certain distance 3 hours
A first.
45 minutes moving in downstream
direction. The other boat covered the
same distance in 4 hours 45 minutes
moving in upstream direction. If the speed 13. A boat travels from point A to B, a
of the stream is 2kmph, find the speed of distance of 12km. From A it travels 4 km
boat in still water? downstream in 15 minutes and the
remaining 8km upstream to reach B. If the
a)15 kmph b)16kmph downstream speed is twice as high as the
upstream speed, what is the average
c)18kmph d)17kmph
speed of the boat for the journey from A to
Solution : B?

Given RS = 2kmph A. 6.44 kmph B. 9.6 kmph

Downstream time – 15/4 hour and C. 10.66 kmph D. 13.33 kmph


Upstream time = 19/4 hours
Solution :
Let BS = X
4 km downstream is covered in 15 min.
Then DS = X + 2 and US = X - 2
Therefore we get DS = 16 kmph and US =
We can write the equation Downstream 8 kmph
Distance = Upstream Distance 1 1
BS = 2 × (𝐷𝑆 + 𝑈𝑆) = 2
× (16 + 8) = 12
15 19
 (X+2 )× =(X−2) ×
4 4
kmph
 X = 17 kmph
We know that Average speed of boat =
𝑈𝑆 ×𝐷𝑆
𝐵𝑆
12. There is a road beside a river. Two
8 ×16
friends started from place ‘A’ moved to a  Average speed = = 10.66
12
temple situated at another place ‘B’ and kmph
then returned to ‘A’ again. One of them
moves on a cycle at a speed of 12kmph,
while other sails on a boat at a speed of
10kmph. If the river flows at speed of 14)A boat rows 16km up the stream and
4kmph, which of the two friends will return 30km downstream taking 5 hour each
to place ‘A’ first? time. The velocity of the current is :

a)Cyclist b)Sailor a)1.1 kmph b)1.2 kmph

c)Both at same time d)None c)1.4 kmph d)1.5 kmph

Solution : Solution :

Average speed of the cycle for complete Upstream 16km in 5 hours => Upstream
16
journey = 12 kmph Speed = 5
Given BS = 12 kmph and RS = 4 kmph Downstream 16km in 5 hours =>
30
We get DS = 16 kmph and US = 8 kmph Downstream Speed = 5

pg. 95
10 Seconds

1 The ratio of the speed of the boat in still


River speed = × (𝐷𝑆 − 𝑈𝑆) =
2
1 30 16 water and stream is
2
×(5 − 5
) = 1.4 kmph
a)3:1 b)4:1
c)5:1 d)6:1
15. A boat takes 4 hours for travelling
downstream from point A to point B aild Solution :
coming back to point A upstream. If the Let speed downstream = 2x kmph
velocity of the stream is 2 kmph and the
speed of the boat in still water is 4 kmph, Then Speed upstream = x kmph
what is the distance between A and B?
Speed of boat in still water : Speed of
a)4km b)6km stream is
1 1
c)7km d)10km
2
× (𝐷𝑆 + 𝑈𝑆) : 2
× (𝐷𝑆 − 𝑈𝑆)
Solution : 1 1
2
× (2𝑥 + 𝑥) : 2
× (2𝑥 − 𝑥)
Let the distance between A and B be x
km. Therefore the ratio is 3:1

Speed downstream = 6 kmph, speed


upstream = 2 kmph. 18) A boat takes 28 hours for travelling
𝑋
+
𝑋
=4 downstream from point A to point B and
6 2 coming back to point C midway between A
hence x = 6 and B. If the velocity of the stream is
6km/hr and the speed of the boat in still
Distance AB = 6 km. water is 9 km/hr, what is the distance
between A and B?
A. 115 km B. 120 km
16)A man rows 750 m in 675 seconds C. 140 km D. 165 km
against the stream and returns in 7 and
half minutes. His rowing speed in still Solution :
water is: Downstream speed = 9+6 = 15
Upstream speed = 9-6 = 3
a)5 kmph b)6 kmph Now total time is 28 hours
c)7 kmph d)8 kmph If distance between A and B is d, then
distance BC = d/2
Solution : Now distance/speed = time, so
750 d/15 + (d/2)/3= 28
Rate upstream = 675 = 10/9 m/sec Solving, d = 120 km
750
Rate downstream 450 m/sec = 5/3 m/sec.
1 19) Speed of a man in still water is 5 km/hr
Rate in still water = × (𝐷𝑆 + 𝑈𝑆) = and the river is running at 3km/hr. The
2
25
m/sec total time taken to go to a place and come
18
back is 10 hours. What is the distance
25 travelled?
Converting 18 m/sec into kmph we get
5kmph A. 10 km B. 16 km
C. 24 km D. 32 km
Solution :
17) A man takes twice as long to row a
Downstream speed= 5+3= 8
distance against the stream as to row the
Upstream speed= 5-3= 2
same distance in favour of the stream.
Let distance travelled = X

pg. 96
10 Seconds

𝑋 𝑋 back in a total of 5 hours. The approx.


Then + = 10
8 2
speed of the stream (in km/hr) is:
X = 16 km
Total distance is 16 +16 = 32 km A. 6 km/hr B. 9 km/hr
C. 12 km/hr D. 16 km/h
20) A boat running upstream takes 9 Solution:
hours 48 minutes to cover a certain Let the speed of the stream be x km/hr.
distance, while it takes 7 hours to cover Then,
the same distance running downstream. Speed downstream = (20 + x) km/hr,
What is the ratio between the speed of the Speed upstream = (20 – x) km/hr.
boat and speed of the water current
respectively? We can take Time Downstream + Time
Upstream = 5
A. 5:2 B. 7:4 40 40
20+𝑥
+ 20−𝑥 = 5
C. 6:1 D. 8:3 X = 9 km/hr appx.
Solution :
Let Speed of boat is ‘X’ and Speed of 23. A boatman goes 2km against the
water current is ‘Y’ current in 1 hour and goes 1km along the
current in 10 minutes. How long will it take
Then DS= X+Y and US = X-Y to go 5km in stationary water:
Distance covered upstream in 9 hrs 48
min = Distance covered downstream in 7 A. 40 minutes B. 1 hour
hours
49 C. 1 hour 15 min D. 1 hour 30 min
(X-Y) × 5 = (X+Y) × 7
𝑥
=
1 Solution:
𝑦 6 1 ×60
Downstream speed = = 6kmph
10
2
Upstream speed = 1 = 2kmph
21)A boat can travel 20 km downstream in
24 min. The ratio of the speed of the boat Speed in still water =
1
× (6 + 2) = 4kmph
in still water to the speed of the stream is 2
4 : 1. How much time will the boat take to 5
Therefore to go 5km in still water it takes
cover 15 km upstream? 4
= 1.25 hours or 1 hour 15 min
A. 20 mins B. 22 mins
C. 25 mins D. 30 mins
24. Find the speed of stream if a boat
Solution : covers 36 km in downstream in 6 hours
20 ×60
Downstream speed = 24 = 50km/hr which is 3 hours less in covering the same
Given the ratio of Speed of the boat and distance in upstream?
speed of the stream is 4:1 A) 1.5 kmph B) 1 kmph
=> BS = 4x and RS = x C) 0.75 kmph D) 0.5 kmph
Downstream speed = 4x+x=5x
Upstream speed = 4x-x = 3x Solution:
5x= 50; x=10
B) 1 kmph
so upstream speed 3×10=30
15 ×30
Time = 60 = 30 mins Explanation:
Speed of the boat upstream = 36/9 = 4
kmph
22. A boat whose speed is 20 km/hr in still
water goes 40 km downstream and comes

pg. 97
10 Seconds

Speed of the boat in downstream = 36/6 = Upstream speed = 24-3 = 21 km/hr


6 kmph
Hence, distance travelled upstream in 5
Speed of stream = 6-4/2 = 1 kmph hours = 21*5 = 105 km.

25. A steamer moves with a speed of 4.5 27. Equal distance is covered by a boat in
km/h in still water to a certain upstream upstream and in downstream in total 5
point and comes back to the starting point hours. Sum of speed of a boat in upstream
in a river which flows at 1.5 km/h. The and downstream is 40 km/hr. Speed of
average speed of steamer for the total boat in still water is 600% more than the
journey is speed of stream. Find the approximate
distance covered by boat in downstream
A) 12 km/h B) 10 km/h (in km).
C) 6 km/h D) 4 km/h A) 45 B) 50
Solution: C) 55 D) 60
D) 4 km/h Solution:
Explanation: B) 50
Speed of streamer = 4.5 km/hrSpeed of Explanation:
water = 1.5 km/hr
let speed of boat= X, speed of stream= Y
Downstream speed = 4.5+1.5 = 6 km/hr
Upstream speed= X-Y
Upstream speed = 4.5 -1.5 = 3 km/hr
Downstream speed= X+Y
Average Speed = (6 X 3) / 4.5 = 4km/hr
Sum of upstream & downstream= (X-Y)
+(X+Y)= 2X
26. The ratio of the speed of a boat So, 2X= 40
downstream and speed of the stream is
9:1. If the speed of the current is 3 km per X= 20 km/hr
hr, find the distance travelled by the boat
upstream in 5 hours. Speed of boat : speed of stream=
600+100 :100= 7:1
A) 100 km B) 98 km
So speed of Stream= 20/7 km/hr
C) 109 km D) 105 km
ATQ, D/( X-Y) + D/( X+Y) = 5
Solution:
D/(120/7) + D/(160/7)= 5
D) 105 km
D= 480×5/49= 48.97 km= 50 Km.
Explanation:
Let the speed of boat in still water is ‘x’
km/hr & that of stream is ‘y’ km/hr. 28. A Woman’s downstream swimming
rate is thrice of her upstream swimming
Then, ATQ rate. If she covers 12 km upstream in 2.5
hours, what distance she will cover in 5
(x+y)/y = 9/1 9y = x + y hours downstream?
x = 8y A) 72 km B) 36 km
y = 3 km/hr C) 56 km D) 42 km
So, x = 24 km/hr Solution:

pg. 98
10 Seconds

A) 72 km => Down Stream = 8


Explanation: 45 ---- 9
Rate of her upstream = 12/2.5 = 4.8 km/hr ? ---- 1
Then, ATQ => Up Stream = 5
Rate of downstream = 4.8 x 3 = 14.4 Speed od current S = ?
km/hr
S = (8 - 5)/2 = 1.5 kmph.
Hence, the distance she covers
downstream in 5 hrs = 14.4 x 5 = 72 kms.
RACE AND GAMES
29. Sravan drove from home to a Some Basic Concepts
neighbouring town at the speed of 50 km/h
1)'A gives B a start of x meters’: This
and on his returning journey, he drove at
statement implies that, while A starts the
the speed of 45 km/h and also took an
race from starting point, whereas, B starts
hour longer to reach home. What distance
10 meters ahead of A. To cover a race of
did he cover?
100 meters in this case, A will have to
A) 350 kms B) 450 kms cover 100 meters while B will have to
cover only (100 - x).
C) 900 kms D) 700 kms
2)'A beats B by x m': This statement
Solution: implies that in the same time, while A
C) 900 kms reached the winning point, whereas, B is
behind A by x m. To cover a race of 100
Explanation: meters in this case, A has covered 100
meters while B has covered only (100 - x )
Let the distance he covered each way = d
kms 3)A can give B a start of t minutes: This
statement implies that A will start t minutes
According to the question, after B starts from the starting point. Both
d/45 - d/50 = 1 A and B will reach the finishing point at the
same time.
=> d = 450 kms.
4)A gives B x meters and t minutes: This
Hence, the total distance he covered in his statement implies that A and B start from
way = d + d = 2 d = 2 x 450 = 900 kms. the starting point at the same instant, but
while A reaches the finishing point, B is
behind by x meters, and, B takes t minutes
30. A man swims downstream 72 km and compared to A to complete the race. So, B
upstream 45 km taking 9 hours each time; covers remaining x meters in extra t
what is the speed of the current? minutes. This gives the speed of B as x/t.

A) 1 kmph B) 3.2 kmph 5)Dead Heat: A dead heat situation is


when all participants reach the finishing
C) 1.5 kmph D) 2 kmph point at the same instant of time.
Solution: Questions:
C) 1.5 kmph 1)In a race of 300 metres A beats B by 15
Explanation: metres or 5 seconds, How much time A
take to complete the race?
72 --- 9
A. 105 sec B. 100 sec
? ---- 1
C. 95 sec D. 90 sec

pg. 99
10 Seconds

Solution : and
Given that ‘A’ beats ‘B’ by 28m or 7 ‘A’ can give ‘C’ 28 points in a 100 points
seconds, which means ‘B’ is running 15m A 100
game => C = 72 ….(Equation 2)
in 5 seconds.
Multiplying Reciprocal of Equation 1 and
Therefore to run 300m Bharath takes 100
Equation 2, we get
seconds and ‘A’ takes (100 - 5) = 95
seconds. B
=
100
C 90

Therefore ‘B’ can give ‘C’ 10 points in a


2)Every 20m Arjun runs, Amit runs 15m. In 100 points game.
a 100m race Arjun beats Amit by
A. 20m B. 25m
5)The state level running championship
C. 35m D. 40m was organised in Mysore. In a km race
category, Arjun beats Bharath by 28m or 7
Solution : seconds. Find ‘Arjun’s time over the
Amit can cover 75m when Arjun runs course.
100m. a)200 seconds b)243 seconds
Therefore Arjun beats Amit by 25m. c)267seconds d)300 seconds
Solution :
3)’A’ and ‘B’ take part in a 100m race. ‘A’ Given that ‘Arjun’ beats ‘Bharath’ by 28m
runs at 5kmph. ‘A’ gives ‘B’ a start of 8m or 7 seconds, which means Bharath is
and still beats him by 8 seconds. The running 28m in 7 seconds.
speed of ‘B’ is
Therefore to run 1000m Bharath takes 250
a)2kmph b)4.14kmph seconds and Arjun takes (250-7) = 243
c)5kmph d)10kmph seconds.

Solution :
The time taken by the ‘A’ to run 100m, T = 6)In a 100m race, ‘A’ covers the distance
100 in 36 seconds and ‘B’ covers in 45
5 = 72 seconds

18
seconds. In this race ‘A’ beats ‘B’ by how
many meters?
Therefore ‘B’ takes 80 seconds to cover
92m. a)20m b)24m
92 18 c)26m d)30m
Speed of B = 80
× 5
= 4.14 kmph
Solution :
Here ‘A’ is beating ‘B’ by 9 seconds.
4)In a game of 100 points, ‘A’ can give ‘B’
20 points and to ‘C’ 28 points. Then ‘B’ In 45 seconds ‘B’ runs 100m. Therefore in
can give ‘C’ 9 seconds ‘B’ runs 20m.
a)9 b)7 Therefore ‘A’ is beating ‘B’ by 20m.
c)8 d)10
Solution : 7)In a 100m race, ‘A’ can beat ‘B’ by 25m
and ‘B’ can beat ‘C’ by 4m. In the same
‘A’ can give ‘B’ 20 points in a 100 points race ‘A’ can beat ‘C’ by
A 100
game => B = 80 ….(Equation 1)
a)20m b)24m

pg. 100
10 Seconds

c)26m d)28m In 90 seconds R is running 375m, to run


1000m R takes 240 seconds.
Solution :
To run 1000m Q takes 210 seconds and P
‘A’ can beat ‘B’ by 25m in a 100m race takes 150 seconds.
A 100
can be taken as = ….(Equation 1)
B 75
and
10)A Tiger runs 7/4 times as fast as a
‘B’ can beat ‘C’ by 4m in a 100m race can
B 100 Deer. If Deer has a start of 84m, after
be taken as C = 96 ….(Equation 2) running how much distance Tiger will
catch the Deer?
Multiplying Equation 1 and Equation 2, we
get a)120m b)124m
A 100 c)100m d)196m
C
= 72
Solution :
Therefore ‘A’ beats ‘C’ by 28m in the same
race. If Tiger runs 7m, Deer runs 4m. In this
case Deer must have a Start of 3m for the
tiger to catch it after running 7m.
8)’Vishal’ can run 22.5m while ‘Sanjay’
Using the cross multiplication
runs 25m. In a km race ‘Sanjay’ beats
‘Vishal’ by 3m start  7m run
a)120m b)124m 84m start  ?
c)100m d)128m Therefore after running 196m Tiger will
catch the Deer.
Solution :
In a 25m race Sanjay beats Vishal by
2.5m. 11)In the Summer Olympics,100m race,
Contestant from Jamaica runs at 8kmph. If
Therefore in 1000m race Sanjay beats
Jamaica contestant gives USA contestant
Vishal by 100m.
a start of 4m and still beat him by 15
seconds. What is the speed of contestant
of USA?
9)P and Q run a Kilometre and P win by 1
minute. P and R run a kilometre and P a)5.5kmph b)5.76kmph
wins by 375 m. Q and R run a kilometre
c)6kmph d)8kmph
and Q wins By 30 Seconds. Find the time
taken by each runner to run a kilometre. Solution :
A. 150 sec, 210 sec, 240sec The time taken by the contestant of
100
B. 120 sec, 160 sec, 200sec Jamaica to run 100m, T = 5 = 45

18

C.100 sec, 160 sec, 90sec seconds

D.140 sec, 200 sec, 230sec Therefore contestant of USA takes 60


seconds to cover 96m.
Solution :
96 18
Speed of USA contestant = 60
× 5
=
P is beating R by 375m.
5.76 kmph
P is beating Q by 1 minute.
Q is beating R by 30 seconds.
12)In a 500m race, the ratio of speeds of
Therefore P is beating R by 90 seconds. two contestants Tortoise and Rabbit is 3:4.

pg. 101
10 Seconds

Tortoise has a start of 140m. Then ‘A’ can give ‘B’ 15 points in a 60 points
Tortoise wins by how many meter? A 60
game => B = 45 ….(Equation 1)
a)20m b)24m
and
c)10m d)19m
‘A’ can give ‘C’ 20 points in a 60 points
A 60
Solution : game => C = 40 ….(Equation 2)
To complete the race Tortoise should run Multiplying Reciprocal of Equation 1 and
360m and Rabbit should run 500. Equation 2, we get
If Tortoise runs 3m, Rabbit runs 4m. B 90
C
= 80
When Tortoise runs 360m, Rabbit runs
480m. Therefore ‘B’ can give ‘C’ 10 points in a 90
points game.
Therefore Tortoise beats Rabbit by 20m.

15)’A’ is 3 times quicker than ‘B’. If ‘A’


13)In a game of 80 points, ‘A’ can give ‘B’ gives ‘B’ a Start of 75 meters, Find the
5 points and to ‘C’ 15 points. Then how length of racecourse so that both of them
many points ‘B’ can give ‘C’ in a game of reach the winning post at the same time.
60 points
A. 100 m B. 155.5 m
a)9 b)7
C. 112.5 m D. 180 m
c)8 d)9
Solution :
Solution :
If ‘A’ runs 3m, ‘B’ runs 1m. In a 3m race ‘B’
‘A’ can give ‘B’ 5 points in a 80 points should have a start of 2m for both of them
A 80
game => = ….(Equation 1) to reach the winning post at the same
B 75
time.
and
Using the cross multiplication
‘A’ can give ‘C’ 5 points in a 80 points
A 80 2m start  3m race
game => = ….(Equation 2)
C 65
75m start  ?
Multiplying Reciprocal of Equation 1 and
Equation 2, we get Therefore winning post should be at a
distance of 112.5m.
B 60
C
= 52

Therefore ‘B’ can give ‘C’ 8 points in a 60 16)In a game of Snooker, ‘A’ can give ‘B’
points game. 16 Points in 80 and ‘A’ can give ‘C’ 15
point in 90. How many points can ‘C’ give
‘B’ in a game of 50?
14)In a game of 60 points, ‘A’ can give ‘B’
15 points and to ‘C’ 20 points. Then how A. 10 Points B. 12 Points
many points ‘B’ can give ‘C’ in a game of C. 5 Points D. 2 Points
90 points
Solution :
a)9 b)7
‘A’ can give ‘B’ 15 points in a 60 points
c)8 d)10 A 80
game => B = 64 ….(Equation 1)
Solution :
and

pg. 102
10 Seconds

‘A’ can give ‘C’ 20 points in a 60 points A.10 B.20


A 90
game => C = 75 ….(Equation 2) C.12 D.18
Multiplying Reciprocal of Equation 1 and Solution :
Equation 2, we get
‘A’ can give ‘B’ 12 points in a 40 points
B 48 A 40
C
=
50 game => B = 28 ….(Equation 1)

Therefore ‘C’ can give ‘B’ 2 points in a 50 and


points game.
‘A’ can give ‘C’ 10 points in a 50 points
A 50
game => C = 40 ….(Equation 2)
17)P can run 100m in 20 second and Q in
25 seconds. P beats Q by Multiplying Reciprocal of Equation 1 and
Equation 2, we get
A.10 m B.20 m B 70
=
C.25 m D.12 m C 80

Therefore ‘C’ can give ‘B’ 10 points in a


Solution :
80 points game.
In 25 Seconds Q runs 100m.
In 5 seconds Q runs 20m.
20)’A’ can run 1 km in 5 minutes 48
Therefore P is beating Q by 20m. seconds and ‘B’ in 6 minutes. How many
metres start can ‘A’ give ‘B’ in 1 km race
so that the race may end in a dead heat?
18)’A’ runs 1.5 Times as fast as ‘B’ can. If A. 20m B. 25m
‘A’ gives ‘B’ a start of 50 m, how far must
the winning post be in order that A and B C. 33.33m D. 45m
reach at the same time
Solution :
A.150 m B.120 m
In a km race ‘A’ will beat ‘B’ by 12
C.125 m D.180 m seconds.

Solution : In 360 seconds ‘B’ runs 1000m.

If ‘A’ runs 1.5m, ‘B’ runs 1m. In a 1.5m In 12 seconds ‘B’ runs 33.33m.
race ‘B’ should have a start of 0.5m for
Therefore ‘A’ should give ‘B’ a start of
both of them to reach the winning post at
33.33 m for the race to end in dead heat.
the same time.
Using the cross multiplication
21)A and B run a km race A wins by 1
0.5 m start  1.5m race
minute. A and C run a km race and A wins
50m start  ? by 425 metres. B and C run a km race and
B win by 25 seconds. Find the time that C
Therefore winning post should be at a takes to run a km race.
distance of 150m.
A. 200 Sec B. 210 Sec
C. 220 Sec D. 225 Sec
19)At a game of billiard, A can give B 12
points in a game of 40 and A can give C Solution :
10 Points in game of 50. How many points
A is beating C by 425m.
can C and B in a game of 80?
A is beating B by 1 minute.

pg. 103
10 Seconds

B is beating C by 25 seconds. B runs 35 m in 7 sec.


Therefore A is beating C by 85 seconds. B covers 200 m in (7/35*200) = 40 sec.
In 85 seconds C is running 425m, to run B's time over the course = 40 sec.
1000m C takes 200 seconds
A's time over the course (40 - 7) sec = 33
sec.
22)Two men, ‘A’ and ‘B’ run a 800 m race,
‘A’ having 560m start and their speeds are
in the ratio 3 : 7. Then, A wins By: 25. A can give B 100 meters start and C
200 meters start in a kilometre race. How
A. 200 m B. 220 m much start can B give C in a kilometre
race?
C. 240 m D.260 m
A) 110.12 meters B) 111.12 meters
Solution :
C) 112.12 meters D) 113.12 meters
To complete the race ‘A’ should run 240m
and ‘B’ should run 800. Solution:
If ‘A’ runs 3m, ‘B’ runs 7m. B) 111.12 meters
When ‘A’ runs 240m, ‘B’ runs 560m. Explanation:
Therefore ‘A’ beats ‘B’ by 140m. A runs 1000 meters while B runs 900
meters and C runs 800 meters.
Therefore, B runs 900 meters while C runs
23) In 100 m race, A covers the distance 800 meters.
in 36 seconds and B in 45 seconds. In this
race A beats B by: So, the number of meters that C runs
when B runs 1000 meters = (1000 x
A) 20m B) 25m 800)/900 = 8000/9 = 888.88 meters
C) 22.5m D) 9m Thus, B can give C (1000 - 888.88) =
Solution: 111.12 meters start

A) 20m
Explanation: 26. At a game of billiards, A can give B 15
points in 60 and A can give C to 20 points
Distance covered by B in 9 sec. = in 60. How many points can B give C in a
(100/45)*9m = 20m game of 90?
A beats B by 20 metres. A) 30 points B) 20 points
C) 10 points D) 12 points
24) In a 200 metres race A beats B by 35 Solution:
m or 7 seconds. A's time over the course
is: C) 10 points

A) 40 sec B) 47sec Explanation:

C) 33sec D) none of these A : B = 60 : 45.

Solution: A : C = 60 : 40.

C) 33sec B/C = (B/A*A/C) = (45/60*60/45) = 45/40 =


90/80 = 90:80
Explanation:
B can give C 10 points in a game of 90.

pg. 104
10 Seconds

27. In a 300 m race, A beats B by 22.5 m A) 127.5 m


or 6 seconds. B's time over the course is:
Explanation:
A) 86 sec B) 80 sec
When A runs 1000 m, B runs 900 m and
C) 76 sec D) None of these when B runs 800 m, C runs 700 m.
Solution: When B runs 900 m, distance that C runs
= (900 x 700)/800 = 6300/8 = 787.5 m.
B) 80 sec
In a race of 1000 m, A beats C by (1000 -
Explanation: 787.5) = 212.5 m to C.
B runs 45/2mts in 6 sec In a race of 600 m, the number of meters
B covers 300m in (6*2/45*300) sec =80 by which A beats C
sec = (600 x 212.5)/1000 = 127.5 m.

28. In a race of 1000 meters, A can beat B 30. In a game of 100 points, A can give B
by 100 meters, in a race of 800 meters, B 20 points and C 28 points. Then, B can
can beat C by 100 meters. By how many give C:
meters will A beat C in a race of 600
meters? A) 8 points B) 10 points
A) 125.5 meters B) 126.5 meters C) 14points D) 40points
C) 127.5 meters D) 128.5 meters Solution:
Solution: B) 10 points
C) 127.5 meters Explanation:
Explanation: A : B = 100 : 80.
When A runs 1000 meters, B runs 900 A : C = 100 : 72.
meters and when B runs 800 meters, C
runs 700 meters. B:C = B/A*A/C = 80/10*100/72 = 10/9 =
100/90 = 100:90
Therefore, when B runs 900 meters, the
distance that C runs = (900 x 700)/800 = B can give C 10 points.
6300/8 = 787.5 meters.
So, in a race of 1000 meters, A beats C by
(1000 - 787.5) = 212.5 meters to C.
CALENDAR
1)If Jan 1st 2020 falls on Wednesday then
So, in a race of 600 meters, the number of
Dec 25, 2020 falls on which day?
meters by Which A beats C = (600 x
212.5)/1000 = 127.5 meters. a)Wednesday b)Friday
c)Saturday d)Sunday
29. In a race of 1000 m, A can beat by 100 Solution :
m, in a race of 800m, B can beat C by
100m. By how many meters will A beat C Total odd days = 30
in a race of 600 m ? 30
Resultant odd days = 7
=2
A) 127.5 m B) 254 m
From Wednesday, 2 odd days is Friday.
C) 184 m D) 212 m
Solution:

pg. 105
10 Seconds

2)If Dec 30th 2021 falls on Thursday, then Total odd days till 15th August 1947 is
June 21st 2022 falls on which day? 5. Therefore the day is Friday.
a)Wednesday b)Tuesday
c)Saturday d)Sunday 4)What is the day of the week on 01-01-
2022?
Solution :
A. Sunday B. Monday
Total odd days = 12
12
C. Thursday D. Saturday
Resultant odd days = 7
=5
Solution :
From Thursday, 5 odd days is Tuesday.
Let us find out the number of odd days
present in 2021 years.
3)What is the day of the week on 15-08- 2021 = 2000 + 21
1947?
For 2000
A. Sunday B. Monday
Number odd days present in 2000 years
C. Thursday D. Friday is 0.
Solution : For 21 years

Let us find out the number of odd days Number of leap years present in 21 years
present in 1946 years. =5
1946 = 1900 + 46 Therefore number of odd days present in
5 leap years = 5 × 2 = 10 or 3
For 1900
Number of non leap years present in 21
Number odd days present in 1900 years years = 16
is 1.
Therefore number of odd days present in
For 46 years 16 non leap years = 16 × 1 = 16 or 2
Number of leap years present in 46 years Number of odd days present in 21
= 11 years = 3 + 2 = 5
Therefore number of odd days present in Total odd days in 1946 years = 3 + 2 = 5
11 leap years = 11 × 2 = 22 or 1
Now let us consider 2022nd year till Jan 1st.
Number of non leap years present in 46
years = 35 Month Jan
Therefore number of odd days present in Days 1
35 non leap years = 35 × 1 = 35 or 0 Odd days 1
Number of odd days present in 46
years = 1 + 0 = 1
Total odd days = 1
Total odd days in 1946 years = 1 + 1 = 2
𝟏
Resultant odd days = 𝟕 = 1
Now let us consider 1947th year till August
15. Total odd days till 1st January 2022 is 5.
Therefore the day is Saturday.

Total odd days = 17


𝟏𝟕 5)If today is Monday, after 105 days it will
Resultant odd days = =3
𝟕 be

pg. 106
10 Seconds

a)Wednesday b)Friday ‘1’ week ‘1’ day is 8 days.


c)Saturday d)Monday Option ‘a’ gives the value 7
Solution : Option ‘b’ gives the value 8
105 days means ‘0’ odd days. Option ‘c’ gives the value 14
From Monday ‘0’ odd days is Monday. Option ‘d’ gives the value 7

6)On what dates of April 2001 did 8)If day before yesterday was Thursday,
Wednesday fall when will Sunday be
a)1st, 8th, 15th, 22nd, 29th a)Tomorrow b)Today
b)2nd, 9th, 16th, 23rd, 30th c)Two days after today
c)3rd, 10th, 17th, 24th d)Day after tomorrow
d)4th, 11th, 18th, 25th Solution :
Solution : If day before yesterday was Thursday,
today is Saturday.
Let us find the day on 1st April 2001.
Therefore Sunday is tomorrow.
For 2000
Number odd days present in 2000 years
is 0. 9)Today is 1st April, The day of the week is
Wednesday. This is a leap year. The day
Now let us consider 2001st year till April 1. of the week on this day after 3 years will
Month Jan Feb Mar Apr be

Days 31 28 31 1 a)Saturday b)Friday


c)Monday d)Sunday
Odd 3 0 3 1
days Solution :
Total Number of odd days we get here is
Total odd days = 7 3.

Resultant odd days = = 0


7 From Wednesday 3 odd days is Saturday.
7

Total odd days till 1st April 2001 is 0


and the day is Sunday. 10)What will be the day on 26th January
2025, if today is Tuesday 26th January
Therefore 4th April is Wednesday. 2021?
a)Saturday b)Friday
7)How many days are there in ‘X’ weeks c)Monday d)Sunday
‘X’ days
Solution :
a)7x2 b)8x
Total Number of odd days we get here is
c)14x d)7 5.
Solution : From Tuesday 5 odd days is Sunday.
Substitute x = 1

pg. 107
10 Seconds

11)The last day of the century cannot be a)2 b)4


a)Wednesday b)Tuesday c)6 d)8
c)Monday d)Sunday Solution :
Solution : Birthdate of Rahul is 22nd August 1952.
Odd days at the end of each century years 1952 calendar repeats every 28 years.
is
So his birthday was on Thursday in the
100 years – 5 odd days – Friday years 1980 and 2008 that is 2 times.
200 years – 3 odd days – Wednesday
300 years – 1 odd day – Monday 15) It was Tuesday on Feb 8, 2005. What
was the day of the week on Feb 8, 2004?
400 years – 0 odd days – Sunday
A. Monday B. Thursday
Therefore it cannot be Tuesday.
C. Friday D. Sunday
Solution :
12)December 1, 2016 is Sunday, then 1st
December 2015 is which day? From Feb 8, 2004 to Feb 8 2005 there are
2 odd days.
a)Saturday b)Tuesday
Going 2 days backward we get Sunday.
c)Monday d)Friday
Solution :
17)If April 11, 1911 was a Tuesday, what
From December 1, 2015 to December 1, was the day on September 17, 1915?
2016 we get 2 odd days.
A. Friday B. Thursday
So from Sunday going 2 days backward
we get Friday. C. Sunday D. Tuesday
Solution :
13)How many times does the 29th day of From April 11, 1911 to April 11, 1915 = 5
month occur in 400 consecutive years? odd days.
a)4492 b)4493 From April 12, 1915 to September 17
1915 we get 19 or 5 odd days
c)4496 d)4497
Mont Ap Ma Ju Ju Au Se
Solution : h r y n l g p
29th day of a month comes 11 times in a Days 19 31 30 31 31 17
non leap year and 12 times in a leap year.
Odd 5 3 2 3 3 3
There are 97 leap years and 303 non leap days
years in 400 years.
Total 10 or 3 odd days.
Therefore answer = 97 × 12 + 303 × 11 =
4497 From Tuesday 3 odd days is Friday.

14)Rahul is celebrating his 60th birthday 18. Today is Monday. After 61 days, it will
22nd August 2012. He was born on be:
Thursday. How many times in these 60 A) Tuesday B) Monday
years his birthday came on Thursday ?

pg. 108
10 Seconds

C) Sunday D) Saturday Total number of odd days = (0 + 0 + 6 + 1)


= 7 = 0 odd days.
Solution:
Given day is Sunday
D) Saturday
Explanation:
21. What was the day of the week on, 16th
Each day of the week is repeated after 7 July, 1776?
days. So, after 63 days, it will be Monday.
A) Tuesday B) Wednesday
After 61 days, it will be Saturday.
C) Monday D) Saturday
Solution:
19. It was Sunday on Jan 1, 2006. What
was the day of the week Jan 1, 2010? A) Tuesday
A) Monday B) Friday Explanation:
C) Sunday D) Tuesday 16th July, 1776 = (1775 years + Period
from 1st Jan, 1776 to 16th July, 1776)
Solution:
Counting of odd days :
B) Friday
1600 years have 0 odd day.
Explanation:
100 years have 5 odd days.
On 31st December, 2005 it was Saturday.
75 years = (18 leap years + 57 ordinary
Number of odd days from 2006 to 2009 = years) = [(18 x 2) + (57 x 1)] = 93 (13
(1 + 1 + 2 + 1) = 5 days. weeks + 2 days) = 2 odd days
On 31st December 2009, it was Thursday. 1775 years have (0 + 5 + 2) odd days = 7
Thus, on 1st Jan, 2010 it is Friday. odd days = 0 odd day.
Jan Feb Mar Apr May Jun Jul

20. What was the day of the week on 28th 31 + 29 + 31 + 30 + 31 + 30 + 16 = 198


May, 2006? days= (28 weeks + 2 days)

A) Sunday B) Friday Total number of odd days = (0 + 2) = 2.

C) Wednesday D) Tuesday Required day was 'Tuesday'.

Solution:
A) Sunday 22. What will be the day of the week 15th
August, 2010?
Explanation:
A) Sunday B) Saturday
28 May, 2006 = (2005 years + Period from
1.1.2006 to 28.5.2006) C) Wednesday D) Monday

Odd days in 1600 years = 0 Solution:

Odd days in 400 years = 0 A) Sunday

5 years = (4 ordinary years + 1 leap year) Explanation:


= (4 x 1 + 1 x 2) = 6 odd days 15th August, 2010 = (2009 years + Period
(31[jan] + 28 [Feb]+ 31[Mar] + 30[April] + 1.1.2010 to 15.8.2010)
28[May] ) = 148 days = (21 weeks + 1 Odd days in 1600 years = 0
day) = 1 odd day.

pg. 109
10 Seconds

Odd days in 400 years = 0 Weekend means Saturday & Sunday


together. In total we have 52 weeks in a
9 years = (2 leap years + 7 ordinary years) year. So there are 52 weekends in a year.
= (2 x 2 + 7 x 1) = 11 = 4 odd days.
In normal we have 104 Weekend Days.
Jan. Feb. Mar. Apr. May. Jun. Jul. Aug.
We know that a Each normal year has 365
(31 + 28 + 31 + 30 + 31 + 30 + 31 + 15) = days or 52 weeks plus one day, and each
227 days = (32 weeks + 3 days) = 3 odd week has two weekend days, which
days. means there are approximately 104
Total number of odd days = (0 + 0 + 4 + 3) weekend days each year.
= 7 = 0 odd days. Whereas in a leap year we have 366 days
Given day is Sunday. it adds one more day to the year. And
what makes the change is the starting day
of the year.
23. What was the day of the week on 17th
June, 1998?
25. If 6th March, 2005 is Monday, what
A) Monday B) Tuesday was the day of the week on 6th March,
2004?
C) Wednesday D) Friday
A) Sunday B) Saturday
Solution:
C) Tuesday D) Wednesday
C) Wednesday
Solution:
Explanation:
A) Sunday
17th June, 1998 = (1997 years + Period
from 1.1.1998 to 17.6.1998) Explanation:
Odd days in 1600 years = 0 The year 2004 is a leap year. So, it has 2
odd days.
Odd days in 300 years = 1
But, Feb 2004 not included because we
97 years has 24 leap years + 73 ordinary
are calculating from March 2004 to March
years.
2005.
Number of odd days in 97 years ( 24 x 2 +
So it has 1 odd day only.
73) = 121 = 2 odd days.
The day on 6th March, 2005 will be 1 day
Jan. Feb. March. April. May. June.
beyond the day on 6th March, 2004.
(31 + 28 + 31 + 30 + 31 + 17) = 168 days
Given that, 6th March, 2005 is Monday.
168 days = 24 weeks = 0 odd day.
6th March, 2004 is Sunday (1 day before
Total number of odd days = (0 + 1 + 2 + 0) to 6th March, 2005).
= 3.
Given day is Wednesday.
26. How many days are there in x weeks x
days?

24. How many weekends in a year? A) 7x * x B) 8x

A) 52 B) 53 C) 14x D) 7

C) 103 D) 104 Solution:

Solution: B) 8x

pg. 110
10 Seconds

Explanation: In january, we have 31 days


x weeks x days = (7x + x) days = 8x days. February - 28 days (Non leap year)
March - 31 days
27. On what dates of April, 2001 did April - 30 days
Wednesday fall?
=> Remaining days => 31 - 20 = 11 in Jan
A) 2nd,9th,16th,23rd + 28 in Feb + 31 in Mar = 11 + 28 + 31 =
70 days.
B) 4th,11th,18th,25th
More 20 days to complete 90 days => up
C) 12th,18th,27th,6th to 20th April
D) 1st,8th,15th,22nd Therefore, after 90 days from today i.e,
Solution: 20th Jan 2017 is 20th Apr 2017.

B) 4th,11th,18th,25th Now, the day of the week will be

Explanation: 90/7 => Remainder '6'

We shall find the day on 1st April, 2001. As the day starts with '0' on sunday

1st April, 2001 = (2000 years + Period 6 => Saturday.


from 1.1.2001 to 1.4.2001) Required day is 20th April, Saturday.
Odd days in 1600 years = 0
Odd days in 400 years = 0 29. How many leap years does 100 years
Jan. Feb. March April have?

(31 + 28 + 31 + 1) = 91 days 0 odd days. A) 25 B) 24

Total number of odd days = (0 + 0 + 0) = 0 C) 4 D) 26

On 1st April, 2001 it was Sunday. Solution:

In April, 2001 Wednesday falls on 4th, B) 24


11th, 18th and 25th. Explanation:
Given year is divided by 4, and the
28. What is 90 days from today? quotient gives the number of leap years.

Hints : Today is 20th January 2017, Here, 100%4 = 25.


Sunday But, as 100 is not a leap year => 25 - 1=
A) 18th April, Friday 24 leap years.

B) 20th April, Saturday


C) 21th April, Sunday 30. January 1, 2007 was Monday. What
day of the week lies on Jan. 1, 2008?
D) 19th April, Saturday
A) Monday B) Tuesday
Solution:
C) Wednesday D) Sunday
B) 20th April, Saturday
Solution:
Explanation:
B) Tuesday
Given Today is 20th January 2017,
Sunday Explanation:

pg. 111
10 Seconds

The year 2007 is an ordinary year. So, it D) If the data in both the statements
has 1 odd day. together are needed.
1st day of the year 2007 was Monday From I, we have : D > A , D> C,
1st day of the year 2008 will be 1 day From II, we have : E >B > D.
beyond Monday
Combining the above two, we get : E > B>
Hence, It will be Tuesday. D> (A and C).
So, E is the tallest.
Data sufficiency Clearly, both the statements are needed to
answer the question
1. In what proportion would Raj, Karan
and Altaf distribute profit among them?
I. Raj gets two-fifth of the profit. 3. In which year was Rahul born?
II. Karim and Altaf have made 75% of the I. Rahul at present is 25 years younger to
total investment. his mother.
A) If the data in statement I alone are II. Rahul's brother, who was born in 1964,
sufficient to answer the question is 35 years younger to his mother.
B) If the data in statement II alone are A) If the data in statement I alone are
sufficient answer the question sufficient to answer the question
C) If the data either in I or II alone are B) If the data in statement II alone are
sufficient to answer the question; sufficient answer the question
D) If the data even in both the C) If the data either in I or II alone are
statements together are not sufficient sufficient to answer the question;
to answer the question
D) If the data in both the statements
Solution: together are needed.
Even both the statements together are not Solution:
sufficient to answer the question.
D) If the data in both the statements
together are needed.
2. Among five friends who is the tallest ? Explanation:
I. D is taller than A and C. From both the given statements, we find
that Rahul is (35 - 25) = 10 years older
II. B is shorter than E but taller than D. than his brother, who was born in 1964,
A) If the data in statement I alone are So, Rahul was horn in 1954
sufficient to answer the question Thus, both the given statements are
B) If the data in statement II alone are needed to answer the query.
sufficient answer the question
C) If the data either in I or II alone are 4. What is Reena's rank in the class?
sufficient to answer the question;
I. There are 26 students in the class.
D) If the data in both the statements
together are needed. II. There are 9 students who have scored
less than Reena.
Solution:
A) If the data in statement I alone are
sufficient to answer the question

pg. 112
10 Seconds

B) If the data in statement II alone are A) If the data in statement I alone are
sufficient answer the question sufficient to answer the question
C) If the data either in I or II alone are B) If the data in statement II alone are
sufficient to answer the question; sufficient answer the question
D) If the data in both the statements C) If the data either in I or II alone are
together are needed. sufficient to answer the question;
Solution: D) If the data even in both the statements
together are not sufficient to answer the
D) If the data in both the statements question
together are needed.
Solution:
Explanation:
C) If the data either in I or II alone are
From I and II, we conclude that there are sufficient to answer the question;
16 students above Reyna in rank. Thus,
Reena’s rank is 17th in the class So, both Explanation:
the statements are necessary.
Clearly, each of the given statements
shows that B is sitting opposite to C or B is
the partner of C.
5. How is D related to A?
I. B is the brother of A.
7. Rohit, 'Kajal, Tanmay and Suman are
II. B is D's son. four friends. Who is the oldest among
A) If the data in statement I alone are them?
sufficient to answer the question I. The total age of Kajol and Tanmay
B) If the data in statement II alone are together is more than that of Suman.
sufficient answer the question II. The total age of Rohit and Kajol
C) If the data either in I or II alone are together is less than that of Suntan.
sufficient to answer the question; A) If the data in statement I alone are
D) If the data in both the statements sufficient to answer the question
together are needed. B) If the data in statement II alone are
Solution: D) If the data in both the sufficient answer the question
statements together are needed. C) If the data either in I or II alone are
Explanation: sufficient to answer the question;

From both the statements together, we D) If the data even in both the
find that D is the father of B and B is the statements together are not sufficient
brother of A. So,D is the father of A. to answer the question

Thus, both the given statements are Solution:


needed. D) If the data even in both the statements
together are not sufficient to answer the
question
6. Who is C's partner in a game of cards
involving four players A. B, C and D? Explanation:

I. D is sitting opposite to A. From given statements, we find that K + T


> S and R - K <S.
II. B is sitting right of A and left of D.
Thus. who is oldest is not known.

pg. 113
10 Seconds

Solution:
8. What is Gagan's age? A) If the data in statement I alone are
sufficient to answer the question
I. Gagan, Vimal and Kunal are all of the
same age. Explanation:
II. Total age of Vimal, Kunal and Anil is 32 From statement I, we can conclude that
and Anil is as old as Vimal and Kunal Dinesh, Arun and Sachin are of the same
together. height So, Arun is not taller than Sachin.
Thus, only statement I is sufficient to
A) If the data in statement I alone are answer the question.
sufficient to answer the question
B) If the data in statement II alone are
sufficient answer the question 10. What will be the total weight of 10
poles each of the same weight?
C) If the data either in I or II alone are
sufficient to answer the question; I. One-fourth of the weight of a pole is 5
kilograms.
D) If the data in both the statements
together are needed. II. The total weight of three poles is 20
kilograms more than the total weight of
Solution: two poles.
D) If the data in both the statements A) If the data in statement I alone are
together are needed. sufficient to answer the question
Explanation: B) If the data in statement II alone are
As given in statements I and II, we have G sufficient answer the question
= V = K, V + K + A = 32 and A = V + K. C) If the data either in I or II alone are
Putting V + K = A in second. we get 2A = sufficient to answer the question;
32 or A = 16. D) If the data even in both the statements
Thus, V + K = 16 and V = K. So, V = K = together are not sufficient to answer the
8. Thus, R = 8 question

Hence, both the statements are needed. Solution:


C) If the data either in I or II alone are
sufficient to answer the question;
9. Is Arun taller than Sachin?
Explanation:
I. Dinesh is of the same height as Arun
and Sachin. From I, we find that the weight of one pole
is (5 x 4) = 20 kg and so the weight of 10
II. Sachin is not shorter than Dinesh. poles is 200 kg.
A) If the data in statement I alone are Thus, I alone is sufficient.
sufficient to answer the question
From II. we have : (weight of 3 poles —
B) If the data in statement II alone are weight of 2 poles) -= 20 kg or weight of
sufficient answer the question one pole - 20 kg.
C) If the data either in I or II alone are So, weight of 10 poles 200 kg.
sufficient to answer the question;
Thus. II alone is also sufficient.
D) If the data even in both the statements
together are not sufficient to answer the
question

pg. 114
10 Seconds

11. What day is the fourteenth of a given From statement I, we conclude that
month Manohar's birthday is on January 14.
which is Thursday. this year.
I. The last day of the month is a
Wednesday. So, only I is needed.
II. The third Saturday of the month was
seventeenth.
13. How is D related to A?
A) If the data in statement I alone are
sufficient to answer the question I. B is the brother of A.

B) If the data in statement II alone are II. B is D's son.


sufficient answer the question A) If the data in statement I alone are
C) If the data either in I or II alone are sufficient to answer the question
sufficient to answer the question; B) If the data in statement II alone are
D) If the data even in both the statements sufficient answer the question
together are not sufficient to answer the C) If the data either in I or II alone are
question sufficient to answer the question;
Solution: D) If the data in both the statements
B) If the data in statement II alone are together are needed.
sufficient answer the question Solution:
Explanation: D) If the data in both the statements
Statement II reveals that 17th Was a together are needed.
Saturday and therefore, 14th was Explanation:
Wednesday So, only statement II is
needed. From both the statements together, we
find that D is the father of B and B is the
brother of A. So,
12. When is Manohar's birthday this year? D is the father of A.
I. It is between January 13 and 15, Thus, both the given statements are
January 13 being Wednesday. needed.
II. It is not on Friday.
A) If the data in statement I alone are 14. Who is C's partner in a game of cards
sufficient to answer the question involving four players A. B, C and D?
B) If the data in statement II alone are I. D is sitting opposite to A.
sufficient answer the question
II. B is sitting right of A and left of D.
C) If the data either in I or II alone are
sufficient to answer the question; A) If the data in statement I alone are
sufficient to answer the question
D) If the data even in both the statements
together are not sufficient to answer the B) If the data in statement II alone are
question sufficient answer the question
Solution: C) If the data either in I or II alone are
sufficient to answer the question;
A) If the data in statement I alone are
sufficient to answer the question D) If the data even in both the statements
together are not sufficient to answer the
Explanation: question

pg. 115
10 Seconds

Solution: I. Prabhakar is heavier than Manoj and


Kamal but lighter than Akash.
C) If the data either in I or II alone are
sufficient to answer the question; II. Manoj is lighter than Prabhakar and
Mask but heavier than Kamal.
Explanation:
A) If the data in statement I alone are
Clearly, each of the given statements sufficient to answer the question
shows that B is sitting opposite to C or B is
the partner of C. B) If the data in statement II alone are
sufficient answer the question
C) If the data either in I or II alone are
15. Vipin's and Javed's salaries are in the sufficient to answer the question;
proportion of 4 : 3 respectively. What is
Vipin's salary ? D) If the data in both the statements
together are needed.
I. Javed's salary is 75% that of Vipin's
salary. Solution:
II. Javed's salary is Rs 4500. D) If the data in both the statements
together are needed.
A) If the data in statement I alone are
sufficient to answer the question Explanation:
B) If the data in statement II alone are From I, we have, P > M, P > K, A > P.
sufficient answer the question
From II we have, M > K, P > M, A > P.
C) If the data either in I or II alone are
sufficient to answer the question; Combining the above two, we have : A > P
> M > K.
D) If the data even in both the statements
together are not sufficient to answer the Thus, Akash is the heaviest. So, both the
question statements are required.

Solution:
B) If the data in statement II alone are 17. How many sons does D have?
sufficient answer the question I. A's father has three children.
Explanation: II. B is A's brother and son of D.
Statement I is merely an interpretation of A) If the data in statement I alone are
the information contained in the question sufficient to answer the question
However, Vipin's salary can be B) If the data in statement II alone are
determined from statement II as follows sufficient answer the question
Let Vipin's and Javed's salaries be 4x and C) If the data either in I or II alone are
ax respectively. sufficient to answer the question;
Then, 3x = 4500 or x = 1500 Vipin's salary D) If the data even in both the
= 4x = Rs 6000. statements together are not sufficient
Thus, II alone is sufficient. to answer the question
Solution:

16. Manoj, Prabhakar, Mash and Kamal D) If the data even in both the statements
are four friends. Who among them is the together are not sufficient to answer the
heaviest '' question
Explanation:

pg. 116
10 Seconds

From both I and II together, we can D) If the data even in both the
conclude that A and B are the children of statements together are not sufficient
D, but the sex of A and the third child of D to answer the question
is not known. So, both the statements
together are also not sufficient to answer Solution:
the question. D) If the data even in both the statements
together are not sufficient to answer the
question
18. On which day in January, Subhas left
for Germany? Explanation:

I. Subhas has so far spent 10 years in From the two statements, we find that
Germany. maximum (243 x 3) i.e, 729 visitors saw
the exhibition, But the exact number
II. Subhas friend Anil left for Germany on cannot be determined
15th February and joined Subhas 20 days
after Subhass arrival.
A) If the data in statement I alone are 20. In a certain code '13' means 'stop
sufficient to answer the question smoking' and '59' means 'injurious habit'.
What is the meaning of '9' and '5'
B) If the data in statement II alone are respectively in that code?
sufficient answer the question
I. '157' means 'stop bad habit'.
C) If the data either in I or II alone are
sufficient to answer the question; II. '839' means 'smoking is injurious'.

D) If the data even in both the A) If the data in statement I alone are
statements together are not sufficient sufficient to answer the question
to answer the question B) If the data in statement II alone are
Solution: sufficient answer the question

D) If the data even in both the statements C) If the data either in I or II alone are
together are not sufficient to answer the sufficient to answer the question;
question D) If the data even in both the statements
Explanation: together are not sufficient to answer the
question
Clearly, even from both the given
statements, we cannot conclude the exact Solution:
date of Subhas leaving for Germany. C) If the data either in I or II alone are
sufficient to answer the question;

19. How many visitors saw the exhibition Explanation:


yesterday? '59' means Injurious habit' and '157'
I. Each entry pass holder can take up to means 'stop bad habit' (from I). Thus, the
three persons with him/her. common code number '5' stands for
common word 'habit'. So. '9' represents
II. In all, 243 passes were sold yesterday 'injurious'. Hence, I is sufficient.
A) If the data in statement I alone are Also, '59' means Injurious habit' and '839'
sufficient to answer the question means 'smoking is injurious'. Thus, the
common code number '9' stands for
B) If the data in statement II alone are common word 'injurious'. So, '5'
sufficient answer the question represents 'habit'. Thus, II is also
C) If the data either in I or II alone are sufficient.
sufficient to answer the question;

pg. 117
10 Seconds

Solution:
21. Rohit, 'Kajal, Tanmay and Suman are C) If the data either in I or II alone are
four friends. Who is the oldest among sufficient to answer the question;
them?
Explanation:
I. The total age of Kajol and Tanmay
together is more than that of Suman. Comparing the information in the question
with statement 1. we find that '2' and '9'
II. The total age of Rohit and Kajol are the codes for 'tie' and 'dip'. So, '7'
together is less than that of Suntan. represents 'button'.
A) If the data in statement I alone are Thus, I alone is sufficient.
sufficient to answer the question
Again, comparing the information in the
B) If the data in statement II alone are question with II, WO find that the common
sufficient answer the question code number '7' stands for the common
word 'button'.
C) If the data either in I or II alone are
sufficient to answer the question; Thus, II alone also is sufficient.
D) If the data even in both the
statements together are not sufficient
to answer the question 23. How many pages of the book X did
Robert read on Sunday?
Solution:
I. The book has 300 pages out of which
D) If the data even in both the statements two-third were read by him before Sunday.
together are not sufficient to answer the
question II. Robert read the last 40 pages of the
book on the morning of Monday.
Explanation:
A) If the data in statement I alone are
From given statements, we find that K + T sufficient to answer the question
> S and R - K <S.
B) If the data in statement II alone are
Thus. who is oldest is not known sufficient answer the question
C) If the data either in I or II alone are
sufficient to answer the question;
22. In a certain code language, '297'
means 'tie clip button'. Which number D) If the data in both the statements
means 'button'? together are needed.
I. In that language '926' means 'clip your Solution:
tie'.
D) If the data in both the statements
II. In that language '175' means 'hole and together are needed.
button'.
Explanation:
A) If the data in statement I alone are
sufficient to answer the question From I and II we find that 200 pages
before Sunday
B) If the data in statement II alone are
sufficient answer the question the last 40 pages on Monday. This means
that he read (300 - (200 + 40)) = 60 pages
C) If the data either in I or II alone are on Sunday.
sufficient to answer the question;
D) If the data even in both the statements
together are not sufficient to answer the
question

pg. 118
10 Seconds

24. In a certain language, 'pit nac mit' D) If the data in both the statements
means 'red pant shirt'. Which word means together are needed.
'pant' in that language?
Explanation:
I. 'mit tim nac sir' means 'he wore red
pant'. From both the given statements, we find
that the number of cards sold this year
II. 'nee jic pit' means 'shirt is dirty'. (2935 x 1.2) = 3522.
A) If the data in statement I alone are So, both I and II together are required.
sufficient to answer the question
B) If the data in statement II alone are
sufficient answer the question 26. In a code, 'lee pee tin' means 'Always
keep smiling'. What is the code for?
C) If the data either in I or II alone are
sufficient to answer the question; I. 'tin lut lee' means 'Always keep left'.

D) If the data even in both the II. 'dee pee' means 'Rose smiling.
statements together are not sufficient A) If the data in statement I alone are
to answer the question sufficient to answer the question
Solution: B) If the data in statement II alone are
D) If the data even in both the statements sufficient answer the question
together are not sufficient to answer the C) If the data either in I or II alone are
question sufficient to answer the question;
Explanation: D) If the data even in both the statements
Clearly, from each of the statements, we together are not sufficient to answer the
find that the code for 'pant is either 'mit' or question
'nee'. Solution:
So, none of them is sufficient to answer C) If the data either in I or II alone are
the question. sufficient to answer the question;
Explanation:
25. How many new year's greeting cards Comparing the information in the question
were sold this year in your shop? with I. we find that 'tin' and 'lee' are the
I. Last year 2935 cards were sold. codes for 'always' and 'keep'.

II. The number of cards sold this year was So. 'pee' represents 'smiling' Thus, I alone
1.2 times that of last year is sufficient Am.

A) If the data in statement I alone are comparing the information in the question
sufficient to answer the question i with II, we find that the common code
word 'pee' stands for the common word
B) If the data in statement II alone are 'smiling'.
sufficient answer the question
Thus, II alone is also sufficient.
C) If the data either in I or II alone are
sufficient to answer the question;
D) If the data in both the statements 27. Is D brother of F ?
together are needed. I. B has two sons of which F is one.
Solution: II. D's mother is married to B.

pg. 119
10 Seconds

A) If the data in statement I alone are


sufficient to answer the question
29. What is the monthly salary of
B) If the data in statement II alone are Praveen?
sufficient answer the question
I. Praveen gets 15% more than Sumit
C) If the data either in I or II alone are while Sumit gets 10% less than Lokesh.
sufficient to answer the question;
II. Lokesh's monthly salary is Its 2500.
D) If the data in both the statements
together are needed. A) If the data in statement I alone are
sufficient to answer the question
Solution:
B) If the data in statement II alone are
D) If the data in both the statements sufficient answer the question
together are needed.
C) If the data either in I or II alone are
Explanation: From I, we conclude that F is sufficient to answer the question;
the son of B.
D) If the data in both the statements
From II, we conclude that B's wife is D's together are needed.
mother.
Solution:
This means that D and F are the sons of B
and D is the brother of F. D) If the data in both the statements
together are needed.
So, both I and II are required.
Explanation:
From both the given statements, we find
28. Time is 8.00 p.m., when can Hemant that : Praveen's salary s 115% of (90% of
get next bus for Ramnagar from Dhanpur? Rs 2500) = Rs 2587.50
I. Buses for Ramnagar leave after every So, both I and II are required.
30 minutes, till 10 p.m.
II. Fifteen minutes ago, one bus has left
for Ramnagar. 30. How many sons does X have?

A) If the data in statement I alone are I. Q and U are brothers of T.


sufficient to answer the question II. R is sister of P and U.
B) If the data in statement II alone are III. R and T are daughters of X.
sufficient answer the question
A) I and 11 only B) I, II and III
C) If the data either in I or II alone are together
sufficient to answer the question;
C) II and III together D) I, II and
D) If the data in both the statements III together are not sufficient
together are needed.
Solution:
Solution:
D) I, II and III together are not sufficient
D) If the data in both the statements
together are needed. Explanation:
Explanation: The information given in the three
statements together is also not sufficient
II reveals that the previous bus had left al to answer the given question.
7.45 p.m
As given in I, the next has would leave
after 30 minutes i.e, at 8.15 p.m.

pg. 120
10 Seconds

BLOOD RELATIONS Questions:


Family Tree 1. A is B's sister. C is B's mother. D is C's
father. E is D's mother. Then, how is A
1)A family tree is a pictorial representation related to D?
of genealogical data.
A) Grandfather B) Grandmother
2)All the female members of the family are
represented by a circle. The name of the C) Daughter D) Granddaughter
person is written inside the circle for
convenience. Solution:

3)All the male members of the family are A is the sister of B and B is the daughter
represented by a square. The name of the of C.
person is written inside the square for So, A is the daughter of C. Also, D is the
convenience. father of C.
4)The relation between two members of So, A is the granddaughter of D.
the family is shown by connecting a
double-headed arrow.
5)The spouse relation is represented by 2. In a family, there are six members A, B,
the two ends of a double-headed arrow. C, D, E and F. A and B are a married
couple, A being the male member. D is the
6)All the family members of the upper only son of C, who is the brother of A. E is
generation are represented above in the the sister of D. B is the daughter-in-law of
family tree. Ex: father, mother, uncle, aunt F, whose husband has died. How is E
etc. The logic can be extended by related to C?
representing the grandparents above the
parents in the family tree. Ex: grandfather A) Sister B) Daughter
and grandmother.
C) Cousin D) Mother
7)All the family members of the same
Solution:
generation are represented in the middle
of the family tree. Ex: brothers, Sisters, A is a male and married to B. So, A is the
cousins, wife, husband, etc. husband and B is the wife. C is the brother
of A. D is the son of C. E. who is the sister
8)All the family members of the next
of D will be the daughter of C. B is the
generation are represented below in the
daughter-in-law of F whose husband has
family tree. Ex: Daughter, son, niece,
died means F is the mother of A.Clearly. E
nephew.
is the daughter of C.

3. Pointing to a person, a man said to a


woman, "His mother is the only daughter
of your father." How was the woman
related to the person?
A) Aunt B) Mother
C) Wife D) Daughter
Solution:
Daughter of your father — Your sister. So,
the person's mother is woman's sister or
the woman is person's aunt.

pg. 121
10 Seconds

4. A girl introduced a boy as the son of' 7.Looking at a portrait of a man, Harsh
the daughter of the father of her uncle. said, "His mother is the wife of my father's
The boy is girl's son. Brothers and sisters I have none." At
whose portrait was Harsh looking?
A) Brother B) Son
A) His son B) His cousin
C) Uncle D) Son-in-law
C) His uncle D) His nephew
Solution:
Solution:
Daughter of uncle's father — Uncle's sister
— Mother; Since Harsh has no brother or sister, so
he is his father's only son. Now, wife of my
Mother's son — Brother father's son — my wife.
So, Harsh's wife is the man's mother or
5. There are six persons A. B, C, D, E and the man is Harsh's son
F. C is the sister of F. B is the brother of
E's husband. D is the father of A and
grandfather of F. There are two fathers, 8. Pointing to a lady, a man said, "The son
three brothers and a mother in the group. of her only brother is the brother of my
Who is the mother? wife." How is the lady related to the man?
A) A B) B A) Mother's sister B) Grandmother
C) C D) E C) Mother-in-law
Solution: D) Sister of father-in-law
D is father of A and grandfather of F. So, Solution:
A is father of F.
Brother of my wife — My brother.in-law;
Thus. D and A are the two fathers. C is the Son of lady's brother is the brother-in-law
sister of F So. C is the daughter of A. of the man. So, lady's brother is man's
father-in-law
Since there is only one mother, it is
evident that E is the wife of A and hence i.e., the lady is the sister of man's father-
the mother of C and F. in-law.
So, B is brother of A There are three
brothers. So. F is the brother of C.
Clearly, A is E's Husband. 9. P is the brother of Q and R. S is R's
mother. T is P's father. Which of the
following statements cannot be definitely
true?
6. A woman introduces a man as the son
of the brother of her mother. How is A) T is Q's father B) S is P's mother
the man, related to the woman? C) P is S's son D) Q is T's son
A) Nephew B) Son Solution:
C) Cousin D) Uncle to Grandson D) Q is T's son
Solution: Explanation:
Brother of mother — Uncle: Uncle's son — P, Q, R are children of same parents. So,
Cousin S who is R's mother and T, who is R's
father will be mother and father of all
three.

pg. 122
10 Seconds

However, it is not mentioned whether Q is 12. Introducing a man, a woman said, 'He
male or female So, D cannot be definitely is the only son of my mother's mother."
true. How is the woman related to the man?
A) Mother B) Aunt
10. In a family of six persons A, B, C, D, E C) Sister D) Niece
and F, there are two married couples.
Solution:
(ii) D is grandmother of A and mother of B.
D) Niece
(iii) C is wife of B and mother of F.
Explanation:
(iv) F is the grand daughter of E.
My mother's mother — My grandmother:
Who among the following is one of the My grandmother's only son — My
couples? maternal uncle. So, the woman is man's
niece.
A) CD B) DE
C) EB D) none of these
13. A is the brother of B. B is the brother of
Solution: C. D is the father of A. Based on these
D) none of these three statements, which of the following
statements cannot be definitely true?
Explanation:
A) B is the brother of A.
C is wife of B. So, one couple is BC. Now,
D in grandmother of A, B is the son of D B) B is the son of D.
and his wife C is the mother of F. SO, D is C) A is the brother of C.
also the grandmother of F But F is the
granddaughter of E. D) C is the brother of A.
So, E is the grandfather of F and the Solution:
husband of D. Thus, DF is another couple
D) C is the brother of A.
Explanation:
11. Q is the brother of R;
A is the brother of B and B is the brother
P is the sister of Q; of C. So, C may be the brother or sister of
A.
T is the brother of S;
S is the daughter of R.
14. A is father of C and D is son of B. E is
Who are the cousins of? brother of A. If C is sister of D, how is B
A) R and P B) P and T related to E?

C) Q and T D) S and T A) Daughter B) Brother

Solution: C) Husband D) Sister-in-law

D) S and T Solution:

Explanation: T is the brother of S, who is D) Sister-in-law


the daughter of R. So, T and S are the Explanation: A is father of C and C is
children of R. sister of D. So, A is father of D.
Now. Q is the brother of R. So, T and S
are the cousins of Q.

pg. 123
10 Seconds

But D is son of B. So, B is the mother of D Solution:


and wife of K Also. E is the brother of A
So, B is the sister-in-law of E. D) Sister
Explanation: Your mother's husband —
Your father; Your father's sister — Your
15. Pointing to a woman, a man said, "Her aunt. So, lady's aunt is man's aunt and
father is the only son of my father." How is therefore lady is man's sister.
the man related to the woman?
A) Father B) Brother
18. Pointing to a photograph, a man said,
C) Uncle D) Brother-in-law "I have no brother or sister but that man’s
father is my father’s son." Whose
Solution: photograph was it?
B) Brother A) His uncle's B) His nephew's
Explanation: The man said, "Her father is C) His son's D) His Cousin's
the only son of my father."
Solution:
Then the woman's father and the man's is
the same C) His son's
=> The woman and the man are brother Explanation: Since the narrator has no
and sisters. brother, his father’s son is he himself. So,
the man who is talking is the father of the
Hence, the man is the brother of the man in the photograph. Thus, the man in
woman. the photograph is his son.

16. Rita told Mani, "The girl I met 19. Pointing to a girl in the photograph,
yesterday at the beach was the youngest Amar said, "Her mother's brother is the
daughter of the brother-in-law of my only son of my mother's father." How is
friend's mother." How is the girl related to the girl's mother related to Amar?
Rita's friend?
A) Mother B) Sister
A) Cousin B) Daughter
C) Aunt D) Grandmother
C) Niece D) Friend
Solution:
Solution:
C) Aunt
A) Cousin
Explanation: Only son of Amar's mother's
Explanation: Daughter of brother-in-law — father — Amar's maternal uncle.
Niece;
So, the girl's maternal uncle is Amar's
Mother's niece — Cousin. maternal uncle. Thus, the girl's mother is
So, the girl is the cousin of Rita's friend. Amar's aunt.

17. A man said to a lady, "Your mother's 20. If 'A $ B' means 'A is brother of B', 'A
husband's sister is my aunt!! How is the @ B'means 'A is wife of B', 'A # B' means
lady related to the man? 'A is daughter of B' and 'A & B'means 'A is
father of B', then which of the following
A) Daughter B) Grand daughter expressions indicates the relationship 'K'
is father-in-law of H'?
C) Mother D) Sister
A) H @ J $ L # P & K

pg. 124
10 Seconds

B) H @ J $ P & L # K
C) H @ J $ L # K & P 23. Pointing to an old man, Kailash said,
"His son is my son's uncle." How is the old
D) H @ P $ J & L # K man related to Kailash?
Solution: A) Brother B) Uncle
C) H @ J $ L # K & P C) Father D) Grandfather
Explanation: K is the father-in-law of H Solution:
means H is the wife of the brother (say, J)
of the daughter (say, L) of K and K is a C) Father
male (brother or father of some person,
say P) i.e H @ J $ L # K $ P or H @ J $ L Explanation: Kailash's son's uncle —
# K & P. Kailash's brother. So. the old man's son is
Kailash's brother
i.e., the old man is Kailash's father.
21. When Anuj saw Manish, he recalled,
"He is the son of the father of my
daughter." Who is Manish? 24. In a family, there are six members A,
A) Brother-in-law B) Brother B, C, D, E and F. A and B are a married
couple, A being the male member. D is the
C) Cousin D) Uncle only son of C, who is the brother of A. E is
the sister of D. B is the daughter-in-law of
Solution: F, whose husband has died. How many
A) Brother-in-law male members are there in the family?

Explanation: Anuj's daughter's mother — A) 1 B) 2


Anuj's wife ; C) 3 D) 4
Anju's wife's father - Anuj's father-in-law; Solution:
Father-in-law's son — Anuj's brother-in- C) 3
law
Explanation: A is a male and married to B.
So, Manish is Anuj's brother-in-law So, A is the husband and B is the wife. C
is the brother of A. D is the son of C. E,
who is the sister of D will be the daughter
22. A man pointing to a photograph says, of C. B is the daughter-in-law of F whose
'The lady in the photograph is my husband has died means F is the mother
nephew's maternal grandmother." How is of A. A is a male. B. the wife, is female. C.
the lady in the photograph related to the the brother, is male. D. the son, is male. E,
man's sister who has no other sister? the sister, is female. F, the mother, is a
female. So, there are three males.
A) Cousin B) Sister-in-law
C) Mother D) Mother-in-law
25. I. A. B, C. D, E and F are six members
Solution:
of a family.
C) Mother
II. One couple has parents and their
Explanation: Clearly, the lady is the children in the family.
grandmother of man's sister's son i.e.. the
III. A is the son of C and E is the daughter
mother of the mother of man's sister's son
of A.
i.e., the mother of man's sister. So, the
V. D is the daughter of F who is the
lady is man's mother.
mother of E.

pg. 125
10 Seconds

Which of the following pairs is the parents Solution:


of the couple?
C) Sister
A) AB B) BC
Explanation:
C) AF D) CF
Only son of woman's grandfather -
Solution: woman's father; Man's brother's father -
Man's father
B) BC
So, man's father is woman's father i.e
Explanation: E is the daughter of A and F woman is the man's sister.
is the mother of E So, A is the father of E
and hence the husband of F. Now, D is
the daughter of F So. D and E are the
daughters of A and F Also. A is the son of 28. Pointing to a photograph, a lady tells
C. Now, only B remains Thus. B and C are Pramod, "I am the only daughter of this
the parents of A. Clearly. B and C are the lady and her son is your maternal uncle."
parents of the couple. How is the speaker related to Pramod's
father?
A) Sister-in-law B) Wife
26. A woman going with a boy is asked by
another woman about their relationship C) Either (a) or (b)
between them. The woman replied, "My D) Neither (a) nor (b)
maternal Uncle and the uncle of his
maternal uncle is the same". How is the Solution:
lady related with that boy?
B) Wife
A) Grandmother and Grandson
Explanation:
B) Mother and Son
Clearly, the speaker's brother is Pramod's
C) Aunt and Nephew maternal uncle. So, the speaker is
Pramod's mother or his father's wife
D) None of these
Solution:
29. Pointing to a man in a photograph, a
C) Aunt and Nephew woman said, "His brother's father is the
Explanation: only son of my grandfather." How is the
woman related to the man in the
Clearly, the brother of woman's mother is photograph?
the same as the brother of the father of
boy's maternal uncle. So, the woman's A) Mother B) Aunt
mother's brother is the boy's maternal C) Sister D) Daughter
uncle's father. Thus, the woman's
mother's brother's son is boy's maternal Solution:
uncle, i.e woman's mother's brother's
C) Sister
daughter is boy's mother.
Explanation:
Only son of woman's grandfather —
27. Introducing a man to her husband, a
Woman's father.
woman said "His brother's father is the
only son of my grandfather". How is the Man's brother's father — Man's father
woman related to this man?
So, the woman is man's sister.
A) Mother B) Aunt
C) Sister D) Aunt

pg. 126
10 Seconds

30. A, B, C, D, E, F and G are members of 2. Find the LCM of 1.2, 1.5, 2 and 5
a family consisting of four adults and three
children, two of whom, F and G are girls. A a) 6 b) 8
and D are brothers and A is a doctor. E is c)10 d) 12
an engineer married to one of the brothers
and has two children. if is married to D Solution:
and G is their child. Who is C?
The given numbers are 1.2, 1.5 and 5,
A) A's son B) E's daughter multiply 10 to all the numbers
C) F's father D) G's brother Then, the numbers are 12, 15 and 10
Solution: LCM of 12, 15 and 10 is 60
60
A) A's son Then, LCM = 10 = 6
Explanation:
E is married to A or D. But B is married to 3. Find the LCM of 1.2, 0.24 and 6
D. Thus, E is married to A.
a) 5 b)10
Thus, A, B, C, D, E are the four adults and
C, F, G are the three children in the family. c) 6 d)12
B and D have a child G. A and E have two
children. They are C and F Now. only F Solution:
and G are girls. The given numbers are 1.2, 0.24 and 6,
So, C is a boy. Thus, C is A's or E's son. multiply 100 to all the numbers
Then, the numbers are 120, 24 and 600
LCM of 120, 24 and 600 is 300
600
NUMBER SYSTEM Then, LCM = 100 = 6

1. Find the L.CM and H.C.F of 6/9, 8/5 and


4/7.
4. If the product 4864*9 P 2 is divisible by
𝟐
a) 24,𝟑𝟏𝟓 b) 36,48 12, the value of p:
3 a) 1 b) 2
c)48,36 d)48, 315
c) 3 d) 4
Solution:
Solution:
𝐿𝐶𝑀 𝑜𝑓 𝑛𝑢𝑚𝑒𝑟𝑎𝑡𝑜𝑟𝑠
LCM of fractions = 𝐻𝐶𝐹 𝑜𝑓 𝑑𝑒𝑛𝑜𝑚𝑖𝑛𝑎𝑡𝑜𝑟𝑠 Clearly 4864 is divisible by 4
LCM of 6, 8 and 4 is 24 So, 9 P 2 must be divisible by 3. So
(9+P+2) must be divisible by 3.
HCF of 9, 5 and 7 is 1
24 So P=1.
 LCM = 1
= 24
𝐻𝐶𝐹 𝑜𝑓 𝑛𝑢𝑚𝑒𝑟𝑎𝑡𝑜𝑟𝑠
H.C.F of Fractions = 𝐿𝐶𝑀 𝑜𝑓 𝑑𝑒𝑛𝑜𝑚𝑖𝑛𝑎𝑡𝑜𝑟𝑠 5. The difference between the place value
and the face value of 7 in the numeral
HCF of 6, 8 and 4 = 2 967452 is
LCM of 9, 5 and 7 is 315 a) 6393 b) 5831
2
 HCF = 315 c) 6993 d) 6339

pg. 127
10 Seconds

Solution: so largest 4 digit number divisible by 88 =


9999-55 = 9944.
(Place value of 7)-(face value of 7)
=7000 - 7= 6993.
9. The sum of the two numbers is 12 and
their product is 35. What is the sum of the
6. A number when divided by 296 leaves reciprocals of these numbers?
75 as remainder. When the same number a) 1/35 b) 12/35
is divided by 37,the remainder will be:
c) 35/8 d) 7/32
a) 1 b) 3
Solution:
c) 5 d) 2
Let a and b are the numbers.Then a+b is
Solution: 12 and ab is 35.
Let the Number be Y. a+b/ab = 12/35
Then Y = 296 q + 75 1/b + 1/a = 12/35
= (37 x 8)q +( 37 x 2) + 1
= 37 (8q + 2) + 1 10. The unit digit in the product (784 x 618
Thus, when the number is divided by 37, x 917 x 463) is:
the remainder is 1 a.5 b.3
c.4 d.2
7. If the number 517?324 is completely Solution:
divisible by 3,then the smallest whole
number in place of ? will be: Unit digit in the given product = Unit digit
in (4 x 8 x 7 x 3) = (672) = 2
a) 2 b) 1
c) 3 d) 7
11. How many of the following numbers
Solution: are divisible by 132?
Sum of digits = (5 + 1 + 7 + x + 3 + 2 + 4) 264,396,462,792,968,2178,5184,6336
= ( 22 + x ), which must be divisible by 3.
A) 4 B) 5
When x=2. [(22 + 2) = 24 is divisible by 3]
C) 6 D) 7
So, the answer is 2
Solution:
A)4
8. The largest 4 digit number exactly
divisible by 88 is? Explanation:
a) 9944 b) 9768 132 = 4 x 3 x 11, So if the number is
divisible by all three numbers 4,3 and
c) 8888 d) 9988 11,then the number is divisible by 132
Solution: also.

The largest 4 digit number is = 9999 264 => 4,3,11(/)

After dividing 9999 with 88, we get 55 as 396 => 4,3,11(/)


remainder 462 => 11,3
792 => 4,3,11(/)

pg. 128
10 Seconds

968 => 11,4 and the other digits are correct, then the
correct answer would be :
2178 => 11,3
A) 553681 B) 555181
5184 => 3,4
C) 555681 D) 556581
6336 => 4,3,11(/)
Solution:
Required number of numbers=4.
C) 555681
Explanation:
12. The sum of all two digit numbers
divisible by 5 is 987 = 3 * 7 * 47.
A) 945 B) 678 So, the required number must be divisible
by each one of 3, 7, 47
C) 439 D) 568
553681 => (Sum of digits = 28, not
Solution: divisible by 3)
A) 945 555181 => (Sum of digits = 25, not
Explanation: divisible by 3)

Required numbers are 10,15,20,25,...,95 555681 is divisible by each one of 3, 7, 47.

This is an A.P. in which a=10,d=5 and


l=95. 15. The difference between the place
Let the number of terms in it be n. Then value and the face value of 7 in the
t=95 numeral 967452 is

So a+(n-1)d=95. A) 6393 B) 5831

10+(n-1)*5=95,then n=18. C) 6993 D) 6339

Required sum=n/2(a+l)=18/2(10+95)=945. Solution:


C) 6993

13. What least number must be subtracted Explanation:


from 13601, so that the remainder is (Place value of 7)-(face value of 7)
divisible by 87?
=7000-7=6993.
A) 23 B) 31
C) 29 D) 37
16. On dividing 2272 as well as 875 by 3-
Solution: digit number N,we get the same
C) 29 remainder. The sum of the digits of N is:

Explanation: A) 13 B) 12

13601/29, remainder is 29 C) 11 D) 10

Required number is 29. Solution:


D) 10

14. A boy multiplied 987 by a certain Explanation:


number and obtained 559981 as his (2272-875) = 1397, is exactly divisible by
answer. If in the answer both 98 are wrong N.

pg. 129
10 Seconds

Now , 1397 = 11 x 127 Explanation:


The required 3-digit number is 127,the Let the Number be Y.
sum of digits is 10.
Then Y = 296 q + 75
= (37 x 8)q +( 37 x 2) + 1
17. Which of the following can be used to
illustrate that not all prime numbers are = 37 (8q + 2) + 1
odd? Thus, when the number is divided by 37,
A) 1 B) 2 the remainder is 1

C) 3 D) 4
Solution: 20. If the number 517?324 is completely
divisible by 3, then the smallest whole
B) 2 number in place of ? will be:
Explanation: A) 2 B) 1
The only even numbers in the list are 2 C) 3 D) 7
and 4, but 4 is not a prime. So 2 can be
used to illustrate the statement that all Solution:
primes are not odd. A) 2
Explanation:
18. How many terms are in the G.P. 3, 6, Sum of digits = (5 + 1 + 7 + x + 3 + 2 + 4)
12, 24, ......., 384 ? = ( 22 + x ), which must be divisible by 3.
A) 8 B) 9 When x=2. [(22 + 2) = 24 is divisible by 3]
C) 10 D) 11 So, the answer is 2
Solution:
A) 8 21. What will come in place of the question
Explanation: mark (?) in the following questions ?

Here a = 3 and r = 6/3 = 2. Let the number 22% of ? + 166.64 = 340


of terms be n A) 788 B) 786
Then, t = 384 => a * r^(n-1) = 384 C) 784 D) 792
=> 3 * 2^(n-1) = 384 => 2^(n-1) = 128 = Solution:
2^(7)
A) 788
=> n-1 = 7 => n = 8.
Explanation:
(kx22)/100 = 340 - 166.64 = 173.36
19. A number when divided by 296 leaves
75 as remainder. When the same number k = (173.36 x 100)/22
is divided by 37,the remainder will be:
k = 788
A) 1 B) 3
C) 5 D) 2
22. The largest 4 digit number exactly
Solution: divisible by 88 is?
A) 1 A) 9944 B) 9768

pg. 130
10 Seconds

C) 8888 D) 9988 25. Find the next number in the series


141,137,146,130,155,119,...
Solution:
A) 147 B) 168
A) 9944
C) 162 D) 182
The largest 4 digit number is = 9999
Solution:
After dividing 9999 with 88, we get 55 as
remainder B) 168
so largest 4 digit number divisible by 88 = Explanation:
9999-55 = 9944.
The pattern is -4, +9,-16, +25, -36
The next number=119+49=168
23) x = y - (50/y), where x and y are both >
0
If the value of y is doubled in the equation 26. What is the simplified result of
above, the value of x will following the steps below in order?

A) decrease B) stay the same 1.Add 5y to 2x

C) increase four fold 2.Multiply the sum by 3

D) increase to more than double 3.Substract x+y from the product

Solution: A) 5x + 14y B) 5x + 16y

D) increase to more than double C) 5x + 5y D) None

Explanation: Solution:

The best approach is to pick a number A) 5x + 14y


that satisfies the rules that x and y are Explanation:
greater than 0. So for example you could
choose 25 for y. Step 1 :2x + 5y
The value of x before doubling is then 25 - Step 2 :3(2x + 5y) = 6x + 15y
50/25 = 23
Step 3 :(6x + 15y) - (x +y) = 6x + 15y - x -
The value after doubling y will be 50 - y (watch that last minus sign!)
50/50 = 49, which is more than double.
This gives 5x + 14y.

24. What smallest number should be


27. Which one of the following cannot be
added to 4456 so that the sum is
the square of a natural number?
completely divisible by 6?
A) 32761 B) 42437
A) 4 B) 3
C) 81225 D) 20164
C) 2 D) 1
Solution:
Solution:
B) 42437
C) 2
Explanation:
Explanation:
The square of a natural number never
4456/6, remainder is 4.
ends in 7.
Required number = (6-4) = 2.

pg. 131
10 Seconds

42437 is not the square of a natural Explanation:


number
LCM of 2, 3, 7 is 42.
=> (700 – 300)/42 = 9 22/42 => 9
28. If x and y are the two digits of the Numbers.
number 653xy such that this number is
divisible by 80, then x+y is equal to?
A) 5 B) 4 31. The difference between a positive
proper fraction and its reciprocal is 9 / 20.
C) 3 D) 2 Then the fraction is :
Solution: A) 3/5 B) 3/10
D) 2 C) 4/5 D) 5/4
Explanation: Solution:
Since 653xy is divisible by 5 as well as 2, C) 4/5
so y = 0.
Explanation:
Now, 653x0 must be divisible by 8.
Let the required fraction be x. Then, (1 / x
So, 3x0 must be divisible by 8. This )- x = 9/20
happens when x = 2
1 - x^(2) / x = 9 / 20 => 20 - 20 * x^(2) = 9
x + y = (2 + 0) = 2. * x.
20 * x^(2) + 9 *x - 20 = 0.
29. A number when divided by 779 gives a => (4 * x + 5) (5 * x - 4) = 0.
remainder 47. By dividing the same
number by 19, what would be the => x = 4 / 5.
remainder?
A)9 B)10 32. What is the smallest number by which
C)12 D)13 2880 must be divided in order to make it
into a perfect square?
Solution:
A) 2 B) 3
Number = ( 779 x a) + 47, where "a" is the
quotient C) 4 D) 5

= (19 x 41 x a) + (19 x 2) + 9 Solution:

= 19 x (41a + 2) + 9 D) 5

= 19 x (New quotient) + 9 Explanation:

therefore Required remainder = 9 By trial and error method, we get


2880/3 = 960 is not a perfect square

30. How many numbers up to 300 and 700 2880/4 = 720 is not a perfect square
are divisible by 2, 3 and 7 both together? 2880/5 = 576 which is perfect square of 24
A) 7 B) 9 Hence, 5 is the least number by which
C) 11 D) 5 2880 must be divided in order to make it
into a perfect square.
Solution:
B) 9

pg. 132
10 Seconds

33. The sum of the two numbers is 12 and 36. A piece of ribbon 4 yards long is used
their product is 35. What is the sum of the to make bows requiring 15 inches of
reciprocals of these numbers? ribbon for each. What is the maximum
number of bows that can be made?
A) 12/35 B) 1/35
A) 8 B) 9
C) 35/8 D) 7/32
C) 10 D) 11
Solution:
Solution:
A) 12/35
B) 9
Explanation:
Explanation:
Let a and b are the numbers. Then a+b is
12 and ab is 35. The maximum number of bows will be 4
yards (= 4 x 36 inches) divided by 15
a+b/ab = 12/35 inches.
1/b + 1/a = 12/35 This gives 9.6. But as a fraction of a bow
is no use, we can only make 9 bows.

34. What is the right most integer of the


expression 65776˄759+ 54697 ˄467 ? 37. What is the greatest of 3 consecutive
A) 4 B) 6 integers whose sum is 24 ?

C) 9 D) 0 A) 6 B) 7

Solution: C) 8 D) 9

C) 9 Solution:

Explanation: D) 9

Last digit for the power of 6 is 6 (always) Explanation:

Power cycle of 7 is 7, 9, 3, 1. The sum of three consecutive integers can


be written as n + (n + 1) + (n + 2) = 3n + 3
Now 467/4 gives a remainder of 3
If the sum is 24, we need to solve the
Then the last digit is 7 ˄3 = 3 equation 3n + 3 = 24;
Last digit is 6 + 3 = 9. => 3n = 21;
=> n = 7
35. Which natural number is nearest to The greatest of the three numbers is
8485, which is completely divisible by 75? therefore 7 + 2 = 9
A) 8475 B) 8500
C) 8550 D) 8525 38. n is a whole number which when
divided by 4 gives 3 as remainder. What
Solution: will be the remainder when 2*n is divided
A) 8475 by 4 ?

Explanation: A) 3 B) 2

8485/75, remainder is 10. C) 1 D) 0

Required number = (8485-10)=8475. Solution:


B) 2

pg. 133
10 Seconds

Explanation: them and every four used a bowl of meat


between them. There were altogether 65
Let n=4*q + 3. Then, 2*n = 8*q + 6 = 4(2*q dishes. How many guests were present at
+ 1) + 2. the party?
Thus when 2*n is divided by 4, the A) 60 B) 74
reminder is 2.
C) 82 D) 58
Solution:
39. The product of a number and its
multiplicative inverse is A) 60
A) 1 B) 0 Explanation:
C) -1 D) Infinity Let,
Solution: no. of rice bowl = x
A) 1 no. of dal bowl = y
Explanation: no. of meat bowl = z
The multiplicative inverse of a number is Then, x + y + z = 65 ....(1)
nothing but a reciprocal of a number.
Also given that, 2x = 3y = 4z
Now, the product of a number and its
multiplicative inverse is always equal to 1. substituting value of y & z in terms of x i.e.
y = 2x/3 and z = x/2 in eq (1),
For example :
We get x = 30.
Let the number be 15
As 1 rice bowl is shared between 2
Multiplicative inverse of 15 = 1/15 guests,
The product of a number and its Therefore, there are 60 guests.
multiplicative inverse is = 15 x 1/15 = 1.

42. What will come in place of the question


40. If V = 12R / (r + R) , then R = mark in the series below?
A) Vr / (12 - V) B) Vr + (V /12 ) 2, 17, 89, 359, 1079, ?
C) V D) V / (r - 12 ) A) 2137 B) 2121
Solution: C) 2377 D) 2159
A) Vr / (12 - V) Solution:
Explanation: D) 2159
We have to rearrange the equation to Explanation:
make R the subject.
Ans.
Start by cross multiplying by (r + R); V (r +
R) = 12R 17 = 2 x 6 + 5

Multiply out the bracket Vr + VR = 12R 89 = 17 x 5 + 4


359 = 89 x 4 + 3

41. At a dinner party every two guests 1079 = 359 x 3 + 2


used a bowl of rice between them, every ? = 1079 x 2 + 1, i.e. ? = 2159
three guests used a bowl of dal between

pg. 134
10 Seconds

will be a cube (with cube root = difference


squared).
43. Find the number of factors of 9321 ?
A) 4 B) 5
45. There are two numbers such that the
C) 7 D) 8 sum of twice the first number and thrice
Solution: the second number is 100 and sum of
thrice the first number and twice the
D) 8 second number is 120. Which is the larger
number?
Explanation:
A) 32 B) 24
9321 = 3 x 13 x 239 = 31 x 13 1 x 239 1 ;
add one to all the powers ie., C) 26 D) 38
(1+1) x (1+1) x (1+1)= 8 ; Solution:
we get eight factors. A) 32
Explanation:
44. A perfect cube is an integer whose Let the first number be K and second
cube root is an integer. For example, 27, number be L
64 and 125 are perfect cubes. If p and q
are perfect cubes, which of the following Then, 2K + 3L = 100.....(1)
will not necessarily be a perfect cube? 3K + 2L = 120.......(2)
A) 8p B) pq solving (1)&(2), we get
C) pq+27 D) -p K = 32 and L = 12
Solution: Hence the largest number is 32.
C) pq+27
Explanation: 46. If a positive integer n, divided by 5 has
A perfect cube will have prime factors that a remainder 2, which of the following must
are in groups of 3; for example 125 has be true?
the prime factors 5 x 5 x 5 , and 64 x 125 A) n is odd
will also be a cube because its factors will
be 4 x 4 x 4 x 5 x 5 x 5 B) n + 1 cannot be a prime number
Consider the answer choices in turn. C) (n + 2) divided by 7 has remainder 2
8 is the cube of 2, and p is a cube, and so D) n + 3 is divisible by 5
the product will also be a cube.
Solution:
pq will also be a cube as shown above.
D) n + 3 is divisible by 5
pq is a cube and so is 27, but their sum
need not be a cube. Consider the case Explanation:
where p =1 and q = 8, the sum of pq and You can find the integers which when
27 will be 35 which has factors 5 x 7 and divided by 5 have a remainder 2 by adding
is not a cube. 2 to all multiples of 5. So we have n = 7 ,
-p will be a cube. 12, 17, 22 etc.

Since the difference between p and q is From this series we can see that n does
raised to the power of 6, this expression not have to be odd.

pg. 135
10 Seconds

Also n + 1 can be a prime because, for 49. 3x + y = 19 and x + 3y = 1. Find the


example, 12 + 1 = 13 value of 2x + 2y
And (n + 2) / 7 has a remainder 2 in some A) 20 B) 18
cases but not all.
C) 11 D) 10
Remember the question asks us for what
MUST be true, and we see that none of Solution:
the statements are true in all cases. D) 10
However, adding 3 to any of the values of
n will always give a multiple of 5. Explanation:
To solve a pair of simultaneous equations
such as those given we can add or
47. What smallest number of 6 digit is subtract them.
divisible by 111?
Adding we get 4x + 4y = 20
A)100011 B)100012
Therefore 2x + 2y = 10
C)100001 D)109090
Solution:
50. Direction: What should come in the
Smallest number of 6 digits is 100000 place of question mark (?) in the following
on dividing 100000 by 111 we get 100 as number series?
remainder 9, 11, 16, 26, ?, 69
therefore Number to be added = (111 - A) 31 B) 38
100) = 11
C) 43 D) 45
therefore Required Number = 100011
Solution:
C) 43
48. What will come in place of the question
mark in the series below? Explanation:
7, 4.5, 5.5, 12, 49, ? Double Difference pattern is followed in
the above series. +2, +5, + 10, +17, +26
A) 393 B) 396
C) 354 D) 367
51. What will come in place of the question
Solution: mark in the series below?
A) 393 3, 5, 15, 45, 113, ?
Explanation: A) 250 B) 243
Ans. C) 247 D) 261
4.5 = 7 x 0.5 + 1 Solution:
5.5 = 4.5 x 1 + 1 B) 243
12 = 5.5 x 2 + 1 Explanation:
49 = 12 x 4 + 1 Ans.
? = 49 x 8 + 1, i.e. ? = 393 5 = 3 + (13 + 1)
15 = 5 + (23 + 2)

pg. 136
10 Seconds

45 = 15 + (33 + 3) => 3x = 87
113 = 45 + (43 + 4) => x = 29 => 29, 31, 33 are three
consecutive odd numbers.
? = 113 + (53 + 5), i.e. ? = 243
Therefore, the middle number is 31.

52. Courier charges for packages to a


certain destination are 65 cents for the first 54. For how many integer values of n will
250 grams and 10 cents for each the value of the expression 4n + 7 be an
additional 100 grams or part thereof. What integer greater than 1 and less than 200?
could be the weight in grams of a package
for which the charge is $1.55 ? A) 48 B) 49

A) 1155 B) 1145 C) 50 D) 51

C) 1040 D) None Solution:

Solution: C) 50

B) 1145 Explanation:

Explanation: 1 < 4n + 7 < 200

The weight will be 250g plus (1.55 - n can be 0, or -1


0.65)/0.10 units of 100g n cannot be -2 or any other negative
250 + 900 = 1150 integer or the expression 4n + 7 will be
less than1.
This is the maximum weight that can be
sent at that price. But, weights exceeding The largest value for n will be an integer <
(200 - 7) /4
250 + 800 will also get charged this
amount (that is what the ‘part thereof’ 193/4 = 48.25, hence 48
implies). The number of integers between -1 and 48
Hence a package weighing 1145 will be inclusive is 50
charged $1145

55. A box contains 90 screws each of 100


53. The sum of three consecutive odd gms and 100 bolts each of 150 gms. If the
numbers is 93. What is the middle entire box weighs 35.5 kg, then the weight
number? of the empty box is ?

A) 31 B) 33 A) 10.5 kgs B) 12 kgs

C) 29 D) 27 C) 9.6 kgs D) 11.5 kgs

Solution: Solution:

A) 31 D) 11.5 kgs

Explanation: Explanation:

Let the three consecutive odd numbers be Screws weight = 90x100 = 9000gms
x, x+2, x+4 bolts weight = 100 x 150 = 15000gms
Then, weight = screws + bolts => 24000 gms
x + x + 2 + x + 4 = 93 =>24 kg

=> 3x + 6 = 93 Given entire box weight = 35.5kg

pg. 137
10 Seconds

empty box = entire box weight - weight => 58. Direction: What should come in the
35.5kg - 24kg => 11.5kg place of question mark (?) in the following
number series?
so the empty box is 11.5kg.
7, 10, 16, 28, ?, 100
A) 34 B) 40
56. Find the remainder when 1234 x 1235
x 1237 is divided by 8. C) 52 D) 60
A) 4 B) 2 Solution:
C) 0 D) 6 C) 52
Solution: Explanation:
D) 6 The pattern followed is +3 , +6 , +12 , +24
, +48
Explanation:
When we divide 1234 by 8, remainder is 2
59. About 4 out of 5 students are left
When we divide 1235 by 8, remainder is 3 handed. If there are totally 40 students in
When we divide 1237 by 8, remainder is 5 an class. How many would be left
handed?
---> 2 x 3 x 5 = 30
A) 40 B) 45
As 30 will not be the remainder because it
is greater than 8, when 30 divided by 8, C) 32 D) 30
remainder = 6. Solution:
C) 32
57. A straight fence is to be constructed Explanation:
from posts 6 inches wide and separated
by lengths of chain 5 feet long. If a certain Probability of left handed = 4/5
fence begins and ends with a post, which
of the following could be the length of the total student = 40
fence in feet? (12 inches = 1 foot). total left handed = 40 x(4/5)
A) 17 B) 18 = 32
C) `19 D) 20
Solution: 60. A sequence of natural number
A) 17 2,3,5,6,10. Such that no square and no
cube occurs in the term. Find the 300th
Explanation: term of sequence ?
The fence will consist of one more post A) 322 B) 319
than there are chains. (e.g. P-c-P-c-P).
C) 321 D) 320
Therefore, a total length has to be a
multiple of the length of the chain plus one Solution:
post (5.5) plus one post extra.We have D) 320
length = (5.5n + 0.5), where n can be any
positive whole number. If n= 3, length =17 Explanation:
1 to 300 = 300 numbers
squares btwn 1 to 300 =
1,4,9,16,25,36,49.....289 i.e, total 17

pg. 138
10 Seconds

cubes = 1,8,27,64,125,216 i.e, 5 2 In how many different ways can the


letters of the word 'LEADING' be arranged
now 1 and 64 are common so total in such a way that the vowels always
numbers that should be removed ( 17+5- come together?
2= 20 )
A) 720 B) 520
total 20 term removed so 300th term
would be 320 C) 700 D) 750
Solution:
PERMUTATIONS AND COMBINATIONS The word 'LEADING' has 7 different
letters.
Permutation formula:
When the vowels EAI are always together,
Permutation is defined as arrangement of they can be supposed to form one letter.
r things that can be done out of total n
things. This is denoted by nPr which is Then, we have to arrange the letters
equal to n!/(n-r)! LNDG (EAI).
Combination formula Now, 5 (4 + 1) letters can be arranged in
5! = 120 ways.
Combination is defined as selection of r
things that can be done out of total n The vowels (EAI) can be arranged among
things. This is denoted by nCr which is themselves in 3! = 6 ways.
equal to n!/r!(n-r)!
Required number of ways = (120 x 6) =
As per the Fundamental Principle of 720.
Counting, if a particular thing can be done
in m ways and another thing can be done
in n ways, then either one of the two can 3. A committee of 5 persons is to be
be done in m + n ways and both of them formed from 6 men and 4 women. In how
can be done in m × n ways. many ways can this be done when at least
Questions: 2 women are included?

1. Out of 7 consonants and 4 vowels, how A) 196 B) 186


many words of 3 consonants and 2 vowels C) 190 D) 200
can be formed?
Solution:
A) 25200 B) 52000
When at least 2 women are included. The
C) 120 D) 24400 committee may consist of 3 women, 2
Solution: men : It can be done in 4C3*6C2 ways

Number of ways of selecting (3 or, 4 women, 1 man : It can be done in


consonants out of 7) and (2 vowels out of 4C4*6C1ways
4) = (7C3*4C2) = 210. or, 2 women, 3 men : It can be done in
Number of groups, each having 3 4C2*6C3 ways.
consonants and 2 vowels = 210. Total number of ways of forming the
Each group contains 5 letters. committees

Number of ways of arranging 5 letters = 4C2*6C3+4C3*6C2+4C4*6C1


among themselves = 5! = 120 = 6 x 20 + 4 x 15 + 1x 6
Required number of ways = (210 x 120) = = 120 + 60 + 6 =186
25200.

pg. 139
10 Seconds

4. A college has 10 basketball players. A Solution:


5-member team and a captain will be
selected out of these 10 players. How The word 'OPTICAL' contains 7 different
many different selections can be made? letters.

A) 1260 B) 1400 When the vowels OIA are always together,


they can be supposed to form one letter.
C) 1250 D) 1600
Then, we have to arrange the letters PTCL
Solution: (OIA).
A team of 6 members has to be selected Now, 5 letters can be arranged in 5! = 120
from the 10 players. This can be done in ways.
10C6 or 210 ways.
The vowels (OIA) can be arranged among
Now, the captain can be selected from themselves in 3! = 6 ways.
these 6 players in 6 ways.
Required number of ways = (120 x 6) =
Therefore, total ways the selection can be 720.
made is 210×6= 1260

8. How many 4-letter words with or without


5.In how many ways can a group of 5 men meaning, can be formed out of the letters
and 2 women be made out of a total of 7 of the word, 'LOGARITHMS', if repetition
men and 3 women? of letters is not allowed?
A) 135 B) 63 A) 4050 B) 3600
C) 125 D) 64 C) 1200 D) 5040
Solution: Solution:
Required number of ways = (7C5*3C2) = 'LOGARITHMS' contains 10 different
(7C2*3C1) = 63 letters.
Required number of words = Number of
arrangements of 10 letters, taking 4 at a
6. 12 people at a party shake hands once time.
with everyone else in the room. How many
handshakes took place? = 10P4
A) 72 B) 66 = 5040.
C) 76 D) 64
Solution: 9. The Indian Cricket team consists of 16
players. It includes 2 wicket keepers and 5
There are 12 people, so this is our n bowlers. In how many ways can a cricket
value. eleven be selected if we have to select 1
So, 12C21= 66 wicket keeper and at least 4 bowlers?
A) 1024 B) 1900

7. In how many different ways can the C) 2000 D) 1092


letters of the word 'OPTICAL' be arranged Solution:
so that the vowels always come together?
D) 1092
A) 360 B) 700
Explanation:
C) 720 D) 120

pg. 140
10 Seconds

We are to choose 11 players including 1 Let us mark these positions as under:


wicket keeper and 4 bowlers or, 1 wicket
keeper and 5 bowlers. (1) (2) (3) (4) (5) (6)

Number of ways of selecting 1 wicket Now, 3 vowels can be placed at any of the
keeper, 4 bowlers and 6 other players in three places out 4, marked 1, 3, 5.
2C1*5C4*9C6 = 840 Number of ways of arranging the vowels =
3P3 = 3! = 6.
Number of ways of selecting 1 wicket
keeper, 5 bowlers and 5 other players in Also, the 3 consonants can be arranged at
2C1*5C5*9C5 =252 the remaining 3 positions.

Total number of ways of selecting the Number of ways of these arrangements =


team = 840 + 252 = 1092 3P3 = 3! = 6.
Total number of ways = (6 x 6) = 36.

10. When four fair dice are rolled


simultaneously, in how many outcomes 12. In how many ways can the letters of
will at least one of the dice show 3? the word EDUCATION be rearranged so
A) 620 B) 671 that the relative position of the vowels and
consonants remain the same as in the
C) 625 D) 567 word EDUCATION?
Solution: A) 4! x 4! B) 5! x 5!
B) 671 C) 4! x 5! D) 3! x 4!
Explanation: Solution:
When 4 dice are rolled simultaneously, C) 4! x 5!
there will be a total of 6 x 6 x 6 x 6 = 1296
outcomes. Explanation:

The number of outcomes in which none of The word EDUCATION is a 9 letter word,
the 4 dice show 3 will be 5 x 5 x 5 x 5 = with none of the letters repeating.
625 outcomes. The vowels occupy 3rd,5th,7th and 8th
Therefore, the number of outcomes in position in the word and the remaining 5
which at least one die will show 3 = 1296 positions are occupied by consonants
– 625 = 671 As the relative position of the vowels and
11. In how many different ways can the consonants in any arrangement should
letters of the word 'DETAIL' be arranged in remain the same as in the word
such a way that the vowels occupy only EDUCATION, the vowels can occupy only
the odd positions? the afore mentioned 4 places and the
consonants can occupy1st,2nd,4th,6th
A) 36 B) 25 and 9th positions.
C) 42 D) 120 The 4 vowels can be arranged in the
3rd,5th,7th and 8th position in 4! Ways.
Solution:
Similarly, the 5 consonants can be
A) 36 arranged in1st,2nd,4th,6th and 9th
Explanation: position in5! Ways. Hence, the total
number of ways = 4! × 5!
There are 6 letters in the given word, out
of which there are 3 vowels and 3
consonants.

pg. 141
10 Seconds

13. From 5 consonants and 4 vowels, how C) 254 D) 216


many words can be formed using 3
consonants and 2 vowels? Solution:

A) 7600 B) 7200 A) 215

C) 6400 D) 3600 Explanation:

Solution: Since each ring consists of six different


letters, the total number of attempts
B) 7200 possible with the three rings is = 6 x 6 x 6
= 216. Of these attempts, one of them is a
Explanation: successful attempt.
From 5 consonants, 3 consonants can be Maximum number of unsuccessful
selected in 5C3 ways. attempts = 216 - 1 = 215.
From 4 vowels, 2 vowels can be selected
in 4C2 ways.
16. If the letters of the word CHASM are
Now with every selection, number of ways rearranged to form 5 letter words such that
of arranging 5 letters is 5P5ways. none of the word repeat and the results
Total number of words = 5C3*4C2*5P5 arranged in ascending order as in a
dictionary what is the rank of the word
= 10x 6 x 5 x 4 x 3 x 2 x 1= 7200 CHASM?
A) 32 B) 24
14. In how many ways can 4 girls and 5 C) 72 D) 36
boys be arranged in a row so that all the
four girls are together? Solution:

A) 18000 B) 17280 A) 32

C) 17829 D) 18270 Explanation:

Solution: The 5 letter word can be rearranged in


5!=120 Ways without any of the letters
B) 17280 repeating.
Explanation: The first 24 of these words will start with
A.
Let 4 girls be one unit and now there are 6
units in all. Then the 25th word will start will CA _ _ _.
They can be arranged in 6! ways. The remaining 3 letters can be rearranged
in 3!=6 Ways. i.e. 6 words exist that start
In each of these arrangements 4 girls can
with CA.
be arranged in 4! ways.
The next word starts with CH and then A,
Total number of arrangements in which
i.e., CHA _ _.
girls are always together = 6! x 4!= 720 x
24 = 17280 The first of the words will be CHAMS. The
next word will be CHASM.
Therefore, the rank of CHASM will be
15. A letter lock consists of three rings
24+6+2= 32
each marked with six different letters. The
number of distinct unsuccessful attempts
to open the lock is at the most?
17. 5 men and 4 women are to be seated
A) 215 B) 268 in a row so that the women occupy the

pg. 142
10 Seconds

even places. How many such holds the first two toys and which one
arrangements are possible? holds the second set of two toys will look
the same. Hence, we need to divide the
A) 2880 B) 1440 result by 2
C) 720 D) 2020 Therefore, total number of ways of
Solution: achieving the 2 - 2 - 1 option is ways
5C2*3C2= 15 ways
A) 2880
Case b. Number of ways of achieving the
Explanation: second option 3 - 1 - 1
There are total 9 places out of which 4 are Three toys out of the 5 can be selected in
even and rest 5 places are odd. 5C3 ways. As the boxes are identical, the
remaining two toys can go into the two
4 women can be arranged at 4 even
identical looking boxes in only one way.
places in 4! ways.
Therefore, total number of ways of getting
and 5 men can be placed in remaining 5
the 3 - 1 - 1 option is 5C3 = 10 = 10 ways.
places in 5! ways.
Total ways in which the 5 toys can be
Hence, the required number of
packed in 3 identical boxes
permutations = 4! x 5! = 24 x 120 = 2880
= number of ways of achieving Case a +
number of ways of achieving Case b= 15
18. In how many ways can 5 different toys + 10 = 25 ways.
be packed in 3 identical boxes such that
no box is empty, if any of the boxes may
hold all of the toys? 19. How many lines can you draw using 3
non collinear (not in a single line) points A,
A) 36 B) 25
B and C on a plane?
C) 24 D) 72
A) 3 B) 6
Solution:
C) 2 D) 4
B) 25
Solution:
Explanation:
A) 3
The toys are different; The boxes are
Explanation:
identical
You need two points to draw a line. The
If none of the boxes is to remain empty,
order is not important. Line AB is the same
then we can pack the toys in one of the
as line BA. The problem is to select 2
following ways
points out of 3 to draw different lines. If we
a. 2, 2, 1 proceed as we did with permutations, we
get the following pairs of points to draw
b. 3, 1, 1 lines.
Case a. Number of ways of achieving the AB , AC
first option 2 - 2 - 1
BA , BC
Two toys out of the 5 can be selected in
5C2 ways. Another 2 out of the remaining CA , CB
3 can be selected in 3C2 ways and the
There is a problem: line AB is the same as
last toy can be selected in 1C1 way.
line BA, same for lines AC and CA and BC
However, as the boxes are identical, the and CB.The lines are: AB, BC and AC ; 3
two different ways of selecting which box lines only.

pg. 143
10 Seconds

So in fact we can draw 3 lines and not 6 C) 11760


and that's because in this problem the
order of the points A, B and C is not Explanation: Required number of ways =
important. (8C5*10C6) = (8C3*10C4) = 11760

20. From a group of 7 men and 6 women, 23. In the below word how many words
five persons are to be selected to form a are there in which R and W are at the end
committee so that at least 3 men are there positions? RAINBOW
on the committee. In how many ways can A) 120 B) 180
it be done?
C) 210 D) 240
A) 564 B) 735
Solution:
C) 756 D) 657
D) 240
Solution:
Explanation:
C) 756
When R and W are the first and last letters
Explanation: of all the words then we can arrange them
We may have (3 men and 2 women) or (4 in 5! ways. Similarly, When W and R are
men and 1 woman) or (5 men only). the first and last letters of the words then
the remaining letters can be arrange in 5!
Required number of ways= ways.
(7C3*6C2)+(7C4*6C1)+7C5 = 756.
Thus, the total number of permutations = 2
x 5! = 2 x 120 = 240
21. In a group of 6 boys and 4 girls, four
children are to be selected. In how many
different ways can they be selected such 24. In a box, there are 5 black pens, 3
that at least one boy should be there? white pens and 4 red pens. In how many
ways can 2 black pens, 2 white pens and
A) 209 B) 290 2 red pens can be chosen?
C) 200 D) 208 A) 180 B) 220
Solution: C) 240 D) 160
We may have (1 boy and 3 girls) or (2 Solution:
boys and 2 girls) or (3 boys and 1 girl) or
(4 boys). A) 180

Required number of ways = Explanation:


(6C1*4C3)+(6C2*4C2)+(6C3*4C1)+6C4 Number of ways of choosing 2 black pens
= (6C1*4C1)+(6C2*4C2)+(6C3*4C1)+6C2 from 5 black pens in 5C2 ways.
= 209. Number of ways of choosing 2 white pens
from 3 white pens in 3C2 ways.

22. In how many ways a committee, Number of ways of choosing 2 red pens
consisting of 5 men and 6 women can be from 4 red pens in 4C2ways.
formed from 8 men and 10 women? By the Counting Principle, 2 black pens, 2
A) 53400 B) 17610 white pens, and 2 red pens can be chosen
in 10 x 3 x 6 =180 ways.
C) 11760 D) 45000
Solution:

pg. 144
10 Seconds

25. How many arrangements of the letters Explanation:


of the word ‘BENGALI’ can be made if the
vowels are to occupy only odd places. There are 8 students and the maximum
capacity of the cars together is 9.
A) 720 B) 576
We may divide the 8 students as follows
C) 567 D) 625
Case I: 5 students in the first car and 3 in
Solution: the second
B) 576 Case II: 4 students in the first car and 4 in
the second
Explanation:
Hence, in Case I: 8 students are divided
There are 7 letters in the word Bengali of into groups of 5 and 3 in8C3 ways.
these 3 are vowels and 4 consonants.
Similarly, in Case II: 8 students are divided
There are 4 odd places and 3 even into two groups of 4 and 4 in 8C4ways.
places. 3 vowels can occupy 4 odd places
in 4P3 ways and 4 constants can be Therefore, the total number of ways in
arranged in 4P4 ways. which 8 students can travel is:
Number of words =4P3 x 4P4= 24 x 24 = 8C3+8C4=56 + 70= 126
576

28. How many parallelograms will be


26. There are 7 non-collinear points. How formed if 7 parallel horizontal lines
many triangles can be drawn by joining intersect 6 parallel vertical lines?
these points?
A) 215 B) 315
A) 10 B) 30
C) 415 D) 115
C) 35 D) 60
Solution:
Solution:
B) 315
C) 35
Explanation:
Explanation:
Parallelograms are formed when any two
A triangle is formed by joining any three pairs of parallel lines (where each pair is
non-collinear points in pairs. not parallel to the other pair) intersect.
There are 7 non-collinear points Hence, the given problem can be
considered as selecting pairs of lines from
The number of triangles formed = 7C3 = the given 2 sets of parallel lines.
35
Therefore, the total number of
parallelograms formed = 7C2 x 6C2 = 315
27. A team of 8 students goes on an
excursion, in two cars, of which one can
seat 5 and the other only 4. In how many 29. In how many different ways can the
ways can they travel? letters of the word 'THERAPY' be
arranged so that the vowels never come
A) 126 B) 120 together?
C) 146 D) 156 A) 1440 B) 720
Solution: C) 2250 D) 3600
A) 126 Solution:

pg. 145
10 Seconds

D) 3600 NUMBER SERIES


Explanation: 1.Find the missing number in the following
series?
Given word is THERAPY.
3, 5, 5, 19, 7, 41, 9?
Number of letters in the given word = 7
A) 71 B) 61
These 7 letters can be arranged in 7!
ways. C) 79 D) 69
Number of vowels in the given word = 2 Solution:
(E, A)
A) 71
The number of ways of arrangement in
which vowels come together is 6! x 2! Explanation:
ways First series : 3, 5, 7, 9
Hence, the required number of ways can Second series : 5, 19, 41, ?
the letters of the word 'THERAPY' be
arranged so that the vowels never come Difference of Second series are 14, 22, 30
together = 7! - (6! x 2!) ways = 5040 - etc
1440 = 3600 ways.
Next term is 41+30 i.e equal to 71.

30. There are 4 books on fairy tales, 5


2. What should come in place of question
novels and 3 plays. In how many ways
mark (?) in the following number series?
can you arrange these so that books on
fairy tales are together, novels are 132 156 ? 210 240
together and plays are together and in the 272
order, books on fairy tales, novels and
plays? A) 196 B) 182

A) 12400 B) 17820 C) 199 D) 204

C) 17280 D) 12460 Solution:

Solution: B) 182

C) 17280 Explanation:

Explanation: The given series follows a logic that

There are 4 books on fairy tales and they 11 x 12, 12 x 13, 13 x 14, 14 x 15, 15 x
have to be put together. They can be 16,...
arranged in 4! ways. So the missing number is 13 x 14 = 182
Similarly, there are 5 novels. They can be
arranged in 5! ways.
3. Find the wrong number in the series.
And there are 3 plays. They can be
arranged in 3! ways. 3, 8, 15, 24, 34, 48, 63
So, by the counting principle all of them A) 15 B) 24
together can be arranged in 4!´5!´3! ways
= 17280 C) 34 D) 48
Solution:
C) 34
Explanation:

pg. 146
10 Seconds

The difference between consecutive 15, 51, 216, 1100, ?, 46452


numbers of the given series are
respectively 5, 7, 9, 11, 13, and 15. A) 5660 B) 6560

Therefore, 24+11=35 But in your problem C) 6750 D) 6630


it is given as 34.so 34 is wrong number Solution:
D) 6630
4. Find the missing number in the given Explanation:
number series?
Let first no. be "k" and multiple be "l"...start
625, 625, 600, ?, 475, 875 value of y with 3
A) 545 B) 700 then,
C) 675 D) 725 [(k x y) + {l x (l-1)}]
Solution: Thus, we get
B) 700 15 x 3 + 3 x 2 = 51
Explanation: 51 x 4 + 4 x 3 = 216
Here the given number series 625, 625, 216 x 5 + 5 x 4 = 1100
600, ?, 475, 875 follows a pattern that
1100 x 6 + 6 x 5 = 6630
625
6630 x 7 + 7 x 6 = 46452
625 + (0 x 0) = 625
Therefore, the missing number is 6630.
625 - (5 x 5) = 625 - 25 = 600
600 + (10 x 10) = 600 + 100 = 700
6. Find out the wrong term in the series 2,
700 - (15 x 15) = 700 - 225 = 475 3, 4, 4, 6, 8, 9, 12, 16
475 + (20 x 20) = 475 + 400 = 875 A) 9 B) 12
Hence, the missing number in the given C) 16 D) 8
number series is 700
Solution:
A) 9
4. What letters will fill the blank?
Explanation:
Z, X, V, T, R, __,_
The given sequence is a combination of
A) M, N B) P, N three series:
C) O, K D) K, S i)1st, 4th, 7th terms i.e, 2, 4, 9
Solution: ii)2nd, 5th, 8th terms i.e, 3, 6, 12
B) P, N iii)3rd, 6th, 9th terms i.e, 4, 8, 16
Explanation: In each one of i, ii, iii, each term is twice
Here the letter series is alternative reverse the preceding term.
order series. So we get P, N. So, 9 is wrong and must be replaced by
(4*2) = 8

5. Find the missing number in the given


series?

pg. 147
10 Seconds

7. Find the missing term in the following A) 188 B) 256


series?
C) 240 D) 180
10000, 11000, 9900, 10890, ?, 10781
Solution:
A) 10423 B) 9801
C) 240
C) 10241 D) 9712
Explanation:
Solution:
The pattern is x1+3, x 2, x1+3, x 3, x1+3,
B) 9801 x4
Explanation: The next number=60x4=240
Clearly, alternatively we add and subtract
10% of a term to obtain the next term of
the series. 10. Complete the given number series?

Thus, 10000 + (10% of 10000)= 11000; 8, 14, 24, 40, ?

11000 - (10% of 11000) = 9900; A) 65 B) 84

9900 + (10% of 9900) = 10890; C) 96 D) 102

10890 - (10% of 10890) = 9801. Solution:

9801 + (10% of 9801) = 9801+980 = A) 65


10781. Explanation:
So, missing term = 10890- (10% of The given number series is 8, 14, 24, 40,
10890)= 9801. ?.
It follows a pattern that,
8. Which Number would come in place of 8 x 1.5 + 2 = 14
the underline mark in the series.
14 x 1.5 + 3 = 24
3, 9, 21, ___ , 93
24 x 1.5 + 4 = 40
A) 39 B) 45
40 x 1.5 + 5 = 65
C) 48 D) 51
Hence, the next number in the given
Solution: number series is 65.
B) 45
Explanation: 11. What comes next in the series...
First difference is 6 and second is 12. But 1A2, 3C4, 7G8, ___
if we take third as 18, then fourth
difference will be 24. Then 93 cannot be A) 15016 B) 16P17
the last term.
C) 14015 D) 13P14
So, we have to take the difference as 6,
Solution:
12, 24 and 48.
A) 15016
So, Answer is 21+24 = 45
Explanation:
The first digit is formed by adding the
9. Find the next number in the series 5, 8,
numbers which are the results of powers
16, 19, 57, 60, .....
of 2 increased by one and second letter is

pg. 148
10 Seconds

the corresponding alphabet to that 36 x 4 = 144


number...last number is the next number
to the first one...therefore 144 x 5 = 720

15=7+8 (i.e. 2^3) 720 x 6 = 4320

O is corresponding alphabet to 15 Hence, the next number in the given


number series is 4320.
and 16 comes after 15

15.Choose the missing terms out of the


12. What will come in place of question given alternatives Z, X, S, I, R, R, ?, ?
mark(?) in the following number series?
A) J, I B) K, M
6859 5832 ? 4096 3375
C) G, I D) J, K
A) 4589 B) 4913
Solution:
C) 5013 D) 5169
C) G, I
Solution:
Explanation:
B) 4913
Z-2=X
Explanation:
X-5=S
The given series follows a pattern that,
S - 10 = I
19 x 19 x 19 = 6859
I - 17 = R
18 x 18 x 18 = 5832
R - 26 = R
17 x 17 x 17 = 4913
R - 37 = G
16 x 16 x 16 = 4096
G - 50 = I
15 x 15 x 15 = 3375
The series 2, 5, 10, 17, 26, 37, 50 is
So, the missing number in the given series obtained by the pattern +3, +5, +7, +9, . . .
= 4913 .

13. Find the next number in the given 15. Look at the series:
number series?
3, 7, 24, 11, __, 19, 19,... What number
6, 6, 12, 36, 144, 720, ? should fill the blank?
A) 4320 B) 3547 A) 14 B) 13
C) 2154 D) 1765 C) 21 D) 16
Solution: Solution:
A) 4320 Two series alternate here, with every third
number following a different pattern.
Explanation:
In the main series, 4 is added to each
The given series is 6, 6, 12, 36, 144, 720,? number to arrive the next.
6x1=6 In alternate series, 5 is subtracted from
6 x 2 = 12 each number t arrive the next number.

12 x 3 = 36 Here in the main series

pg. 149
10 Seconds

3+4=7 nine = 4(number of letters)


7 + 4 = 11 sixty = 5(number of letters)
11 + 4 = 15 ninety = 6(number of letters)
15 + 4 = 19 seventy = 7(number of letters)
In alternate series sixty six = 8(number of letters)
24 - 5 = 19 ninety six=96(number of letters)
19 - 5 = 14... Then, likewise choose the number which
has 9 letters in its Spelling.

16. Find the next number in the given


number series 18. What will come in the place of the (?)
in the following number series?
1, 3, 10, 41, 206, ?
5 14 45 130 (?) 1182
A) 1237 B) 897
A) 222 B) 397
C) 987 D) 1007
C) 404 D) 415
Solution:
Solution:
A) 1237
B) 397
Explanation:
Explanation:
The given number series is 1, 3, 10, 41,
206, ? The pattern of given series is :
It follows a pattern that, 14 = (5x3) - 1
1x2+1=3 45 = (14x3) + 3
3 x 3 + 1 = 10 130 = (45x3) - 5
10 x 4 + 1 = 41 (?) = (130x3) + 7
41 x 5 + 1 = 206 => (?) = 397
206 x 6 + 1 = 1237 1182 = (397x3) - 9
Hence, the next number in the given Thus, the missing number is 397.
number series is 1237.

19. Find out the wrong term in the series


17.What is the next number in series? 2, 5, 10, 50, 500, 5000
10, 9, 60, 90, 70, 66, ? A) 5000 B) 500
A) 73 B) 19 C) 10 D) 50
C) 96 D) 22 Solution:
Solution: A) 5000
C) 96 Explanation:
Explanation: Each term of the series is the product of
the preceding two terms
ten = 3(number of letters)

pg. 150
10 Seconds

So, 5000 is wrong and it must be replaced 22.Next number in the number sequence
by (500*50) = 25000 is
22, 11, 33, 16.5, 49.5, ?
20. CGE, RNP, FJH, OKM, IMK, __ A) 24.75 B) 41
A) JHL B) HJL C) 33.5 D) 26.75
C) LHJ D) JKL Solution:
Solution: A) 24.75
C) LHJ Explanation:
Explanation: The given number series is 22, 11, 33,
16.5, 49.5, ?
Here it can be seen that it is not a
continuous series. 22 / 2 = 11
So, take alternative terms. 11 x 3 = 33
CGE, FJH, IMK and RNP, OKM,__ 33 / 2 = 16.5
Between R and O there is a gap of two 16.5 x 3 = 49.5
letters. so the first letter in the answer is L.
Proceeding like this the answer is LHJ 49.5 / 2 = 24.75
Hence, the next number series is 24.75.

21. Find the next number in the given


number sequence? 23. Choose the missing terms in the given
101, 98, 93, 86, 75, ? series from the four alternatives given
below.
A) 62 B) 68
C, E, G, J, M, Q, ?
C) 71 D) 73
A) T B) U
Solution:
C) P D) V
A) 62
Solution:
Explanation:
B) U
The given number sequence is 101, 98,
93, 86, 75? Explanation:

101 - 3 = 98 The letters move 2, 2, 3, 3, 4, 4, ... steps


forward.
98 - 5 = 93
93 - 7 = 86
24. Which digits will come in the place of
86 - 11 = 75 '?' in the following series?
75 - 13 = 62 372, 823, 644, 582, 46?, 8?7
Here the series follows a pattern that A) 4, 4 B) 6, 2
consecutive subtraction of prime numbers.
Hence, the next number in the sequence C) 3, 2 D) 2, 5
is 62. Solution:
B) 6, 2

pg. 151
10 Seconds

Explanation: Explanation:
Given series is 372, 823, 644, 582, 46?, This is an alternating series in alphabetical
8?7 order. The middle letters follow the order
ABCDE. The first and third letters are
Here the series follow the rule that : alphabetical beginning with J. The third
372 = 3+7+2 = 12 letter is repeated as a first letter in each
subsequent three-letter segment.
823 = 8+2+3 = 13
644 = 6+4+4 = 14
27. Find the next number in the series
582 = 5+8+2 = 15 given 18, 90, 450, 2250?
46? = 4+6+? = 16 => ? = 6 A) 4520 B) 6520
8?7 = 8+?+7 = 17 => ? = 2 C) 7250 D) 11250
So, the missing numbers are 6 and 2 Solution:
D) 11250
25. Find the next number in the given Explanation:
number series?
The given number series is 18, 90, 450,
10, 6, 8, 15, ? 2250, ?
A) 34 B) 24 18 x 5 = 90
C) 28 D) 30 90 x 5 = 450
Solution: 450 x 5 = 2250
A) 34 2250 x 5 = 11250
Explanation: Hence, the next number in the given
series is 11250.
The given number series is 10, 6, 8, 15, ?
10 x 0.5 + 1 = 6
28. Find the next number in the given
6x1+2=8
number sequence?
8 x 1.5 + 3 = 15
9, 19, 21, 43, 45, ...
15 x 2 + 4 = 34
A) 88 B) 54
Hence, the next number in the given
C) 91 D) 49
series is 34.
Solution:
C) 91
26. What should be replaced in the place
of '?' in the following series? Explanation:
JAK, KBL, LCM, MDN? The given number series 9, 19, 21, 43, 45,
... follows a pattern that
A) MEN B) NEM
Alternately (x 2 + 1) and (x 1 + 2)
C) NEO D) OEP
9 x 2 + 1 = 19
Solution:
19 x 1 + 2 = 21
C) NEO
21 x 2 + 1 = 43

pg. 152
10 Seconds

43 x 1 + 2 = 45 Solution:
45 x 2 + 1 = 91 A) Grass
Hence, the next number in the given Explanation:
sequence is 91.
Cup is used to drink something with the
help of lips. Similarly, birds collect grass
with the help of beak to make her nest.
29. Next number in the given series is
Hence, Cup : Lip :: Bird : Grass.
9 7 10 6 11 ?
A) 5 B) 4
2. Vaunt : Flaunt :: Disparate : ?
C) 13 D) 15
A) Similar B) Homogenous
Solution:
C) Contrast D) Alike
A) 5
Solution:
Explanation:
C) Contrast
The given number series is 9 7 10 6 11 ?
Explanation:
Here in this we have two number
sequences, they are Vaunt - Vaunt means to boast or to praise
about something excessively.
9, 10, 11, ... and 7, 6, 5, ...
Flaunt - Flaunt also means to boast about
Hence, the next number in the given something or to praise something.
number series is 6 - 1 = 5.
Flaunt is the synonym of the word Vaunt.
Disparate - Disparate means essentially
30. Choose the missing terms in the given different in its kind or unlike or not able to
series from the four alternatives given be compared.
below.
Then the required word is the synonym of
GMQ, JOT, MQW? Disparate.
A) PTZ B) ORY Here in the given options, Contrast is the
C) VXY D) PSZ only word which is similar in meaning of
Disparate.
Solution:
Hence, Vaunt : Flaunt :: Disparate :
D) PSZ Contrast.
Explanation:
The first letter moves three steps forward. 3. Biped : Quadruped :: Ostrich : ?
The second letter moves two steps
forward. The third letter moves three steps A) Cat B) Kangaroo
forward. C) Penguin D) Duck
Solution:
Analogy A) Cat
1. Cup : Lip :: Bird : ? Explanation:
A) Grass B) Forest
C) Beak D) Bush

pg. 153
10 Seconds

A biped is an animal that uses two legs for Vital means full of energy, lively
walking and the quadrupeds are animals
that use four legs for walking. So, they are antonyms.

From the Given options, Cat only have 4 So, for the 2nd part of the analogy, you
legs. are looking for a antonym to trite

So Biped : Quadruped :: Ostrich : Cat. Trite means lacking originality or


freshness. Overused. Kind of like a cliche
So, an antonym could be original, unique,
4. Paw : Cat :: Hoof : ? innovative.
A) Donkey B) Lion
C) Elephant D) Horse 7. Fumble is to Finesse as Malign is to
Solution: A) Slander B) Extol
D) Horse C) Criticize D) Assurance
Explanation: Solution:
The cat's foot is called a paw. The horse's B) Extol
foot is called a hoof.
Explanation:
Fumble means grope, mishandle, drop,
5. Grass : Erosion :: Dam : ? mistake, blunder, bungle.
A) Water B) Freeze Finesse means skill, grace, elegance,
assurance, diplomacy, discretion.
C) Current D) Rain
So, they are opposites.
Solution:
Similarly,
A) Water
Malign means slander or criticize.
Explanation:
Extol means being kind or good-natured.
Here Grass is to Erosion and whereas A person without finesse often fumbles. A
Dam is to Water. person without extol in his heart often
Since Grass prevents Erosion similarly maligns.
Dam prevents Water flow. Extol means praise or exalt, benevolence
Hence, Grass : Erosion :: Dam : Water.

8. Choose the Antonym of the given word


6. Lethargic : Vital :: Trite : ? from the following options 'Jettison'

A) Unique B) Slug A) Salvage B) Submerge

C) Lazy D) Skinny C) Repent D) Descend

Solution: Solution:

A) Unique A) Salvage

Explanation: Explanation:

Lethargic means "sluggish", kind of 'Jettison' means to discard and 'salvage'


inactive means to rescue something discarded.
Hence, 'salvage' is the word which is

pg. 154
10 Seconds

farthest in meaning to the given word Exhibit and Display are synonyms for each
'Jettison'. other. Similarly, Send and Emit are
synonyms for each other. Hence, Exhibit :
display :: send : Emit.
9. Choose the word which gives similar
meaning to the given word from the
following options. ‘Autopsy' 12) 6 : 18 :: 4 : ?
A) Diagnosis B) Rebuild A) 14 B) 6
C) Manufacture D) Post C) 16 D) 8
mortem
Solution:
Solution:
D) 8
D) Post mortem
Explanation:
Explanation:
Here the given number analogy follows
Autopsy is the medical term for two patterns that,
postmortem.
6 18 --> 6 x 3
4 12 --> 4 x 3
10. Melt : Liquid :: Freeze : ?
OR
A) Condense B) Solid
6 18 ----> 6x6/2 = 18
C) Ice D) Crystal
4 8 ----> 4x4/2 = 8
Solution:
As 12 is not available in the options, 8 is
B) Solid correct answer.
Explanation:
Here the relation in the given words is, 13. Complete the Analogy?
First is the process of formation of the
second. Grain : Stock :: Stick : ?

That is, when anything melts, it forms A) Bundle B) String


liquid. Similarly, when anything freezes it C) Collection D) Heap
forms solid. Hence, Melt : Liquid :: Freeze
: Solid. Solution:
A) Bundle
11. Exhibit : display :: send : ? Explanation:
A) Stamp B) Receive Here the relation between the given words
is that the second word is the collection of
C) Show D) Emit the first word.
Solution: As the collection of grains is called Stock,
D) Emit similarly, the collection of sticks is called
as Bundle.
Explanation:
Hence, Grain : Stock :: Stick : Bundle.
Here the given words represent
synonyms.
14. Seismography : Earthquake ::
Taseometer : ?

pg. 155
10 Seconds

A) Landslides B) Strains Solution:


C) Resistances D) Volcanoes C) Membrane
Solution: Explanation:
B) Strains Fence protects the Yard & Membrane
protects the Cell.
Explanation:
Fence surrounds the Yard and allows the
Seismography is an instrument to certain objects in & out of it whereas a
measure the intensity of an earthquake. Membrane surrounds the Cell & only
Similarly, taseometer is an instrument to allows specific nutrients inside of it.
measure strains. Hence, Yard is to Fence as Cell is to
Membrane.

15. Choose a similar word?


Kolkata, Mumbai, Mangalore 18. Fill in the blanks with suitable
prepositions from the given options.
A) Hyderabad B) Delhi
The dog was sitting _______ the table
C) Cochin D) Jaipur waiting _______ catch the ball that was
thrown ________ him _________ his
Solution:
master.
C) Cochin
A) below, for, to, by B) under,
Explanation: for, towards, by

All cities belong to port cities of India. C) up, to, towards, by D) under, to,
towards, by
Solution:
16. Transition : Change :: Immobility : ?
D) under, to, towards, by
A) Stillness B) Liveliness
Explanation:
C) Action D) Busyness
The dog was sitting under the table
Solution: waiting to catch the ball that was thrown
A) Stillness towards him by his master.

Explanation:
Given Transition is to Change, and we 19. Essential Part for the given word from
know that Transition and change are the following options.
Synonyms. Similarly, Immobility means Respiration
not moving and synonym of immobility are
motionlessness, stillness,... A) Oxygen B) Carbon monoxide

Hence, Transition is to change as C) Mouth D) Circulation


immobility is to stillness.
Solution:
A) Oxygen
17. Yard is to Fence as Cell is to
Explanation:
A) Mitochondria B) Cytoplasm
A person or animal must take in oxygen
C) Membrane D) Nucleus for respiration to occur. A mouth (choice a)
is not essential because breathing can

pg. 156
10 Seconds

occur through the nose. Choices b and d Modern is used to refer to something new
are clearly not essential and can be ruled whereas antiquated is used with respect to
out. something old.
In other words, both terms are the
opposite of each other.From the above
20. Choose the pair which shows the analysis, the missing word should be a
same relation of the given pair from the word that represents the opposite of weak.
following options? Strong is used to with great influence.
BRISTLE : BRUSH Therefore, the apt word is strong or any
synonym of strong like potent, powerful,
A) arm : leg B) key : piano keen,...
C) recline : chair D) stage : curtain Hence, Modern : Antiquated :: Weak :
Strong.
Solution:
B) key : piano
23. Rotate : Energy :: Stop : ?
Explanation:
A) Spin B) Break
A bristle is a part of a brush; a key is a
part of a piano. C) Inertia D) None
Solution:
21. The workers union decided to go on To rotate is to energy to stop is to inertia.
strike since the management was From the definition itself we know that
adamant and did not agree to their terms. inertia is a property by virtue of which a
The discussion had reached a /an--------. body continues to be in the state of rest or
in state of motion. So over here whenever
A) Climax B) impetus energy is supplied then the body starts to
C) impasse D) obstacle move that is a displacement is there and it
is rotating where as whenever the body is
Solution: stopped an inertia of rest is created.
C) impasse So, Rotate : Energy :: Stop : Inertia.
Explanation:
Impasse means, a situation in which no 24. Thread is to String as Cask is to
progress is possible, especially because
of disagreement; a deadlock. A) Barrel B) Disc

So it is Impasse. C) Wallet D) Box


Solution:

22. Modern : Antiquated :: Weak : ? A) Barrel

A) Strong B) Fragile Explanation:

C) Inadequate D) Puny Here Thread and String are Synonyms of


each other. Then the answer will be the
Solution: synonym of Cask.
A) Strong Cask is nothing but a large barrel-like
container used for storing liquids, typically
Explanation:
alcoholic drinks.
Ex : Drum, Vessel, Tank, Barrel,
Hogshead, Firkin,...

pg. 157
10 Seconds

Here from the options Barrel is the only Hence, Develop : Assess :: Train :
synonym of Cask. Analyze.
Thus, Thread : String :: Cask : Barrel
28. Bill : Law :: ? : Insect
25) 583 is related to 293 in the same way A) Pupa B) Stage
as 488 is related to____
C) Larva D) Bird
A) 291 B) 378
Solution:
C) 487 D) 581
C) Larva
Solution:
Explanation:
B) 378
A Bill becomes a Law; a Larva becomes
Explanation: an insect. (Or)
Sum of digits of the first number is 2 more A Bill turns into a law; a Larva turns into
than the sum of digits of the second an Insect.
number.

29. Fossil : Extinction :: Puddle : ?


26. Convent : Cloister :: Eyrie : ?
A) Wet B) Rain
A) Show B) Open
C) Lake D) Dry
C) Hide D) Nest
Solution:
Solution:
B) Rain
D) Nest
Explanation: Here the relation between a
Explanation: fossil and extinction is that a fossil remains
after an extinction. That is fossils are
Here the first two words are synonyms. evidence of an extinction.
Similarly, Eyrie means a large nest. So,
the synonym of Eyrie is nest.
Hence, Convent : Cloister :: Eyrie : Nest. Similarly, puddle remains after it is rained.
That is puddles are evidence of rain.
Therefore, Fossil is to Extinction as
27. Develop is to assess as train is to Puddle is to Rain.
A) Change B) Educate
C) Analyze D) Recruit 30. Grapes : Wine :: Squid : ?
Solution: A) Ocean B) Salad
C) Analyze C) Calamari D) Fishing
Explanation: Solution:
When you develop a product, you assess C) Calamari
its performance for proper functioning.
Explanation:
Similarly, when you train a person to do a
job, you analyze his/her performance to Here the given words follow a relation in
understand how good he/she is at it. such a way that, grapes are made into

pg. 158
10 Seconds

wine. Similarly, squid are made into 7 4 1


calamari.
7 4 6
Hence, Grapes : Wine :: Squid : Calamari
Only at these places 4 is preceded by 7
but not followed by 3.
Order and Ranking
In this chapter generally the ranks of a 2. In a row of boys, If A who is 10th from
person from both sides left or right or from the left and B who is 9th from the right
top and from bottom are mentioned and interchange their positions, A becomes
total numbers of persons are asked. In 15th from the left. How many boys are
other words a set, group or series of there in the row?
numerals is given and the candidates is
asked to trace out numerals following A) 23 B) 31
certain given conditions. Sometimes the C) 27 D) 28
questions are based on their interchanged
positions. Solution:
There are mainly three/four types of A) 23
questions -
Explanation:
1. Total persons and position of one
Clearly, A’s new position is 15th from the
person (either from left or right) are given.
left. But this is the same as B’s earlier
2. Position of more than one person is position which is 9th from the right.
given.
3. Over lapping conditions:
3. How many such pairs of digits are there
a. Overlapping case: When (left + right) in the number 421579368 each of which
positions > Total number of students has as many digits between them in the
number as when they are arranged in
b. Not overlapping case: When Total ascending order?
number of students > (left +right) positions
A) None B) One
4. Ascending/Descending Order according
age, height, marks etc. C) Two D) Three
Questions: Solution:
1. How many 4's are there preceded by 7 D) Three
but not followed by 3?
Explanation:
5 9 3 2 1 7 4 2 6 9 7 4 6 1 3 2 8
Given Number: 4 2 1 5 7 9 3 6 8
7 4 1 3 8 3 2 5 6 7 4 3 9 5 8 2 0
1 8 7 4 6 3 Ascending order: 1 2 3 4 5 6 7 8 9
A) Four B) Three Hence the required pairs are 12, 49, 16
C) Six D) Five
Solution: 4. Sam ranked 9th from the top and 38th
from the bottom in a class. How many
A) Four
students are there in the class?
Explanation:
A) 45 B) 47
7 4 2
C) 46 D) 48
7 4 6
Solution:

pg. 159
10 Seconds

C) 46 C) 3 D) 0
Explanation: Solution:
Number of students in class = (8 + 1 + 37) A) 1
= 46
Explanation:
Only one such symbol is there in the given
5. Nitin ranks 18th in a class of 49 arrangement.
students. What is rank from the last?
It is 4@5.
A) 31 B) 18
C) 32 D) 19
8. How many such digits are there in the
Solution: number 7346285 which are as far away
from the beginning of the number, as they
C) 32 will be when arranged in ascending order
Explanation: within the number?

Number students behind the nitin in rank = A) None B) one


(49 - 18) = 31 C) Two D) Three
Nitin is 32nd from the last. Solution:
C) Two
6. In a queue, Amrita is 10th from the front Explanation:
while Mukul is 25th from behind and
Mamata is just in the middle of the two. If Given Number: 7346285
there be 50 persons in the queue. What
position does Mamata occupy from the Ascending Number : 2345678
front? Answer is Two. Those digits are 3,4
A) 14th B) 16th
C) 18th D) 20th 9. In a row, Kumar is at 7th place from the
Solution: left and Pawan is at 9th place from the
right. When they interchange the positions
C) 18th Kumar becomes 11th from left. How many
were seated in the row?
Explanation:
A) 19 B) 20
Number of persons between Amrita and
Mukul = 50 - (10 + 25) = 15. Since C) 21 D) 27
Mamata lies in middle of these 15
persons, so Mamata`s position is 8th from Solution:
Amrita i.e. 18th from the front. A) 19
Explanation:
7. Read the arrangement carefully and Initially from left 7th = kumar
give the answer of following questions?
K$23DBE8HM4@5JF4%K1+WR#AA*415. from right 9th = pawan
How many such symbols are there which
Interchange kumar = 11th from left, from
is not immediately preceded by a letter but
right kumar = 9th;
immediately followed by a number?
11+8 = 19;
A) 1 B) 2

pg. 160
10 Seconds

8 is D) 34
1,2,3,4,5,6,7(pawan),8,9,10,11(kumar),12,
13,14,15,16,17,18,19; Explanation:

simply count the 9th place from right we Total number of students in the class = 28
will get total as 19. + 6 i.e, from top (7-1)
28 + 6 = 34.

10. What should come next in the


following number series? 13. If the third day of a month is Tuesday,
98765432187654321765432 which of the following would be the 4th
1 day before the 27th day of that month?

A) 9 B) 8 A) Tuesday B) Monday

C) 7 D) 6 C) Wednesday D) Sunday

Solution: Solution:

D) 6 B) Monday

Explanation: Explanation:

Here the series is: 987654321, 87654321, Dates on which Tuesday fall are 3,10,17,
7654321, 65... and 24
Hence, the 27th day of the month will be
(Tuesday + 3 days=Friday)
11. How many combinations of two-digit
numbers having 8 can be made from the Required day of the month = Friday - 4 =
following numbers? Monday

8, 5, 2, 1, 7, 6
A) 10 B) 6 14. 382 473 568 728 847 629. If 382 is
written as 238, 473 as 347 and so on, then
C) 9 D) 11 which of the following two numbers will
have least difference between them?
Solution:
A) 473 & 382 B) 629 & 728
D) 11
C) 629 & 847 D) 728 & 847
Explanation:
Solution:
The possible two-digit numbers are: 88,
85, 82, 81, 87, 86, 58, 28, 18, 78, 68 D) 728 & 847
These are 11 in number Explanation:
The given numbers are :
12. Sanjeev ranks seventh from the top 382 473 568 728 847 629
and twenty eighth from the bottom in a
class. How many students are there in the When changes are made as given in the
class? question, numbers become as follows :

A) 33 B) 36 238 347 856 872 784 962

C) 35 D) 34 Again now, find out the difference between


the numbers given in options as given
Solution: below:

pg. 161
10 Seconds

(1) 473 - 382 => 347 - 238 = 109 Solution:


(2) 629 - 728 => 962 - 872 = 90 A) 37
(3) 629 - 847 => 962 - 784 = 178 Explanation:
(4) 728 - 847 => 872 - 784 = 88 Number of boys in the class = (18 + 1
+18) = 37

15. Rajan is sixth from the left end and


vinay is tenth from the right end in a row of 18. Some boys are sitting in a row. P is
boys. If there are eight boys between sitting fourteenth from the left and Q is
Rajan and Vinay, how many boys are seventh from the right. If there are four
there in the row? boys between P and Q, how many boys
are there in the row?
A) 24 B) 26
A) 26 B) 24
C) 23 D) 25
C) 25 D) 22
Solution:
Solution:
A) 24
C) 25
Explanation:
Explanation:
Number of boys in the row = (6 + 10 + 8) =
24 Number of boys in the row = number of
boys until P + number of boys between P
and Q + number of boys including Q and
16. 517 325 639 841 792. What will be those behind Q = 14 + 4 + 7 = 25.
the first digit of the second highest number
after the positions of only the 2nd, 3rd
digits within each number are 19. If the positions of the first and the third
interchanged? digit within each number are interchanged,
which of the following will be the third digit
A) 8 B) 2 of the second lowest number? 987, 514,
C) 7 D) 9 658, 487, 404, 269

Solution: A) 8 B) 9

C) 7 C) 2 D) 4

Explanation: Solution:

The new numbers formed are 571, 352, D) 4


693, 814, 729 Explanation:
The second highest number is 729, its first According to the question, after the
digit is 7 position of the first and third digit
interchanged new numbers are

17. A class of boys stands in a single line, 789, 415, 856, 784, 404, 962
one boy is 19th in order from both the 962, 856, 789, 784, 404, 415
ends. How many boys are there in the
class? Here the second lowest number is 404
and the last digit of it is '4'.
A) 37 B) 39
C) 27 D) 38

pg. 162
10 Seconds

20. In a class of 180, where girls are twice So, 6th, 13th, 20th, 27th February were all
the number of boys, Rupesh [a boy] Thursdays.
ranked 34th from the top. If there are 18
girls ahead of Rupesh, how many boys Thus 2nd march, 2003 was 3 days after
are after him in rank? Thursday, i.e Sunday

A) 45 B) 44
C) 60 D) can't be determined 23. How many even numbers are there in
the following sequence of numbers which
Solution: are immediately followed by an odd
number as well as immediately preceded
B) 44 by an even number?
Explanation: 85867689327534223552281
No. of boys up to the 34th rank = 34-18 = 1931751
16 A) 2 B) 3
Total no of boys = 180x1/2+1 = 60 C) 4 D) 5
Number of boys after the rank of Rupesh Solution:
= 60 - 16 = 44
C) 4
Explanation:
21. If the day before yesterday was
Saturday, what day will fall on the day We have to find out even number - even
after tomorrow? number - odd number sequence.
A) Friday B) Thursday 8586768932753422355228
11931751
C) Wednesday D) Tuesday
There are four such such even numbers:
Solution: 6,8,2 and 8.
C) Wednesday
Explanation: 24. In a class of 50 students M is eighth
If day before yesterday was Saturday, so from top. H is 20th from bottom. How
today is Monday. Thus, tomorrow will be many students are there between M and
Tuesday and day after tomarrow will be H?
Wednesday. A) 22 B) 23
C) 24 D) can't be
22. If the 30th January 2003 was determined
Thursday, what was the day on 2nd Solution:
march, 2003?
A) 22
A) Sunday B) Thursday
Explanation:
C) Tuesday D) Saturday
M is 8th from the top.
Solution:
H is (50 - 20 + 1 =) 31 st from the top.
A) Sunday
Number of students between M and H =
Explanation: (31 - 8- 1=)22
30th January, 2003 was Thursday.

pg. 163
10 Seconds

25. The positions of the first and sixth C) 2 D) 7


digits in the number 5109238674 are
interchanged. Similarly, the positions of Solution:
the second and the seventh digits are A) 3
interchanged and so on. which of the
following will be the third digit from the Explanation:
right end after the after the
5th to the right of 12th digit from the right
rearrangement?
end means (12 - 5 = ) 7th digit from the
A) 9 B) 0 right.ie 3.
C) 6 D) 3
Solution: 28. If Tuesday falls on 4th of month, then
which day will fall three days after the 24th
B) 0 ?
Explanation: A) Monday B) Tuesday
There are 10 digits. Now, third digit from C) Friday D) Thursday
the right means (10-3+1 = ) 8th digit from
the left. But, from the question, third and Solution:
eighth digits are interchanged. Hence, the
D) Thursday
required digit is the third digit of the given
number, ie 0.

Explanation:
26. In a row of thirty boys, R is fourth from The 4th day is Tuesday, then 11, 18, 25th
the right end and W is tenth from the left also Tuesdays
end. How many boys are there between R
and W? Then 3 days after 24th is 27th, which is
Thursday.
A) 15 B) 16
C) 17 D) can't be determined
29. If the day before yesterday was
Solution: Thursday, when will Sunday be?
B) 16 A) tomorrow B) today
Explanation: C) day after tomorrow D) none
W = 10th from the left Solution:
R = 4th from the right = (30 - 4 + 1= ) 27th A) tomorrow
from left
Explanation:
No of boys between R and W = (27 - 10) -
1 = 16 If day before yesterday was Thursday,
then today is saturday.
Tomorrow will be Sunday.
27. Which of the following is fifth to the
right of the twelth digit from the right end
of the below arrangement? 30. If it is possible to form a number which
18594712583659276452926 is a perfect square of a two digit odd
4123514283 number using the second, fourth, seventh
digits of the number 739142658 using
A) 3 B) 1 each only once, which of the following is

pg. 164
10 Seconds

the second digit of that two-digit odd 1. Job repairing and painting of office
number? building require funds
A) 4 B) 7 2. A has necessary competence to
undertake such study
C) 3 D) none
A) Only 1 is true
Solution:
B) Only 2 is true
D) none
C) Either 1 or 2 is true
Explanation:
D) Both 1 and 2 follow
The 2nd, 4th, 7th digits are 3, 1, 6
respectively. Solution:
The perfect square of a two-digit odd D) Both 1 and 2 follow
number, formed using these digits, is 361
Explanation:
and 361 = 19^2.
Clearly A wishes to study the degree of
effect of pay revision on job satisfaction of
employees. This means that job
Deductive Reasoning satisfaction can be measured and A is
capable of making such a study. So, both
1. "If it does not rain throughout this
are not implicit.
month, most farmers would be in trouble
this year"
Assumptions: 3. If he is intelligent, he will pass the
examination
1. Timely rain is essential for farming
Assumptions:
2. Most farmers are generally dependent
on rains 1. To Pass, he must be intelligent
A) Only 1 is true 2. He will pass the examination
B) Only 2 is true A) Only 1 is true
C) Either 1 or 2 is true B) Only 2 is true
D) Both 1 and 2 follow C) Either 1 or 2 is true
Solution: D) Neither 1 nor 2 is true
D) Both 1 and 2 follow Solution:
Explanation: A) Only 1 is true
It is mentioned that farmers will be in Explanation:
trouble without rain. This means that
timely rain is essential. Also, it shows that The statement mentions that he will pass if
farmers are dependent on rain. So, both 1 he is intelligent. So,1 is implicit. Further,
and 2 are true. this means that it is not necessary that he
will pass. So, 2 is not implicit

2. "I would like to study the impact of pay


revision on job satisfaction of employees." 4. If you are an engineer, we have a
- A tells B challenging job for you

Assumptions: Assumptions:

pg. 165
10 Seconds

1. We need an engineer 6. The 'M' Cooperative housing society


has put up a notice at its gate that sales
2. You are an engineer persons are not allowed inside the society
A) Only 1 is true Assumptions:
B) Only 2 is true 1. All the sales persons stay away from 'M'
C) Either 1 or 2 is true Cooperation society

D) Neither 1 or 2 is true 2. The security guard posted at the gate


may be able to stop the sales persons
Solution: entering the society
A) Only 1 is true A) Only 1 is true
Explanation: B) Only 2 is true
Clearly job is offered to an engineer. This C) Either 1 or 2 is true
means that he is needed. So,1 is true. The
word "If" in the statement 2 makes it false D) Both 1 and 2 follow
Solution:

5. Banks should always check financial D) Both 1 and 2 follow


status before lending money to a client. Explanation:
Assumptions : Since both the assumptions follow from
I. Checking before lending would give a the given statement, so both 1 and 2 are
true picture of the client's financial status. implicit

II. Clients sometimes may not present the


correct picture of their ability to repay loan 7. Retired persons should not be
amount to the bank. appointed for the posts in organisations
A) If only assumption I is Implicit Assumptions:
B) If only assumption II is Implicit 1. Retried persons may lack zeal and
C) If either I or II is Implicit commitment to carry out executive's work.

D) If both I and II are Implicit 2. Retried persons do not take interest in


the work and welfare of new
Solution: organisations.
A) If only assumption I is Implicit A) Only 1 is true
Explanation: B) Only 2 is true
I is implicit in the norm prescribed in the C) Either 1 or 2 is true
sentence. This is why checking is being
advised. Again, what would the banks D) Neither 1 or 2 is true
check? Obviously what the clients reveal. Solution:
Banks would assume the revelation to be
true. Hence II is not implicit. However, II is D) Neither 1 or 2 is true
vague. If cross - checking is what the
speaker has in mind, II would become Explanation:
implicit. Since both 1 and 2 does not follow from
the statement, so neither of them is
implicit

pg. 166
10 Seconds

8. The office building needs repairing just b is also not implicit because in the
as urgently as it needs internal as well as sentence 'only' indicates false meaning.
external painting
Assumptions:
10. The Highway police authority put up
1. Efficiency of people working in the office large boards at regular intervals indicating
cannot be improved unless office building the speed limit and dangers of over-
is repaired. speeding on the highways.
2. Repairing and painting of office building Assumptions:
require funds
I. Most of the motorists may drive their
A) Only 1 is true vehicles within the speed limit on the
highways.
B) Only 2 is true
II. Motorists generally ignore such
C) Either 1 or 2 is true cautions and over-speed on the highways.
D) Neither 1 nor 2 is true A) If only assumption I is Implicit
Solution: B) If only assumption II is Implicit
D) Neither 1 nor 2 is true C) If either I or II is Implicit
Explanation: D) If both I and II are Implicit
Nothing can be deduced regarding the Solution:
effect of repairs of office building on
efficiency of workers, or the requirement of A) If only assumption I is Implicit
funds for repairs, from the given
statement. So, neither 1 nor 2 is true Explanation:
The boards have been put with the
assumption that they may have a positive
9. Statement : Let there be a signboard impact.
also indicating the directions and
instructions.
Assumptions : 11. The Chairman of the company urged
all the employees to refrain from making
a) Signboard can be prepared without long personal calls during working hours in
using any language. order to boost productivity.
b) Signboard is the only effective tool to Assumptions :
indicate directions.
I. Majority of the employees may respond
A) Both a and b are not sufficient positively to the Chairman's appeal
B) Only a is sufficient II. Most of the employees may continue to
make long personal calls during working
C) Only b is sufficient hours.
D) Both a and b are sufficient A) If only assumption I is Implicit
Solution: B) If only assumption II is Implicit
A) Both a and b are not sufficient C) If either I or II is Implicit
Explanation: D) If both I and II are Implicit
The statement doesn't give any clue to the Solution:
use of language. Hence a is not implicit.

pg. 167
10 Seconds

A) If only assumption I is Implicit B) If only assumption II is Implicit


Explanation: C) If either I or II is Implicit
I is implicit: when urge someone to do D) If both I and II are Implicit
something, you assume a positive
response. For the same reason, II is not Solution:
implicit. B) If only assumption II is Implicit
Explanation:
12. If the city bus which runs between I is not implicit because health is not the
Ramanagar and Sant colony is extended focus of the statement. II is implicit in the
to vasant, vihar,it will be convenient . very need for substitution.
Appeal of residents of ramanagar to city
bus company
Assumptions: 14. In Bombay, railway trains are
indispensable for people in the subrubs to
1. The convenience of city bus company is reach their places of work on time
much more important than the needs of
consumers Assumptions:
2. The city bus company is indifferent to 1. Railway trains are the only mode of
the aspirations of the residents of Sant transport available in the subrubs of
colony Bombay
A) Only 1 is true 2. Only railway trains run punctually
B) Only 2 is true A) Only 1 is true
C) Neither 1 nor 2 is true B) Only 2 is true
D) Both 1 and 2 follow C) Either 1 or 2 is true
Solution: D) Neither 1 nor 2 is true
C) Neither 1 nor 2 is true Solution:
Explanation: B) Only 2 is true
Clearly the city bus company is intended Explanation:
to provide bus services according to the
The railway trains are indispensable for
needs as per local residents and not as
people to reach the place on time does not
per their own convenience, So 1 is not
mean that there are no other means of
true. In the same way 2 is not true
transport but shows that trains alone run
on time. So, 1 is not true.
13. If farmers want to improve their yield,
they must use organic fertilizers in place of
15. The Civic administration has asked the
chemical fertilizers.
residents of the dilapidated buildings to
Assumptions: move out as these buildings will be
demolished within the next thirty days
I. Chemical fertilizers have certain ill
effects on health Assumptions :
II. Chemical fertilizers do not produce as I. The Civic administration may be able to
much yield as the organic fertilizers. demolish these buildings as per schedule.
A) If only assumption I is Implicit

pg. 168
10 Seconds

II. The residents of these buildings may Time taken by Sriram to complete the wok
vacate and stay elsewhere = 30 days
A) If only assumption I is Implicit So, from both either by statement I or
statement II we can find the time to
B) If only assumption II is Implicit complete the work.
C) If either I or II is Implicit
D) If both I and II are Implicit 17. Statement: Should India encourage
Solution: exports, when most things are insufficient
for internal use itself?
D) If both I and II are Implicit
Arguments:
Explanation:
Yes. We have to earn foreign exchange to
I is implicit in the deadline given. II is pay for our imports.
implicit in the notice given to the residents
to move out. No. Even selective encouragement would
lead to shortages.
A. Only argument I is strong
16. Manish completes a work in 20 days.
After 8 days Sriram joined the work. In B. Only argument II is strong
how many days will the work be C. Either I or II is strong
completed?
D. Neither I nor II is strong
Statements :
E. Both I and II are strong
I. Sriram takes twice the time than Manish
to complete the work. Answer: Option A
II. Sriram completes half the work in 15 Explanation:
days.
Clearly, India can export only the surplus
A) Statement I alone is sufficient to and that which can be saved after fulfilling
answer the question its own needs, to pay for its imports.
Encouragement to export cannot lead to
B) Statement II alone is sufficient to shortages as it shall provide the resources
answer the question for imports. So, only argument I holds.
C) Either the statements I & II is
sufficient to answer the question
18. Statement: Should all the drugs
D) Neither the statements I & II is patented and manufactured in Western
sufficient to answer the question countries be first tried out on sample basis
Solution: before giving licence for sale to general
public in India?
C) Either the statements I & II is sufficient
to answer the question Arguments:

Explanation: Yes. Many such drugs require different


doses and duration for Indian population
From statement I, and hence it is necessary.
Time taken by Sriram to complete the No. This is just not feasible and hence
work = 40 days cannot be implemented.
From statement II, A. Only argument I is strong
B. Only argument II is strong

pg. 169
10 Seconds

C. Either I or II is strong A. Only argument I is strong


D. Neither I nor II is strong B. Only argument II is strong
E. Both I and II are strong C. Either I or II is strong
Answer: Option A D. Neither I nor II is strong
Explanation: E. Both I and II are strong
Clearly, health of the citizens is an issue of Answer: Option E
major concern for the Government. So, a
product like drugs, must be first studied Explanation:
and tested in the Indian context before The students union formation shall be a
giving licence for its sale. So, only step towards giving to students the basic
argument I holds strong. education in the field of politics. However,
it shall create the same political
atmosphere in the campus. Thus, both the
19. Statement: Should India make efforts arguments hold strong.
to harness solar energy to fulfil its energy
requirements?
Arguments: 21. Statement: Should India give away
Kashmir to Pakistan?
Yes, Most of the energy sources used at
present is exhaustible. Arguments:

No. Harnessing solar energy requires a lot No. Kashmir is a beautiful state. It earns a
of capital, which India lacks in. lot of foreign exchange for India.

A. Only argument I is strong Yes. This would help settle conflicts.

B. Only argument II is strong A. Only argument I is strong

C. Either I or II is stron B. Only argument II is strong

D. Neither I nor II is strong C. Either I or II is strong

E. Both I and II are strong D. Neither I nor II is strong

Answer: Option A E. Both I and II are strong

Explanation: Answer: Option A

Clearly, harnessing solar energy will be Explanation:


helpful as it is an inexhaustible resource Clearly, India cannot part with a state that
unlike other resources. So, argument I is a major foreign exchange earner to it.
holds. But argument II is vague as solar So, argument I holds strong. Further,
energy is the cheapest form of energy. giving away a piece of land unconditionally
and unreasonably is no solution to settle
disputes. So, argument II is vague.
20. Statement: Should there be students
union in college/university?
Arguments: 22. Statement: Should cottage industries
be encouraged in rural areas?
No. This will create a political atmosphere
in the campus. Arguments:

Yes, it is very necessary Students are Yes. Rural people are creative.
future political leaders.

pg. 170
10 Seconds

Yes. This would help to solve the problem Arguments:


of unemployment to some extent.
Yes. The young students should not be
A. Only argument I is strong burdened with such examinations which
hampers their natural growth.
B. Only argument II is strong
No. The students will not study seriously
C. Either I or II is strong as they will get automatic promotion to the
D. Neither I nor II is strong next class and this will affect them in
future.
E. Both I and II are strong
A. Only argument I is strong
Answer: Option B
B. Only argument II is strong
Explanation:
C. Either I or II is strong
Clearly, cottage industries need to be
promoted to create more job opportunities D. Neither I nor II is strong
for rural people in the villages themselves. E. Both I and II are strong
The reason that rural people are creative
is vague. So, only argument II holds Answer: Option E
Explanation:
23. Statement: Should young Clearly, neither the students can be
entrepreneurs be encouraged? burdened with studies at such a tender
age, nor can they be left free to take
Arguments: studies casually, as this shall weaken their
Yes. They will help in industrial basic foundation. So, both the arguments
development of the country. follow.

Yes. They will reduce the burden on


employment market. 25. Statement: Should Indian scientists
A. Only argument I is strong working abroad be called back to India?

B. Only argument II is strong Arguments:

C. Either I or II is strong Yes. They must serve the motherland first


and forget about discoveries, honours,
D. Neither I nor II is strong facilities and all.
E. Both I and II are strong No. We have enough talent; let them stay
where they want.
Answer: Option E
A. Only argument I is strong
Explanation:
B. Only argument II is strong
Clearly, encouraging the young
entrepreneurs will open up the field for the C. Either I or II is strong
establishment of new industries. Thus, it
shall help in industrial development and D. Neither I nor II is strong
not only employ the entrepreneurs but E. Both I and II are strong
create more job opportunities for others as
well. So, both the arguments hold strong. Answer: Option D
Explanation:
24. Statement: Should all the annual Clearly, every person must be free to work
examinations up to Std. V be abolished? wherever he wants and no compulsion
should be made to confine one to one's

pg. 171
10 Seconds

own country. So, argument I is vague. B. Only argument II is strong


However, talented scientists can be of
great benefit to the nation and some C. Either I or II is strong
alternatives as special incentives or better D. Neither I nor II is strong
prospects may be made available to them
to retain them within their motherland. So, E. Both I and II are strong
argument II also does not hold.
Answer: Option D
Explanation:
26. Statement: Should we scrap the
The age of a person is no criterion for
system of formal education beyond
judging his mental capabilities and
graduation?
administrative qualities. So, none of the
Arguments: arguments holds strong.
Yes. It will mean taking employment at an
early date.
28. Statement: Should new big industries
No. It will mean lack of depth of be started in Mumbai?
knowledge.
Arguments:
A. Only argument I is strong
Yes. It will create job opportunities.
B. Only argument II is strong
No. It will further add to the pollution of the
C. Either I or II is strong city.
D. Neither I nor II is strong A. Only argument I is strong
E. Both I and II are strong B. Only argument II is strong
Answer: Option B C. Either I or II is strong
Explanation: D. Neither I nor II is strong
Clearly, argument I is vague because at E. Both I and II are strong
present too, many fields are open to all
Answer: Option C
after graduation. However, eliminating the
post-graduate courses would abolish Explanation:
higher and specialized studies which lead
to understanding things better and deeply. Opening up of new industries is
So, argument II is valid. advantageous in opening more
employment avenues, and
disadvantageous in that it adds to the
pollution. So, either of the arguments
27. Statement: Should there be an upper
holds strong.
age limit of 65 years for contesting
Parliamentary/ Legislative Assembly
elections?
29. Statement: Should high chimneys be
Arguments: installed in industries?
Yes. Generally, people above the age of Arguments:
65 lose their dynamism and will power.
Yes. It reduces pollution at ground level.
No. The life span is so increased that
people remain physically and mentally No. It increases pollution in upper
active even up to the age of 80. atmosphere.

A. Only argument I is strong A. Only argument I is strong

pg. 172
10 Seconds

B. Only argument II is strong Time and Work


C. Either I or II is strong
Concept
D. Neither I nor II is strong
Let us assume a well has to be dug which
E. Both I and II are strong has to be done in 10 days. If a person can
Answer: Option A dig a well in 10 days, then in 1 day he will
dig 1/10th of that well. This basic
Explanation: approach can be applied to solve a
majority of the time and work problems.
Pollution at ground level is the most
hazardous in the way of being injurious to
LCM Approach
human and animal life. So, argument I
alone holds.
The same concept can be learned with
unit’s work approach as well, which
assumes the total work to be done as the
30. Statement: Does India need so many LCM (Learn how to calculate LCM) of the
plans for development? number of days taken by each of the
Arguments: persons to complete the work. Let's
assume that Trump can do a piece of work
Yes. Nothing can be achieved without in 20 days working alone and Putin can do
proper planning. it in 30 days of his own. Now in the above-
mentioned case, let us assume that the
No. Too much time, money and energy is
work consists of the LCM of 20 & 30 i.e.
wasted on planning.
60 units to be done by Trump & Putin.
A. Only argument I is strong Since Trump completes 60 units in 20
days, so he completes 60/20 = 3 units of
B. Only argument II is strong work per day. Similarly Putin completes 60
C. Either I or II is strong units of the work in 30 days, so he
completes 60/30 = 2 units per day. They
D. Neither I nor II is strong are doing the same work together, so they
do 3 + 2 = 5 units per day. So, 60 units will
E. Both I and II are strong be done in 60/5 = 12 days.
Answer: Option A
Questions:
Explanation:
Before indulging in new development 1. There was a competition in an
program it is much necessary to plan the engineering college on fast completion of
exact target, policies and their the project work among two groups A and
implementation and the allocation of funds B. The group A contains 9 students and
which shows the right direction to work. group B contains 6 students. If the group
So, argument I holds strong. Also, A students can complete the work in 88
planning ensures full utilization of days by working 6 hours a day then group
available resources and funds and B students can complete the same work in
stepwise approach towards the target. So, how many days if they work 8 hours per
spending a part of money on it is no day?
wastage. Thus, argument II is not valid.
a)55 b)66 c)99
d)87

Solution: We can solve this problem by


using the formula:

pg. 173
10 Seconds

𝑀1 ×𝐷1×𝑇1 𝑀2 ×𝐷2×𝑇2 a)28 b)32


=
𝑊1 𝑊2

c)34 d)36

Solution:
M1= 9 D1= 88 T1= 6 W1=W
1
M2= 6 D2 = ? T2=8 W2= W capacity of 5 men in one day =
84

1
capacity of 8 girls in one day = 84
9×88×6 6×𝐷2×8
𝑊
= 𝑊
If 5M=8G then 10M=16G
D2=99 days
We need to find the capacity of 10men
2. The capacity of 1 man is equal to 3 and 5 girls = 10M+5G = 16M+5G = 21G
2
women. 34 men planted 5th of total plants
1
in 8 days working 9 hours a day. How capacity of 8 girls one day = 84
many more women required to plant the
remaining plants in 6 days working 9 1
capacity of 1 girl in one day =
hours per day? 84×8

a.68 b.204 c.34 d. 102 capacity of 21 girls to do work in one day


21 1
= =
84×8 32
Solution: We can solve this problem by
using the formula: Therefore they require 32 days to
complete the work.
𝑀1 ×𝐷1×𝑇1 𝑀2 ×𝐷2×𝑇2
𝑊1
= 𝑊2

M1= 34 D1= 8 T1= 9


2 4. If 8 boys and 6 men can build a wall in
W1=5w
10 days while 48 boys and 26 men can
build the same wall in 2 days, then what is
M2= ? D2 = 6 T2= 9 the time taken by 20 boys and 15 men to
3
W2= 5 build the same wall?

a)8 b)4 c)6


d)10
34×8×9 𝑀2×6×9
2 = 3 Solution:
𝑤 𝑤
5 5

capacity of 8 boys and 6 men to do work


M2=68 1
in one day = 10
More men required = 68-34 =34
capacity of 48 boys and 26 men to do
1
Therefore, number of women work in one day = 2
required=34× 3 = 102
1
8B+6M = 10 →1

1
3. How long would 10 men and 5 girls take 48B+26M = 2 →2
to complete the work if 5 men or 8 girls
can complete the same work in 84 days?

pg. 174
10 Seconds

1 Capacity of Anita =1 means, in one day


Solving 1 and 2, we get 16M= which
10
1 Anita can complete 1 unit of work.
implies 1M = 160 →3
Capacity of Vanitha =2 means, in one day
1
Substitute 3 in 1 then, B= Vanitha can complete 2 units of work.
128

15 20 1 If both Anita and Vanitha are working


We need 15M+20B= 160 +128 = 4 together then they can complete 3 units of
work in one day.
1
If capacity is 4 then, they require 4 days
to complete the work. Therefore, to complete total 18 units of
18
work Anita and Vanitha need = 3 = 6 days

5. Anita can complete a work in 18 days.


Vanitha can complete the same work in NOTE:
half the time taken by Anita. Anita and
Vinutha can together complete the work in 1. Use LCM method when individual
how many days? time data is given for at least 2
persons.
a)5 b)6 c)8 2. Use fraction method when
d)10 individual time data is not given for
at least 2 persons.
Solution:

Fraction method
6. Disha can complete a story book in 20
capacity of Anita to do work in one day = days. Madan can complete the same book
1 in 30 days. Disha and Madan can together
18 complete the book in how many days?
capacity of Vanitha to do work in one day a)6 b)12
1
=
9
c)18 d)25
Anita and Vanitha together can complete
1 1 1 Solution:
the work = 18 + 9 = 6

Disha→ 20 Madan→ 30
Therefore they require 6 days to complete
the work
Total work = LCM = 60
LCM method: 𝑡𝑜𝑡𝑎𝑙 𝑤𝑜𝑟𝑘
Capacity of Disha = 𝑡𝑖𝑚𝑒 𝑔𝑖𝑣𝑒𝑛 𝑓𝑜𝑟 𝐷𝑖𝑠ℎ𝑎
Anita→ 18 Vanitha→ 9 60
=20

Total Work = LCM of 18 and 9 = 18 𝑡𝑜𝑡𝑎𝑙 𝑤𝑜𝑟𝑘


Capacity of Madan = 𝑡𝑖𝑚𝑒 𝑔𝑖𝑣𝑒𝑛 𝑓𝑜𝑟 𝑀𝑎𝑑𝑎𝑛
Capacity of Anita in one day = 60
𝑡𝑜𝑡𝑎𝑙 𝑤𝑜𝑟𝑘 18
=30 = 2
𝑡𝑖𝑚𝑒 𝑔𝑖𝑣𝑒𝑛 𝑓𝑜𝑟 𝐴𝑛𝑖𝑡𝑎
=18 =1
Capacity of Disha =3 means, in one day
Capacity of Vanitha in one day = Disha can complete 3 units of work.
𝑡𝑜𝑡𝑎𝑙 𝑤𝑜𝑟𝑘 18
𝑡𝑖𝑚𝑒 𝑔𝑖𝑣𝑒𝑛 𝑓𝑜𝑟 𝑣𝑎𝑛𝑖𝑡ℎ𝑎
= 9 =2

pg. 175
10 Seconds

Capacity of Madan =2 means, in one day Solution:


Madan can complete 2 units of work.
Always take least efficient person`s
If both Disha and Madan are working capacity=1
together then they can complete 5 units of
work in one day. Shalini is the least efficient person, So
Shalini`s capacity =1
So, to complete total 60 units of work
60
Disha and Madan need= 5 =12days. Chaitra`s capacity=2 and Pushpa`s
capacity= 3

Shalini+ Chaitra +Pushpa = 1+2+3 = 6


7. Puneeth and Amit are deployed for a
task. Puneeth is twice as good as Shalini, Chaitra and Pushpa are working
workman as Amit. Puneeth and Amit for 2 days so, total work completed=6×
together can complete work in 18 days. 2=12
Then Puneeth alone can do in?
12
Therefore, Chaitra requires = 2
= 6 days.
a)27 b)37

c)18 d)54
9. Liam and James can complete a work
Solution: in 15 days and 10 days respectively. They
started doing the work together but after 2
Always take least efficient person`s days James had to leave and Liam alone
capacity = 1 completed the remaining work. The whole
work was completed in how many days?
Amit is the least efficient person, So
Amit`s capacity = 1 and Puneeth`s a.12 b.14
capacity=2
c.18 d. 20
Puneeth + Amit = 1+2 = 3
Solution:
Puneeth and Amit are working for 18 days
so, total work= 3× 18=54 Liam→ 15 James→ 10
54
Therefore, Puneeth requires = = Total work = LCM = 30
2
27days.
𝑡𝑜𝑡𝑎𝑙 𝑤𝑜𝑟𝑘
Capacity of Liam = 𝑡𝑖𝑚𝑒 𝑔𝑖𝑣𝑒𝑛 𝑓𝑜𝑟 𝐿𝑖𝑎𝑚
30
=15 =2
8.Shalini takes twice as much time as
𝑡𝑜𝑡𝑎𝑙 𝑤𝑜𝑟𝑘
Chaitra and thrice as much time as Capacity of James = 𝑡𝑖𝑚𝑒 𝑔𝑖𝑣𝑒𝑛 𝑓𝑜𝑟 𝐽𝑎𝑚𝑒𝑠
Pushpa to finish a piece of work. Working 30
together, they can finish the work in 2 =10 =3
days. Chaitra if working alone can do the
same work in how many days? Capacity of Liam =2 means, in one day
Liam can complete 2 units of work.
a)6 days b)5 days
Capacity of James =3 means, in one day
c)4 days d)7 days James can complete 3 units of work.

pg. 176
10 Seconds

If both Liam and James are working So, for 3 days they can complete 3 × 3 = 9
together then they can complete 5 units of units of work, and 11 units of work is left.
work in one day.
After 2 days Lohith left the work,
So, for 2 days they can complete 5× 2 = remaining work is completed byHemanth
10 units of work and 20 units of work is 11
= 1 = 11 days
left.
Therefore Hemanth worked for 11 days
After 2 days James left the work,
alone.
remaining work is completed by Liam in
20
= 2 = 10 days

So total number of days required = 11. Rohith hired Sunil and Anil to do a
2+10=12 days. piece of work for Rs 600. Sunil alone can
do it in 8 days and Anil alone can do it in 6
days. With the help of Swapnil they
completed the work in 3 days. Then what
10. Lohith can do a work in 10 days while is the share of Anil in the amount?
Hemanth can do the same work in 20
days. They started work together. After 3 a)300 b)225
days Lohith left the work and Hemanth
completed it. For how many days c)75 d) None
Hemanth worked alone?
Solution:
a)10 b)11
Total work = LCM of 8 and 6 = 24
c)20 d)13
Work done by Sunil in one day = 24/8 = 3
Solution:
Work done by Anil in one day = 24/6 = 4
Lohith → 10 Hemanth→ 20
Let Work done by Swapnil in one day = x
Total work = LCM = 20
Work done by Anil in 3 days = 12
𝑡𝑜𝑡𝑎𝑙 𝑤𝑜𝑟𝑘
Capacity of Lohith = 𝑡𝑖𝑚𝑒 𝑔𝑖𝑣𝑒𝑛 𝑓𝑜𝑟 Lohith
20 Since Anil is doing half of the total work,
= =2 his share of money is Rs 300.
10

𝑡𝑜𝑡𝑎𝑙 𝑤𝑜𝑟𝑘
Capacity of Hemanth =
𝑡𝑖𝑚𝑒 𝑔𝑖𝑣𝑒𝑛 𝑓𝑜𝑟 𝐽𝑎𝑚𝑒𝑠
20
=20 =1 12. Sunita is 3 times as efficient as
Shwetha. Working alone Shwetha can
Capacity of Lohith=2 means, in one day complete the work in 10 days. They
Lohith can complete 2 units of work. agreed to do a work for Rs 1000. They
started doing work together. After how
Capacity of Hemanth =1 means, in one many days sunita should leave the work
day Hemanth can complete 1 unit of work. so that shwetha gets Rs 400 share.

If both Lohith and Hemanth are working a)2 days b)3 days
together then they can complete 3 units of
work in one day. c)4 days d)1 days

pg. 177
10 Seconds

Solution: c)16 d)20


Solution:
Sunita’s one day work = 3 and Shwetha’s
one day work =1 A+ B capacity to do work in one day =
1
30
→1
Therefore Total work = 10
B +C capacity to do work in one day =
Out of total work shwetha should complete 1
→2
4 units to get Rs 400 share. Therefore 24

sunita is completing 6 units of work and C + A capacity to do work in one day


leaving after 2 days. 1
= 30 →3

Add equations 1, 2 and 3


1
13.’A’ can do a piece of work in 4 hours; Then A + B+ C =
16
‘B’ and ‘C’ can together do it in 3 hours; ‘A’
and ‘C’ together can do it in 2 hours. How Therefore 16 days is the answer
long will ‘B’ alone take to do it?

a)10days b)11 days 15. ’A’, ‘B’ and ‘C’ completed 60% of work
in 6 days. Then ‘A’ left the job. ‘B’ and ‘C’
c)12 days d)13 days completed 25% of remaining work in 3
days. Then ‘B’ left the job. ‘C’ completed
remaining work in 12 days. How much
Solution: time ‘C’ takes to complete the work alone?
A`s capacity to do work in one hour = a)20 days b)21 days
1
→1 c)40 days d)41 day
4

B+C capacity to do work in one hour = Solution:


1
→2
3 Total work = 100%
A+C capacity to do work in one day = Work done by
1
→3
2 A+B+C=60%
Substitute equation 1 in 3 B + C = 10%
1
Then the value of C = 4 C completed 30% of work in 12 days
Substitute the value of C in equation Therefore, to complete 100% of work C
1
need 40 days.
Then the value of B is equals to 12

Therefore, B requires 12 days to complete


16. ‘A’ does 80% of work in 20 days. He
the work.
then calls in ‘B’ and they together finish
remaining work in 3 days. How long ‘B’
alone would take to do whole work?
14. A and B can do a piece of work in 30
days, while B and C can do the same a)20 days b)21 days
work in 24 days and C and A in 20 days
c)40 days d)37.5 days
then A, B and C can complete the work in
how many days? Solution:
a)12 b)14 A can complete 80% of work in 20 days

pg. 178
10 Seconds

Therefore, to complete 100% of work A Total man days required = 10× 20 =200
need 25 days
20 men worked for eight days to complete
A`s capacity to do work in one day =20× 8 = 160 man days required
1
=25 →1
So out of 200man days of work 160 is
A and B can complete 20% of work in completed, 40 is the remaining man days
of work.
three days
Therefore to complete 100% of work A Now there are 4 men to complete 40man
and B requires 15 days days of work
40
A + B capacity to do work in one day Therefore they require = 4 = 10days
1
=15 →2

Substitute equation one in equation 2 19. ’A’, ‘B’ and ‘C’ can do a piece of work
1 in 20, 30 and 60 days respectively. In how
We get B = 37.5 many days can ‘A’ do the work if he is
assisted by ‘B’ and ‘C’ on every third day?
Therefore, B requires 37.5 days to
complete the work. a) 5 b)10
c)15 d)2
17. 12 men can complete a work in 9 Solution:
days. After they worked for 6 days, 6 more
men joined them. How many more days Total work = LCM of 20 ,30 and 60 = 60
they need to complete the remaining capacity of A to do work in one day = 3
work?
a)5 b)2 capacity of B to do work in one day = 2

c)1 d)6 capacity of C to do work in one day = 1


Solution:
According to the question first two days
Total man days of work required = 12× only A will be working and third day all the
9 =108 days three will be working.
Man days of completed in 6 days =
12× 6 = 72 The amount of work done by A, B and C
in three days is equal to 12.
Man days of work remaining = 36
Therefore, to complete 60 units of work 15
Therefore to complete 36 man days of days are required.
36
work 18 men require = = 2days.
18

18. Raju hired twenty men to complete a 20. Sneha, Neha and Kamala can do the
work in 10 days. After they worked for work in 5 days 10 days and 15 days
eight days 16 men left the work. How respectively. All the three started working
many more days are required to complete on alternate days starting with Sneha.
the remaining work? After two cycles Neha and Kamala left the
job. Then Sneha worked for one more
a)10 b)20 day. The fraction of work that is left is
c)15 d)18 1 𝟏
a) 20 b) 𝟏𝟓
Solution:

pg. 179
10 Seconds

1 1 Thus, the total numbers of days required =


c) d)
25 30
4 + 7 = 11 days.
Solution:
Total work = LCM of 5 ,15 and 30 = 30
22)P can complete a work in 12 days
capacity of Sneha to do work in one day = working 8 hours a day.Q can complete the
6 same work in 8 days working 10 hours a
capacity of Neha to do work in one day = day. If both p and Q work together,working
3 8 hours a day,in how many days can they
complete the work?
capacity of Kamala to do work in one day
=2 A) 60/11 B) 61/11

The amount of work done by all the three C) 71/11 D) 72/11


in a three days cycle is 11. Explanation:

Therefore, they complete 22 units in two P can complete the work in (12 x 8) hrs =
cycle. 96 hrs
Q can complete the work in (8 x 10)
Sneha worked for one more day and hrs=80 hrs
completes 6 units of work.
Therefore, P's 1 hour work=1/96 and Q's
Total work completed till now is 28units 1 hour work= 1/80
and remaining work is two units. (P+Q)'s 1 hour's work =(1/96) + (1/80) =
Therefore the fraction of work that is left 11/480. So both P and Q will finish the
2 1
= = work in 480/11 hrs
30 15
Therefore, Number of days of 8 hours
each = (480/11) x (1/8) = 60/11
21)A, B and C can do a piece of work in
24 days, 30 days and 40 days
respectively. They began the work 23)A,B,C together can do a piece of work
together but C left 4 days before the in 10 days.All the three started workingat it
completion of the work. In how many days together and after 4 days,A left.Then,B
was the work completed? and C together completed the work in 10
A) 11 days B) 12 days more days.In how many days can
complete a work alone ?
C) 13 days D) 14 days
A) 25 B) 24
Explanation:
C) 23 D) 21
One day's work of A, B and C = (1/24 +
1/30 + 1/40) = 1/10. Explanation:

C leaves 4 days before completion of the (A+B+C) do 1 work in 10 days.


work, which means only A and B work So (A+B+C)'s 1 day work=1/10 and as
during the last 4 days. they work together for 4 days so workdone
Work done by A and B together in the last by them in 4 days=4/10=2/5
4 days = 4 (1/24 + 1/30) = 3/10. Remaining work=1-2/5=3/5
Remaining Work = 7/10, which was done (B+C) take 10 more days to complete 3/5
by A,B and C in the initial number of days. work. So( B+C)'s 1 day work=3/50
Number of days required for this initial Now A'S 1 day work=(A+B+C)'s 1 day
work = 7 days. work - (B+C)'s 1 day work=1/10-3/50=1/25

pg. 180
10 Seconds

A does 1/25 work in in 1 day => 60M + 90W = 100M


Therefore 1 work in 25 days. => 40M = 90W
=> 4M = 9W.
24)If 6 men and 8 boys can do a piece of From the given data,
work in 10 days while 26 men and 48 boys
can do the same in 2 days what is time 1M can do the work in 100 days
taken by 15 men and 20 boys? => 4M can do the same work in 100/4= 25
A) 4 days B) 6 days days.

C) 7 days D) 5 days => 9W can do the same work in 25 days.

Explanation: => 1W can do the same work in 25 x 9 =


225 days.
Given that
Hence, 1 woman can do the same work in
6 men and 8 boys can do a piece of work 225 days.
in 10 days
26 men and 48 boys can do the same in 2
days 26)3 men, 4 women and 6 children can
complete a work in 7 days. A woman does
As the work done is equal, double the work a man does and a child
does half the work a man does. How many
10(6M + 8B) = 2(26M + 48B) women alone can complete this work in 7
60M + 80B = 52M + 96B days ?

=> M = 2B A) 6 B) 9 C) 5 D) 7

=> B = M/2 ……(1) Explanation:

Now Put (1) in 15M + 20B Let 1 woman's 1 day work = x.

=> 15M + 10M = 25M Then, 1 man's 1 day work = x/2 and 1
child's 1 day work x/4.
Now, 6M + 8B in 10 days
So, (3x/2 + 4x + + 6x/4) = 1/7
=> (6M + 4M) 10 = 100M
28x/4 = 1/7 => x = 1/49
Then D(25M) = 100M
1 woman alone can complete the work in
=> D = 4 days. 49 days.
So, to complete the work in 7 days,
number of women required = 49/7 = 7.
25)10 men and 15 women together can
complete a work in 6 days. It takes 100
days for one man alone to complete the
same work. How many days will be 27)A works twice as fast as B. If B can
required for one woman alone to complete complete a work in 18 days independently,
the same work? the number of days in which A and B can
together finish the work is:
A) 215 days B) 225 days
A) 4 days B) 6 days
C) 235 days D) 240 days
C) 8 days D) 10 days
Explanation:
Explanation:
Given that
(10M + 15W) x 6 days = 1M x 100 days

pg. 181
10 Seconds

Ratio of rates of working of A and B =2:1. 20 days. They started the work together
So, ratio of times taken =1:2 and A leaves after 2 days and B leaves
after 4 days from the beginning. How long
Therefore, A's 1 day's work=1/9 will work lost?
B's 1 day's work=1/18 A. 8 2/3 days B. 9 2/3 days
(A+B)'s 1 day's work= 1/9 + 1/18 = 1/6 C. 10 2/3 days D. 10 days
so, A and B together can finish the work in Answer: Option C
6 days
Explanation:
2/10 + 4/15 + x/20 = 1
28)A is 30% more efficient than B. How
much time will they, working together, take x = 32/3 = 10 2/3
to complete a job which A alone could
have done in 23 days ?
A) 9 days B) 11 days Mixture and Alligation
C) 13 days D) 15 days Alligation: It refers to a rule that helps to
find the ratio in which two or more
Explanation: ingredients at a given price are mixed to
Ratio of times taken by A and B = 100 : produce a mixture of specified price.
130 = 10 : 13. Mean Price: It is the cost price of a unit
Suppose B takes x days to do the work. quantity of a mixture which is prepared by
mixing two or more ingredients.
Then, 10 : 13 :: 23 : x => x = ( 23 x
13/10) => x = 299 /10. Alligation rule: It says that if two
ingredients at a given price are mixed to
A's 1 day's work = 1/23 ; produce a mixture at the given price, the
ratio of quantity of cheaper ingredient and
B's 1 day's work = 10/299 . quantity of dearer ingredient is given by;
(A + B)'s 1 day's work = ( 1/23 + 10/299 )
= 23/299 = 113 .
Therefore, A and B together can complete
the work in 13 days.

29)A can do a piece of work in 4 days. B


can do it in 5 days. With the assistance of
C they completed the work in 2 days. Find
in how many days can C alone do it?
A. 10 days B. 20 days
C. 5 days D. 4 days
Answer: Option B
Explanation:
Cheaper quantity: Dearer quantity: (d-m) :(
C = 1/2 - 1/4 - 1/5 = 1/20 => 20 days
m-c)

30) A can do a piece of work in 10 days


and B can do it in 15 days and C can do it

pg. 182
10 Seconds

Questions: If the process is repeated one more time


and 10 liters of the mixture are removed,
1. A mixture of 150 liters of wine and water then amount of milk removed = 4/5 * 10 =
contains 20% water. How much more 8 liters.
water should be added so that water
becomes 25% of the new mixture? Amount of water removed = 2 liters.
A. 7 liters B. 15 liters Remaining milk = (16 - 8) = 8 liters.
C. 10 liters D. 9 liters Remaining water = (4 -2) = 2 liters.
Answer: Option C The required ratio of milk and water in the
final mixture obtained = (8 + 10):2 = 18:2 =
Explanation: 9:1.
Number of liters of water in150 liters of the
mixture = 20% of 150 = 20/100 * 150 = 30
liters. 3. In what ratio should two varieties of
sugar of Rs.18 per kg and Rs.24 kg be
P liters of water added to the mixture to mixed together to get a mixture whose
make water 25% of the new mixture. cost is Rs.20 per kg?
Total amount of water becomes (30 + P) A. 1:3 B. 3:1
and total volume of mixture is (150 + P).
C. 1:2 D. 2:1
(30 + P) = 25/100 * (150 + P)
Answer: Option D
120 + 4P = 150 + P => P = 10 liters.
Explanation:

2. A vessel contains 20 liters of a mixture


of milk and water in the ratio 3:2. 10 liters
of the mixture are removed and replaced
with an equal quantity of pure milk. If the
process is repeated once more, find the
ratio of milk and water in the final mixture
obtained?
A. 9:1 B. 4:7
C. 7:1 D. 2:5
Answer: Option A
Explanation:
Milk = 3/5 * 20 = 12 liters, water = 8 liters 4. How many liters of oil at Rs.40 per liter
If 10 liters of mixture are removed, amount should be mixed with 240 liters of a
of milk removed = 6 liters and amount of second variety of oil at Rs.60 per liter so
water removed = 4 liters. as to get a mixture whose cost is Rs.52
per liter?
Remaining milk = 12 - 6 = 6 liters
A. 120 liters B. 180 liters
Remaining water = 8 - 4 = 4 liters
C. 110 liters D. 160 liters
10 liters of pure milk are added, therefore
total milk = (6 + 10) = 16 liters. Answer: Option D

The ratio of milk and water in the new Explanation:


mixture = 16:4 = 4:1

pg. 183
10 Seconds

(144 + 36)/216 * 100% = 5/6 * 100% =


83.33%.

7. In a can, there is a mixture of milk and


water in the ratio 4 : 5. If it is filled with an
additional 8 litres of milk the can would be
full and ratio of milk and water would
become 6 : 5. Find the capacity of the
can?
5. Two varieties of wheat - A and B
costing Rs. 9 per kg and Rs. 15 per kg A. 40 B. 44
were mixed in the ratio 3 : 7. If 5 kg of the C. 48 D. 52
mixture is sold at 25% profit, find the profit
made? Answer: Option B
A. Rs. 13.50 B. Rs. 14.50 Explanation:
C. Rs. 15.50 D. Rs. 16.50 Let the capacity of the can be T litres.
Answer: Option D Quantity of milk in the mixture before
adding milk = 4/9 (T - 8)
Explanation:
After adding milk, quantity of milk in the
Let the quantities of A and B mixed be 3x mixture = 6/11 T.
kg and 7x kg.
6T/11 - 8 = 4/9(T - 8)
Cost of 3x kg of A = 9(3x) = Rs. 27x
10T = 792 - 352 => T = 44.
Cost of 7x kg of B = 15(7x) = Rs. 105x
Cost of 10x kg of the mixture = 27x + 105x
= Rs. 132x 8. In what ratio should a variety of rice
costing Rs. 6 per kg be mixed with another
Cost of 5 kg of the mixture = 132x/10x (5) variety of rice costing Rs. 8.75 per kg to
= Rs. 66 obtain a mixture costing Rs. 7.50 per kg?
Profit made in selling 5 kg of the mixture = A. 5 : 6 B. 3 : 4
25/100 (cost of 5 kg of the mixture) =
25/100 * 66 = Rs. 16.50 C. 7 : 8 D. 8 : 9
Answer: Option A
6. In a mixture of milk and water, the Explanation:
proportion of milk by weight was 80%. If,
in a 180 gm mixture, 36 gms of pure milk Let us say the ratio of the quantities of
is added, what would be the percentage of cheaper and dearer varieties = x : y
milk in the mixture formed? By the rule of allegation, x/y = (87.5 - 7.50)
A. 80% B. 100% / (7.50 - 6) = 5/6

C. 84% D. 87.5%
E. None of these 9. A mixture of 70 litres of milk and water
contains 10% water. How many litres of
Answer: Option E water should be added to the mixture so
that the mixture contains 12 1/2% water?
Explanation:
A. 2 B. 8 C. 4 D. 5
Percentage of milk in the mixture formed =
[80/100 (180) + 36] / (180 + 36) * 100% = Answer: Option A

pg. 184
10 Seconds

Explanation: 11. Two vessels P and Q contain 62.5%


and 87.5% of alcohol respectively. If 2
Quantity of milk in the mixture = 90/100 litres from vessel P is mixed with 4 litres
(70) = 63 litres. from vessel Q, the ratio of alcohol and
After adding water, milk would form 87 water in the resulting mixture is?
1/2% of the mixture. A. 16 : 5 B. 14 : 5
Hence, if quantity of mixture after adding x C. 16 : 7 D. 19 : 5
liters of water, (87 1/2) / 100 x = 63 => x =
72 Answer: Option D
Hence 72 - 70 = 2 litres of water must be Explanation:
added.
Quantity of alcohol in vessel P = 62.5/100
* 2 = 5/4 litres
10. All the water in container A which was Quantity of alcohol in vessel Q = 87.5/100
filled to its brim was poured into two * 4 = 7/2 litres
containers B and C. The quantity of water
in container B was 62.5% less than the Quantity of alcohol in the mixture formed =
capacity of container A. If 148 liters was 5/4 + 7/2 = 19/4 = 4.75 litres
now transferred from C to B, then both the As 6 litres of mixture is formed, ratio of
containers would have equal quantities of alcohol and water in the mixture formed =
water. What was the initial quantity of 4.75 : 1.25 = 19 : 5.
water in container A?
A. 648 B. 888
12. A vessel of capacity 90 litres is fully
C. 928 D. 1184 filled with pure milk. Nine litres of milk is
Answer: Option D removed from the vessel and replaced
with water. Nine litres of the solution thus
Explanation: formed is removed and replaced with
water. Find the quantity of pure milk in the
B has 62.5% or (5/8) of the water in A. final milk solution?
Therefore, let the quantity of water in
container A(initially) be 8k. A. 72 B. 72.9
Quantity of water in B = 8k - 5k = 3k. C. 73.8 D. 74.7
Quantity of water in container C = 8k - 3k Answer: Option B
= 5k
Explanation:
Container: A B C
Let the initial quantity of milk in vessel be
Quantity of water: 8k 3k 5k T litres.
It is given that if 148 liters was transferred Let us say y litres of the mixture is taken
from container C to container B, then both out and replaced by water for n times,
the containers would have equal quantities alternatively.
of water.
Quantity of milk finally in the vessel is then
5k - 148 = 3k + 148 => 2k = 296 => k = given by [(T - y)/T]n * T
148
For the given problem, T = 90, y = 9 and n
The initial quantity of water in A = 8k = 8 * = 2.
148 = 1184 liters.
Hence, quantity of milk finally in the
vessel
= [(90 - 9)/90]2 (90) = 72.9 litres.

pg. 185
10 Seconds

should be mixed to obtain a new mixture


in vessel C containing half milk and half
13. The ratio in which the price at Rs.7.20 water?
a kg be mixed with rice at Rs. 5.70 a kg to
produce a mixture worth Rs.6.30 a kg is: A. 1:1 B. 1:3
A. 1:3 B. 2:3 C. 1:2 D. 7:5
C. 3:4 D. 4:5 Answer: Option D
Answer: Option A Explanation:
Explanation: Milk in A=4/7 of whole milk in B=2/5 of
whole milk in mixture A and B =1/2 of the
(Dearer Rice): (Cheaper Rice) = 60 : 90 = whole.
2:3
Let the C.P of unit quantity be Re.1
Required ratio = 1/10 = 1/14 =14 : 10 =7: 5
14. In what ratio must a grocer mix teas
worth Rs.60 a kg and Rs.65 a kg.So that
by selling the mixture at Rs. 68.20 a kg,
He may gain 10%? 17. How much water must be added to a
bucket which contains 40 liters of milk at
A. 3:2 B. 3:4 the cost price of Rs.3.50 per liter so that
the cost of milk reduces to Rs.2 per liter?
C. 3:5 D. 4:5
A. 25 liters B. 28 litres
Answer: Option A
C. 30 liters D. 35 liters
Explanation:
Answer: Option C
S.P of 1kg mix = RS.68.20, Gain = 10%
C.P of 1kg mix =Rs.(100/110 x 68.20) Explanation:
=Rs.62 (Cheaper Tea) : (Dearer Tea) = 3 :
2 Total cost price =Rs(40 x 7/2) = Rs.140
Cost per litre =Rs.2,

15. In what ratio must water be mixed with Total quantity = 140/2 = 70 Litres.
milk to gain 16 2/3 % by selling the Water to be added =(70-40) =30 Litres.
mixture at cost price?
A. 1:6 B. 2:3
18. In what proportion must water be
C. 4:3 D. 6:1 added to spirit to gain 20% by selling it at
Answer: Option A the cost price?

Explanation: A. 2:5 B. 1:5

Let the C.P of the milk be Re. 1 per litre C. 3:5 D. 4:5
Then S.P of 1 litre of mix =Re 1, Gain% Answer: Option B
=50/3% C.P of 1 Litre of Mix =Rs(100 x
3/350 x 1) =Re 6/7 Explanation:
Ratio of water and milk =1/7 = 6/7 = 1: 6 Let the C.P be Re.1 per liter
S.P of 1 litre of mix =Re 1,
16. Milk and water in two vessels A and B Gain = 20%
are in the ratio 4:3 and 2:3 respectively in
C.P of 1 litre mix =Rs(100/120 x 1)
what ratio the liquids in both the vessels
=Re(5/6)

pg. 186
10 Seconds

Ratio of water and spirit = 1/6 =5/6 = 1 : 5. same quantity. But now the wine to water
ratio became 3:2. How much water did
Rajesh add?
19. Find the ratio in which rice at Rs. 7.20 A. 3/5 B. 1/2
a kg be mixed with rice at Rs. 5.70 a kg to
produce a mixture worth Rs. 6.30 a kg. C. 1/4 D. 2/7
A. 1 : 3 B. 2 : 3 Answer : Option C
C. 3 : 4 D. 4 : 5
Answer : Option B 24)In a pot, there is a mixture of milk and
water in the ratio 4 : 5. If it is filled with an
additional 8 litres of milk, the pot would be
20. In what ratio must a grocer mix two full and ratio of milk and water would
varieties of tea worth Rs. 60 a kg and Rs. become 6 : 5. Find the capacity of the pot
65 a kg so that by selling the mixture at ?
Rs. 68.20 a kg he may gain 10%? A) 11 lit B) 22 lit
A. 3 : 2 B. 3 : 4 C) 33 lit D) 44 lit
C. 3 : 5 D. 4 : 5 Answer: B) 22 lit
Answer: Option A Explanation:
Let the capacity of the pot be 'P' litres.
21. The cost of Type 1 rice is Rs. 15 per Quantity of milk in the mixture before
kg and Type 2 rice is Rs. 20 per kg. If both adding milk = 4/9 (P - 8)
Type 1 and Type 2 are mixed in the ratio
of 2 : 3, then the price per kg of the mixed After adding milk, quantity of milk in the
variety of rice is: mixture = 6/11 P.
A. Rs. 18 B. Rs. 18.50 6P/11 - 8 = 4/9(P - 8)
C. Rs. 19 D. Rs. 19.50 10P = 792 - 352 => P = 44.
Answer : Option A The capacity of the pot is 44 liters.

22. 8 litres are drawn from a cask full of 25) A mixture of 150 liters of wine and
wine and is then filled with water. This water contains 20% water. How much
operation is performed three more times. more water should be added so that water
The ratio of the quantity of wine now left in becomes 25% of the new mixture?
cask to that of water is 16 : 65. How much
wine did the cask hold originally? A) 10 liters B) 20 liters

A. 18 litres B. 24 litres C) 30 liters D) 40 liters

C. 32 litres D. 42 litres Answer: A) 10 liters

Answer : Option B Explanation:


Number of liters of water in 125 liters of
the mixture = 20% of 150 = 1/5 of 150 =
23. Rajesh has a container which has a 30 liters.
mixture of wine and water in it. Wine and
water are in the ratio 4:1. Rajesh spills Let us Assume that another 'P' liters of
some of the mixture by accident. He then water are added to the mixture to make
replaces the spilled amount with water of water 25% of the new mixture. So, the

pg. 187
10 Seconds

total amount of water becomes (30 + P) 27) A milk man sells the milk at the cost
and the total volume of the mixture price but he mixes the water in it and thus
becomes (150 + P) he gains 9.09%. The quantity of water in
the mixture of 1 liter is :
Thus, (30 + P) = 25% of (150 + P)
A) 83.33 ml B) 90.90 ml
Solving, we get P = 10 liters
C) 99.09 ml D) can't be
determined
26) A vessel contains 20 liters of a mixture Answer: A) 83.33 ml
of milk and water in the ratio 3:2. 10 liters
of the mixture are removed and replaced Explanation:
with an equal quantity of pure milk. If the
process is repeated once more, find the Profit (%) = 9.09 % = 1/11
ratio of milk and water in the final mixture Since the ratio of water and milk is 1 : 11,
obtained ?
Therefore the ratio of water is to mixture =
A) 5:3 B) 1:4 1:12
C) 4:1 D) 9:1 Thus the quantity of water in mixture of 1
Answer: D) 9:1 liter = 1000 x (1/12) = 83.33 ml

Explanation:
Milk = 3/5 x 20 = 12 liters, water = 8 liters 28) In a mixture of milk and water the
proportion of water by weight was 75%. If
If 10 liters of mixture are removed, amount in 60 gm of mixture 15 gm water was
of milk removed = 6 liters and amount of added, what would be the percentage of
water removed = 4 liters. water ? (Weight in gm)
Remaining milk = 12 - 6 = 6 liters A) 80% B) 70%
Remaining water = 8 - 4 = 4 liters C) 75% D) 62%
10 liters of pure milk are added, therefore Answer: A) 80%
total milk = (6 + 10) = 16 liters.
Explanation:
The ratio of milk and water in the new
mixture = 16:4 = 4:1 Water in 60 gm mixture=60 x 75/100 = 45
gm. and Milk = 15 gm.
If the process is repeated one more time
and 10 liters of the mixture are removed, After adding 15 gm. of water in mixture,
total water = 45 + 15 = 60 gm and weight
then amount of milk removed = 4/5 x 10 = of a mixture = 60 + 15 = 75 gm.
8 liters.
So % of water = 100 x 60/75 = 80%.
Amount of water removed = 2 liters.
Remaining milk = (16 - 8) = 8 liters.
29) A mixture of 150 liters of wine and
Remaining water = (4 -2) = 2 liters. water contains 20% water. How much
more water should be added so that water
Now 10 lts milk is added => total milk = 18 becomes 25% of the new mixture ?
lts
A) 5 lit B) 10 lit
The required ratio of milk and water in the
final mixture obtained= (8 + 10):2 = 18:2 = C) 15 lit D) 20 lit
9:1.
Answer: B) 10 lit
Explanation:

pg. 188
10 Seconds

Number of liters of water in 150 liters of R is sitting next to the right of P and P is
the mixture = 20% of 150 = 20/100 x 150 second to the right of O.
= 30 liters.
P liters of water added to the mixture to
make water 25% of the new mixture. Q1. Who is sitting in the center of the row?

Total amount of water becomes (30 + P) 1. N 2. O


and total volume of mixture is (150 + P). 3. S 4. U
(30 + P) = 25/100 x (150 + P)
120 + 4P = 150 + P => P = 10 liters. Q2. Which of the following people are
sitting to the right of S?

30) One type of liquid contains 25 % of 1. OTPQ 2. OTPR


benzene, the other contains 30% of 3. UNVM 4. UOTPR
benzene. A can is filled with 6 parts of the
first liquid and 4 parts of the second liquid.
Find the percentage of benzene in the
Q3. Which of the following statements is
new mixture.
true with respect to the above
A) 27 % B) 26 % arrangement?
C) 29 % D) 21 % 1. There are three persons sitting between
P and S
Answer: A) 27 %
2. W is between M and V.
Explanation:
3. N is sitting between V and U
Let the percentage of benzene = X
4. S and O are neighbours sitting to the
(30 - X)/(X- 25) = 6/4 = 3/2 immediate right of T
=> 5X = 135
X = 27 Q4. Who are the immediate neighbours of
So, required percentage of benzene = 27 T?
% 1. O, P 2. O, R
3. N, U 4. V, U

Seating arrangement
1 to 5)Directions for Questions 1 to 5: Q.5. If Q and P, O and N, M and T, and W
Read the given information carefully and and R interchange their positions then
answer the questions that follow. which of the following pairs of friends is
sitting at the ends?
Eleven friends M, N, O, P, Q, R, S, T, U, V
and W are sitting in the first row of the 1. P and Q
stadium watching a cricket match. 2. Q and R
T is to the immediate left of P and third to 3. P and W
the right of U.
4. W and R
V is the immediate neighbour of M and N
and third to the left of S. Explanation :
M is the second to the right of Q, who is at The arrangement of the persons is
one of the ends.
QWMVNUSOTPR

pg. 189
10 Seconds

6 to 10)Directions for Questions 6 to 10: Q. 10 Which of the following has the


Read the following information given second person sitting between the first
below and answer the questions that and third persons?
follow.
1. SPQ 2. VRT
P, Q, R, S, T, U and V are seven friends
and are sitting in a circle facing the center 3. QRP 4. VUS
of the circle.
V is second to the left of S and is the Explanation :
neighbour of T and U.
S is not a neighbour of R or T.
P is neighbour of Q and R.

Q.6. Which of the following is correct?


1. Q is between P and S.
2. S is between U and P.
3. T is to the immediate right of V.
4. U is to the immediate left of V.

11 to 16)Study the following information


carefully and answer the given questions.
Q.7. Which of the following has the pair
with the second person sitting to the Twelve people are sitting in two parallel
immediate right of the first person? rows containing six people each such that
they are equidistant from each other.
1. QU 2. VU
In row 1: P, Q, R, S, T and V are seated
3. TR 4. PT
and all of them are facing South.
In row 2: A, B, C, D, E and F are seated
Q8. Which of the following will be S’s and all of them are facing North.
position after T and S interchange their Therefore, in the given seating
places? arrangement, each member seated in a
row faces another member of the row.
1. Neighbour of V and R
S sits third to the right of Q, either S or Q
2. To the immediate left of R sits at an extreme end of the line. The one
3. To the immediate right of U who faces Q sits second to the right of E.
Two people sit between B and F, neither B
4. Neighbour of R and P nor F sits at extreme end of the line. The
immediate neighbour of B faces the
person who sits third to the left of P. R and
Q9. What is the position of R? T are immediate neighbours. C sits
second to the left of A. T does not face the
1. Second to the left of Q immediate neighbour of D.
2. Third to the right of U
3. To the immediate left of P Q11. Who amongst the following sit at the
extreme ends of the rows?
4. None of these

pg. 190
10 Seconds

(a) S, D (b) Q A
(c) V, C (d) P, D

Q12. Who amongst the following faces S?


(a) A (b) B (c) C (d) D
17 to 22)Twelve people are sitting in two
parallel rows containing six people each
Q13. How many persons are seated such that they are equidistant from each
between V and R? other. In row 1, Anika, Bianca, Catlyn,
Dhruv, Esha and Feroj are sitting facing
(a) one (b) two South. In row 2, Parul, Qadir, Rudra,
Saumya, Tanuj and Vikram are sitting
(c) three (d) four
facing North. Therefore, in the given
seating arrangement, each member sitting
in a row faces another member of the
Q14. P is related to A in the same way as other row. Three persons sit between
S is related to B based on the given Catlyn and Dhruv. Either Catlyn or Dhruv
arrangement. Which of the following is T sits at an extreme end of the line. The one
related to, following the same pattern? who faces Dhruv sits third to the left of
(a) C (b) D (c) E (d) F Rudra. Saumya faces the one who sits
third to the left of Anika and he cannot sit
adjacent to Rudra. The immediate
neighbour of Qadir faces the immediate
Q15. Which of the following is true neighbour of Anika. Only one person sits
regarding T? between Parul and Tanuj, who is facing
(a) F faces T the one sitting on the immediate right of
Esha. Neither Esha nor Feroj faces Rudra.
(b) V is an immediate neighbor of T Vikram and Qadir cannot sit adjacent to
each other.
(c) F faces the one who is second to
the right of T
(d) T sits at one of the extreme ends of the Q17. Who among the following faces
line. Bianca?
(e) Q sits second to the right of T (a) Parul (b) Qadir
(c) Tanuj (d) Rudra
Q16. Four of the following five are alike in
a certain way based on the given
arrangement and so form a group. Which Q18. Who among the following sit at the
is the one that does not belong to that extreme ends of the rows?
group? (a) Dhruv, Qadir (b) Tanuj, Feroj
(a) A-T (b) B-T (c) Esha, Saumya (d) Bianca, Tanuj
(c) F-P (d) C-V (e) E-Q
Explanation : Q19. If Esha is related to Parul in the
same way as Catlyn is related to Saumya,
which of the following is Anika related to,
following the same pattern?
(a) Vikram (b) Rudra

pg. 191
10 Seconds

(c) Qadir (d) Tanuj outside and four of them are facing the
centre.
• E faces outside, both the immediate
Q20. How many persons are sitting neighbours of E face the centre. H sits
between Esha and Feroj? second to the right of E. B sits third to the
(a) None (b) One left of E.

(c) Two (d) Three • D faces the centre. Both the immediate
neighbours of D face outside.
• G sits second to the left of A B sits third
Q21. Four of the following five are alike in to the right of H
a certain way based on the given
arrangement and so form a group. Which • F is an immediate neighbour of D. C is
is the one that does not belong to that an immediate neighbour of G.
group? • D is not an immediate neighbour of B
(a) Parul–Dhruv (b) Qadir–Anika
(c) Qadir–Feroj (d) Saumya–Bianca Question 23: Who sits to the immediate
right of E.

Q22. Which of the following is true a) A b) B c) C d) D e) E


regarding Parul?
(a) Qadir is an immediate neighbour of Question 24: Who sits to the immediate
Parul. left of H.
(b) Parul faces the immediate a) A b) B c) C d) D e) F
neighbour of Bianca.
(c) Parul sits second from the left end of
the row. Question 25: Who sits to the immediate
right of F.
(d) Parul sits at one of the extreme ends.
a) A b) B c) C d) D e) E
(e) Parul faces the one who is second
from the right end of the row.
Question 26: Who sits diagonally opposite
of B.
Explanation :
a) A b) B c) C d) D e) F

Question 27: Who sits diagonally opposite


of E.
a) A b) B c) C d) D e) E

Question 28: Which of the following is odd


23 to 28)Study the following information
man out
and answer the questions given below:
a) H b) E c) G d) A
Eight friends A, B, C, D, E, F, G and H are
sitting in a circle, but not necessarily in the
same order. Four of them are facing
Explanation:

pg. 192
10 Seconds

is the one that does not belong to that


group?
1) HA 2) FH
3) GC 4) DA

Q32. If all the people are made to sit in an


alphabetical order, in clockwise direction,
starting from A, the position of whom
amongst the following remains the same
(excluding A)?
1) E 2) F 3) C 4) G
29 to 34)Study the information given
below and answer the given questions.
Eight friends A, B, C, D, E, F, G and H are Q33. How many people are seated
sitting in a circle, but not necessarily in the between A and C (counting clockwise from
same order. Four of them are facing A)?
outside and four of them are facing the
centre. 1) Two 2) Four

• 1. E faces outside, Both the immediate 3) None 4) One


neighbours of E face the centre. H sits
second to the right of E. B sits third to the
left of E. Q34. Who amongst the following sits
exactly between F and C (and is also their
• 2. D faces the centre. Both the neighbour)?
immediate neighbours of D face outside.
1) E 2) B 3) G 4) A
• 3. G sits second to the left of A. B sits
third to the right of H
• 4. F is an immediate neighbour of D. C is Explanation :
an immediate neighbour of G.
• 5. D is not an immediate neighbour of B.

Q29. Who amongst the following sits on


the immediate right of H?
1) A 2) D
3) C 4) G

Q30. Who amongst the following sits third


to the right of A?
1) D 2) E 3) F 4) A

Q31. Four of the following five are alike in


a certain way, based on the information
given above and so form a group. Which

pg. 193
10 Seconds

35 to 40)P, Q, R, S, T, U, V and W are Question 40: Which of the following is odd


sitting round the circle and are not all man out
facing the centre:
a) T b) U c) W d) S e) R
P is second to the right of T who is the
neighbour of R and V. Explanation :

S is shorter than P but taller than V.


S is not the neighbour of P.
T W U and P are facing same direction
while other faces opposite direction
Q is shorter than S but taller than U.
V is the neighbour of U.
R is shorter than V and V is not as tall as
U. 41 to 45)In a school, Seven friends – M, V,
K, D, T, J and R are standing facing to
Q is not between S and W. W is not east in a particular order. Each of these
between U and S. seven friends are studying in different
classes- IIIrd. IVth, Vth, VIth, VIIth, VIIIth
T is facing the centre and is taller than H and IXth standards. Each of them has
but shorter than R. different favourite colours- yellow, blue,
red, white, black, green, and violet.

Question 35: Which of the following pair is J likes red and studies in Class Vth.
odd man out K which is third to the left of V, is at one
a)VS b) UP c) TW d) RS end.
R likes violet and studies in class IIIrd.

Question 36: Who sits to the immediate D car is between V and J.


left of W. M studies in Class VIIIth and does not like
a) W b) S c) Q d) V e) R green and yellow.
K likes white and does not study in VIIth
and in IVth.
Question 37: Who sits to the immediate
right of T. R is fourth to the right of K.

a) W b) S c) Q d) V e) R D studies in VIth and likes black.


M is next to right of R.

Question 38: Who sits diagonally opposite T does not study in IVth. V does not like
of T. green.

a) W b) S c) Q d) W e) P seven friends studying in different classes-


IIIrd. IVth, Vth, VIth, VIIth, VIIIth and IXth
standards.
Question 39: Who sits diagonally opposite
of U.
Q41. Which of the cars are on both the
a) W b) S c) Q d) V e) P sides of M car ?
A.V and D B.D and K

pg. 194
10 Seconds

C.R and T D.V and R circular table but not facing the centre.
Each member has a different zodiac sign,
viz Leo, Virgo, Libra, Cancer, Aries,
Q42. Which of the following statement is Gemini, Pisces and Scorpio, but not
correct ? necessarily in the same order. T is third to
the right of P. The one whose sun-sign
A.D is next left of V. Leo is second to the left of the one whose
sun-sign is Libra.
B.J is next left of K.
Y’s sun-sign is Libra and is sitting exactly
C.J is at one end.
between P and L. The one whose sun-
D.K is next second to the right of D. sign is Pisces sits second to the right of N.
The one whose sun-sign is Aries is
second to the right of the person whose
Q43. Which one of the following sun-sign is Gemini. P sits third to the left
statements is correct ? of the person whose sun-sign is Virgo.
Neither Q nor L is the immediate
A.R car is in between V and K. neighbour of N. Q is fourth to the left of L.
N’s sun-sign is neither Cancer nor Ariel.
B.M is next left to T car. The person whose sun-sign is Leo is
C.R is next right of M. sitting second to the right of the person
whose sign is Cancer. R’s sun-sign is Leo
D.D is fourth right of T. and is not an immediate neighbor of N.

Q44. Which of the following groups of cars Q46. Which of the following is N’s sun-
is to the right of V ? sign?
A.M, R and D B.T, M and R a) Pisces b) Scorpio
C.D, J and K D.J, M and R c) Gemini d) Can’t be determined

Q45. Which one of the following is the Q47. Who sits third to the right of L?
correct position of T ?
a) P b) Y c) R d) Q
A. Next to the left of M
e) None of these
B. Next to the left of J
C. Between J and R
Q48. What is Y’s position with respect to
D. Fourth to the right of D. Q?
Explanation : a) Third to the left b) Fourth to the left
c) Second to right d) Third to the right

Q49. How many persons are there


between P and N?
a) None b) Two
c) Three d) Four

45 to 50) L, N, P, R, M, Q, T and Y are the


members of a committee sitting around a Q50. What is the sun-sign of P?

pg. 195
10 Seconds

a) Gemini b) Libra Trapezium ½ h(a+b) a+b+c+d


c) Leo d) Can’t be determined
3D Figures
Shape Volume Curved Total
Explanation :
(Cubic Surface Surface
units) Area Area
(CSA) (TSA)
(Square
units)

Cube a³ 4 a² 6 a²
Cuboid l×b× 2 h (l + 2 (lb
h b) +bh
+hl)
Sphere (4/3) π 4 π r² 4 π r²

Mensuration Hemisphere (⅔) π 2 π r² 3 π r²

2D Figures Cylinder π r² h 2π r h 2πrh +
Shape Area(Sq units) Perimete 2πr²
r Cone (⅓) π πrl πr (r +
(Sq r² h l)
units)
1. What is the area of an equilateral
Square a² 4a triangle of side 16 cm?
A. 48√3 cm2 B. 128√3 cm2 C.
9.6√3 cm2 D. 64√3 cm2
Rectangle l×b 2 ( l + b)
Explanation:

Circle πr² 2πr Area of an equilateral triangle = √3/4 S2


a+b+c If S = 16, Area of triangle = √3/4 * 16 * 16
= 64√3 cm2;
Scalene √[s(s−a)(s−b)(s−
Triangle c)],
2. If the sides of a triangle are 26 cm, 24
Where, s = cm and 10 cm, what is its area?
A. 120 cm2 B. 130 cm2
(a+b+c)/2
Isosceles ½×b×h 2a + b C. 312 cm2 D. 315 cm2
Triangle
Explanation:
Equilateral (√3/4) × a² 3a
Triangle The triangle with sides 26 cm, 24 cm and
Right Angle ½×b×h b+ 10 cm is right angled, where the
Triangle hypotenu hypotenuse is 26 cm.
se + h
Area of the triangle = 1/2 * 24 * 10 = 120
Rhombus ½ × d1 × d2 4 × side cm2
Parallelogr b×h 2(l+b)
am

pg. 196
10 Seconds

3.The perimeter of a triangle is 28 cm and 12x2 = 6912


the inradius of the triangle is 2.5 cm. What
is the area of the triangle? x2 = 576 = 4 * 144 = 22 * 122 (x > 0)

A. 25 cm2 B. 42 cm2 => x = 2 * 12 = 24

C. 49 cm2 D. 70 cm2 Ratio of the breadth and the areas = 3x :


12x2 = 1 : 4x = 1: 96.
Explanation:
Area of a triangle = r * s
7. The area of the square formed on the
Where r is the inradius and s is the semi diagonal of a rectangle as its side is 108
perimeter of the triangle. 1/3 % more than the area of the rectangle.
If the perimeter of the rectangle is 28
Area of triangle = 2.5 * 28/2 = 35 cm2 units, find the difference between the
sides of the rectangle?

4. Find the area of trapezium whose A. 8 B. 12 C. 6 D. 2


parallel sides are 20 cm and 18 cm long, Explanation:
and the distance between them is 15 cm.
Let the sides of the rectangle be l and b
A. 225 cm2 B. 275 cm2 respectively.
C. 285 cm2 D. 315 cm2 From the given data,
Explanation: (√l2 + b2) = (1 + 108 1/3 %)lb
Area of a trapezium = 1/2 (sum of parallel => l2 + b2 = (1 + 325/3 * 1/100)lb
sides) * (perpendicular distance between
them) = 1/2 (20 + 18) * (15) = 285 cm2 = (1 + 13/12)lb
= 25/12 lb
5. Find the area of a parallelogram with => (l2 + b2)/lb = 25/12
base 24 cm and height 16 cm.
12(l2 + b2) = 25lb
A. 262 cm2 B. 384 cm2
Adding 24lb on both sides
C. 192 cm2 D. 131 cm2
12l2 + 12b2 + 24lb = 49lb
Explanation:
12(l2 + b2 + 2lb) = 49lb
Area of a parallelogram = base * height =
24 * 16 = 384 cm2 but 2(l + b) = 28 => l + b = 14
12(l + b)2 = 49lb

6. The ratio of the length and the breadth => 12(14)2 = 49lb
of a rectangle is 4 : 3 and the area of the => lb = 48
rectangle is 6912 sq cm. Find the ratio of
the breadth and the area of the rectangle? Since l + b = 14, l = 8 and b = 6
A. 1 : 96 B. 1 : 48 l - b = 8 - 6 = 2m.
C. 1 : 84 D. 1 : 68
Explanation: 8. The length of a rectangular plot is thrice
its breadth. If the area of the rectangular
Let the length and the breadth of the plot is 867 sq m, then what is the breadth
rectangle be 4x cm and 3x respectively. of the rectangular plot?
(4x)(3x) = 6912 A. 8.5 m B. 17 m

pg. 197
10 Seconds

C. 34 m D. 51 m Cost of the second carpet = (45)(12.96 *


7) = 315 (13 - 0.04) = 4095 - 12.6 = Rs.
Explanation: 4082.40
Let the breadth of the plot be b m.
Length of the plot = 3 b m 11. What will be the cost of building a
(3b)(b) = 867 fence around a square plot with area
equal to 289 sq ft, if the price per foot of
3b2 = 867 building the fence is Rs. 58?
b2 = 289 = 172 (b > 0) A. Rs. 3944 B. Rs. 3828
b = 17 m. C. Rs. 4176 D. None of these
Explanation:
9. The length of a rectangular floor is more Let the side of the square plot be a ft.
than its breadth by 200%. If Rs. 324 is
required to paint the floor at the rate of Rs. a^2 = 289 => a = 17
3 per sq m, then what would be the length Length of the fence = Perimeter of the plot
of the floor? = 4a = 68 ft.
A. 27 m B. 24 m Cost of building the fence = 68 * 58 = Rs.
C. 18 m D. 21 m 3944.

Explanation:
Let the length and the breadth of the floor 12. The area of a square is equal to five
be l m and b m respectively. times the area of a rectangle of
dimensions 125 cm * 64 cm. What is the
l = b + 200% of b = l + 2b = 3b perimeter of the square?
Area of the floor = 324/3 = 108 sq m A. 600 cm B. 800 cm
l b = 108 i.e., l * l/3 = 108 C. 400 cm D. 1000 cm
l2 = 324 => l = 18. Explanation:
Area of the square = s * s = 5(125 * 64)
10. An order was placed for the supply of => s = 25 * 8 = 200 cm
a carpet whose breadth was 6 m and
length was 1.44 times the breadth. What Perimeter of the square = 4 * 200 = 800
be the cost of a carpet whose length and cm.
breadth are 40% more and 25% more
respectively than the first carpet. Given
that the ratio of carpet is Rs. 45 per sq m? 13. A wire in the form of a circle of radius
3.5 m is bent in the form of a rectangule,
A. Rs. 3642.40 B. Rs. 3868.80 whose length and breadth are in the ratio
C. Rs. 4216.20 D. Rs. 4082.40 of 6 : 5. What is the area of the rectangle?

Explanation: A. 60 cm2 B. 30 cm2

Length of the first carpet = (1.44)(6) = 8.64 C. 45 cm2 D. 15 cm2


cm Explanation:
Area of the second carpet = 8.64(1 + The circumference of the circle is equal to
40/100) 6 (1 + 25/100) the permeter of the rectangle.
= 51.84(1.4)(5/4) sq m = (12.96)(7) sq m

pg. 198
10 Seconds

Let l = 6x and b = 5x 2(6x + 5x) = 2 * 22/7 16. The perimeter of a square is equal to
* 3.5 the perimeter of a rectangle of length 16
cm and breadth 14 cm. Find the
=> x = 1 circumference of a semicircle whose
Therefore l = 6 cm and b = 5 cm Area of diameter is equal to the side of the square.
the rectangle = 6 * 5 = 30 cm2 (Round off your answer to two decimal
places)
A. 77.14 cm B. 47.14 cm
14. The area of a square is 4096 sq cm.
Find the ratio of the breadth and the length C. 84.92 cm D. 94.94 cm E. 23.57 cm
of a rectangle whose length is twice the Explanation:
side of the square and breadth is 24 cm
less than the side of the square. Let the side of the square be a cm.
A. 18 : 5 B. 7 : 16 Parameter of the rectangle = 2(16 + 14) =
60 cm Parameter of the square = 60 cm
C. 5 : 16 D. 5 : 32 i.e. 4a = 60
Explanation: A = 15
Let the length and the breadth of the Diameter of the semicircle = 15 cm
rectangle be l cm and b cm respectively.
Let the side of the square be a cm. Circimference of the semicircle
a^2 = 4096 = 2^12 = 1/2(a)(15)
a = (2^12)^1/2 = 26 = 64 = 1/2(22/7)(15) = 330/14 = 23.57 cm to
two decimal places
L = 2a and b = a - 24
b : l = a - 24 : 2a = 40 : 128 = 5 : 16
17. A cube of side one meter length is cut
into small cubes of side 10 cm each. How
15. The parameter of a square is double many such small cubes can be obtained?
the perimeter of a rectangle. The area of A. 10 B. 100 C. 1000
the rectangle is 480 sq cm. Find the area
of the square. D. 10000 E. None of these
A. 200 sq cm B. 72 sq cm Answer: Option C
C. 162 sq cm D. Cannot be Explanation:
determined
Along one edge, the number of small
Explanation: cubes that can be cut
Let the side of the square be a cm. Let the = 100/10 = 10
length and the breadth of the rectangle be
l cm and b cm respectively. Along each edge 10 cubes can be cut.
(Along length, breadth and height). Total
4a = 2(l + b) number of small cubes that can be cut =
10 * 10 * 10 = 1000
2a = l + b
l . b = 480
18. The dimensions of a room are 25 feet
We cannot find ( l + b) only with the help * 15 feet * 12 feet. What is the cost of
of l . b. Therefore a cannot be found . white washing the four walls of the room at
Area of the square cannot be found. Rs. 5 per square feet if there is one door
of dimensions 6 feet * 3 feet and three

pg. 199
10 Seconds

windows of dimensions 4 feet * 3 feet 21. The ratio of the volumes of two cubes
each? is 729 : 1331. What is the ratio of their
total surface areas?
A. Rs. 4800 B. Rs. 3600 C. Rs. 3560
A. 81 : 121 B. 9 : 11
D. Rs. 4530 E. None of these
C. 729 : 1331 D. 27 : 121
Explanation:
Explanation:
Area of the four walls = 2h(l + b)
Ratio of the sides = ³√729 : ³√1331 = 9 :
Since there are doors and windows, area 11
of the walls = 2 * 12 (15 + 25) - (6 * 3) -
3(4 * 3) = 906 sq.ft. Ratio of surface areas = 92 : 112 = 81 :
121
Total cost = 906 * 5 = Rs. 4530

22. The length of a rectangle is two - fifths


19. The radius of a wheel is 22.4 cm. of the radius of a circle. The radius of the
What is the distance covered by the wheel circle is equal to the side of the square,
in making 500 resolutions. whose area is 1225 sq.units. What is the
A. 252 m B. 704 m C. 352 m area (in sq.units) of the rectangle if the
rectangle if the breadth is 10 units?
D. 808 m E. None of these
A. 140 B. 156 C. 175
Answer: Option B
D. 214 E. None of these
Explanation:
Explanation:
In one resolution, the distance covered by
the wheel is its own circumference. Given that the area of the square = 1225
Distance covered in 500 resolutions. sq.units

= 500 * 2 * 22/7 * 22.4 = 70400 cm = 704 => Side of square = √1225 = 35 units
m The radius of the circle = side of the
square = 35 units Length of the rectangle
= 2/5 * 35 = 14 units
20. The volumes of two cones are in the
ratio 1 : 10 and the radii of the cones are Given that breadth = 10 units
in the ratio of 1 : 2. What is the length of Area of the rectangle = lb = 14 * 10 = 140
the wire? sq.units
A. 2 : 5 B. 1 : 5 C. 3 : 5
D. 4 : 5 E. None of these 23. The sector of a circle has radius of 21
Explanation: cm and central angle 135. degree Find its
perimeter?
The volume of the cone = (1/3)*3.14*r^2*h
A. 91.5 cm B. 93.5 cm C. 94.5 cm
Only radius (r) and height (h) are varying.
D. 92.5 cm E. None of these
Hence, (1/3) and pi may be ignored.
Explanation:
V1/V2 = r1^2*h1/r2^2*h2 => 1/10 =
(1)^2*h1/(2)^2*h2 Perimeter of the sector = length of the arc
+ 2(radius)
=> h1/h2 = 2/5
= (135/360 * 2 * 22/7 * 21) + 2(21)
i.e. h1 : h2 = 2 : 5
= 49.5 + 42 = 91.5 cm

pg. 200
10 Seconds

Side = √ (10² +10²) = √200


24.The diameter of Road Roller is 84 cm Area = 200 m²
and its length is 150 cm. It takes 600
revolutions to level once on a particular
road. Then what is the area of that road in 27.A hemispherical bowl of diameter 16cm
m²? is full of ice cream. Each student in a class
A. 2376 B. 2476 is served exactly 4 scoops of ice cream. If
the hemispherical scoop is having a radius
C. 2496 D. 2516 of 2cm, then ice cream is served to how
many students?
Answer: A. 2376
A. 16 B. 32
Area: 600*2*22/7*42/100*150/100 = 2376
C. 64 D. 128
Answer: A. 16
25.A smaller triangle is having three sides.
Another big triangle is having sides exactly Explanation:
double the sides of the smaller triangle.
Then what is the ratio of Area of Smaller 2/3*π*8³ = n*4*2/3*π*2³
triangle to Area of the bigger triangle? n = 16
A. 1:2 B. 2:1
C. 1:4 D. 4:1 28.A hollow cylindrical tube is made of
Answer: C. 1:4 plastic is 4 cm thick. If the external
diameter is 18 cm and length of the tube is
Explanation: 59cm, then find the volume of the plastic?
Smaller triangle sides = a, b, c A. 10380 cm³ B. 10384 cm³
Area = √s(s-a) (s-b) (s-c); C. 10440 cm³ D. 10444 cm³
s = a+b+c/2 Answer: B. 10384 cm³
= √(a+b+c)(b+c-a)(a+c- b)(a+b-c)/4 Explanation:
Bigger triangle =2a, 2b, 2c R = 9, r = 5
Area = √(a+b+c)(b+c-a)(a+c- b)(a+b-c) V = 22/7*59(92 - 52)
Ratio = 1:4 = 22/7*59(81 - 25)
= 10384
26.ABCD is a square of 20 m. What is the
area of the least-sized square that can be
inscribed in it with its vertices on the sides 29.What is the radius of the circle whose
of ABCD? area is equal to the sum of the areas of
two circles whose radii are 20 cm and 21
A. 100 m² B. 120 m² cm?
C. 200 m² D. 250 m² A. 27m B. 28m
Answer: C. 200 m² C. 29m D. 30m
Explanation: Answer: C. 29m
It touches on midpoints on the sides of the Explanation:
square ABCD
πR² = πr1² + πr2²

pg. 201
10 Seconds

πR² = π(r1² + r2²) 3. Only I & IV follow 4. None of


these
R² = (400 + 441)
Explanation:
R = 29

30.A well with 14 m diameter is dug up to


49 m deep. Now the soil taken out during
dug is made into cubical blocks of 3.5m
side each. Then how many such blocks
were made?
A. 22 B. 44
C. 88 D. 176 2.Statements:
Answer: D. 176 1)Some ships are boats.
Explanation: 2)All boats are submarines.
22/7*7²*49 = n*(7/2)³ 3)Some submarines are yatches.
n = 176 Conclusion:
I. Some yatches are boats.
Syllogism II. Some submarines are boats.
DIRECTIONS for questions(1 to 10): In III. Some submarines are ships.
each of the questions below are given
three statements, followed by conclusions: IV. Some yatches are ships
I, II, III, IV. You have to take the given 1. All follow 2. Only II and III follow
statements to be true even if they seem to
be at variance from commonly known 3. Only III follows 4. Only IV follows
facts. Read the conclusions and then
decide which of the given conclusions Explanation:
logically follows from the given statements
disregarding commonly known facts.

1.Statements:
1)Some Cats are Rats.
2)All bats are tables.
3)All Rats are Bats. 3.Statements:

Conclusion: 1)All Carrots are birds.

I. Some Cats are bats II. All bats 2)Some telephones are Carrots.
are rats 3)All bedsheets are telephone.
III. All tables are cats IV. All bats Conclusion:
are cats
I. All bedsheet are birds
1. Only I & II follow 2. Only II
follows II. Some bedsheet are birds

pg. 202
10 Seconds

III. Some birds are telephone IV. All I. All Jalebis are Barfi
telephone are birds
II. All Tikkis are Samosas
1. Only I follows 2. Only II
follows III. All Samosas are Barfi

3. Only I and III follow 4. Only III follows IV. All Barfi are Jalebi
1. Only I and II follow
2. Only I and III follow
3. Only II and III follow
4. All follow
Explanation:

4.Statements:
1)Most CPUs are keyboards.
2)No keyboard is a Mouse.
3)All Mouses are CPU.
Conclusion:
I. Some keyboards are CPU 6.Statements:

II. All CPU’s are Mouse 1)Some eyes are ears.

III. No Mouse is a keyboard 2)Some ears are lungs.

IV. Some Mouse are keyboard 3)All lungs are hands

1. Only I follows 2. Only II and III follow Conclusion:

3. Only I and III follow 4. Only II follows I. Some hands are eyes.

Explanation: II. Some hands are ears


III. Some lungs are eyes
IV. No hand is eye
1. None follow 2. Only IV follows
3. Only II follows 4. Only III follows
Explanation:

5.Statements:
1)Samosas are Jalebi.
2)All Jalebis are Tikki.
3)All Tikkis are Barfi
Conclusion: Or

pg. 203
10 Seconds

III. Some Gold are Silver


IV. No Silver is Gold
1. Only I follows 2. Only III follows
3. Only IV follows
4. Only II and IV follow
Explanation

7.Statements:
1)All liquids are solids.
2)Some solids are gases.
3)All gases are clouds
9.Statements:
Conclusion:
1)Some messages are whatsapp.
I. Some clouds are solids
2)All Hikes are whatsapp.
II. Some clouds are liquids
3)All whatsapp are facebook.
III. Some gases are liquids
Conclusion:
IV. Some solids are clouds
I. Some facebook are message
1. None follows 2. Only I and II
follow II. All hikes are facebook

3. Only III and IV follow III. Some messages are hikes

4. Only I and IV follow IV. Some message are facebook

Explanation: 1. All follow


2. Only I, II and III follow
3. Only I, II and IV follow 4.
Only III and IV follow
Explanation

8.Statements:
1)All Gold are Platinum.
2)No Platinum is silver.
3)Some Diamonds are silver.
10.Statements:
Conclusion:
1)No watch is cycle.
I. Some Diamonds are Gold
2)No cycle is Motorbike.
II. Some Diamonds are Platinum

pg. 204
10 Seconds

3)Some auto are motorbike


Conclusion:
I. No Motorbike is watch
II. No motor bike is cycle
III. Some cycles are watches
IV. All Motorbikes are watches 12. Considering given statements as true,
select a logical conclusion based on the
1. None follows given statements.
2. Only I follows Statements:
3. Only I and III follow Lady's Finger is tastier than cabbage
4. None of these Cauliflower is tastier than Lady's Finger
Explanation : Cabbage is not tastier than peas

a. Peas are as tasty as lady's finger


b. Peas are as tasty as cauliflower and
lady's finger
c. Peas is not tastier than lady's finger
d. Cauliflower is tastier than cabbage
11. Considering given statements as true, e. None of the above options
select a logical conclusion based on the
given statements. Explanation :
Statements: > means tastier
No tree is a flower Cauliflower>Lady's Finger > Cabbage
Some trees are fruits Peas > Cabbage. Peas can be placed
anywhere before cabbage.
Conclusions:
So only d follows.
(I) Fruits that are trees are not flowers.
(II) No fruit is a flower.
13. Considering given statements as true,
select a logical conclusion based on the
a. Only conclusion I follows given statements.

b. Only conclusion II follows Statements:

c. Either conclusion I or II follows Some A are B.

d. Neither conclusion I nor II follows Some C are A.

Explanation :
Conclusions:
(I) Some C are B.
(II) Some B are A.

pg. 205
10 Seconds

a. Only conclusion I follows


b. Only conclusion II follows
c. Either conclusion I or II follows
d. Neither conclusion I nor II follows
Explanation:

15. Considering given statements as true,


select a logical conclusion based on the
given statements.

Statements:
14. Considering given statements as true, Some roads are highways.
select a logical conclusion based on the
given statements. Some highways are seaways.
Statements: Some seaways are airways.
Some cats are animals.
All plants are animals. Conclusions:
All animals are plants. I. Some airways are roads.
II. Some seaways are roads.
Conclusions: III. Some airways are highways.
I. Some cats are plants. IV. Some roads are airways.
II. All plants are animals.
III. All animals are cats. a. Only III and IV follow
IV. All plants are cats. b. Only III and IV follow
c. Only I and II follow
a. Only II and III follow d. All of conclusions follow
b. Only II follows e. None of the above follow
c. Only I, II and IV follow Explanation:
d. Only I and II follow
e. None of the above options

Explanation :

pg. 206
10 Seconds

16. Considering given statements as true, All E are D.


select a logical conclusion based on the
given statements. No D is A.

Statements: Conclusions:

All boots are sandals. I. Some Q are not A.

No sandal is shoe. II. Some Q are not D.

All shoes are rings. III. All Q are D.


IV. No Q is A.

Conclusions:
I. Some boots are rings. a. Only III follows

II. All sandals are boots. b. Either I or IV follow

III. Some rings are sandals. c. Either II or III follow and I follows

IV. All rings are shoes. d. Only IV follows


e. All of the above follow

a. Either I or II follow Explanation:

b. Either III or IV follow For a conclusion to be true, it has to be


true in every possible Venn diagram.
c. Only II and IV follow
From given Venn diagrams, we can say
d. Neither I nor II follow that conclusion I always follows.
e. All of the above follow Also, drawing all possible Venn diagrams
show that satisfying conditions in given
statements, either conclusion II or III will
Explanation: be true. Both cannot be true
simultaneously. It is either one of the two.

18. Considering given statements as true,


select a logical conclusion based on the
17. Considering given statements as true,
given statements.
select a logical conclusion based on the
given statements. Statements:
All mangoes are fruits.
Statements: No fruit is an apple.
Some Q are E. Some apples are vegetables.

pg. 207
10 Seconds

Some fruits are carrots. d. III and IV


Explanation:
Conclusions:
I. No mango is an apple.
II. Some vegetables are not apples.
III. Some mangoes are carrots.
IV. Some vegetables are apples.

a. Only I and IV follow


b. Only I, II or IV follows
20. Four arguments are given. Each is a
c. Either III or IV follow and I follows set of three sentences. Decide which set
d. Either II or IV follows has third sentence as logical conclusion of
first and second sentences considering
e. None of the above options. the first two to be correct.
Explanation:
I. All cats are babies. All babies are young.
All cats are young.
II. Some nerds are fools. Some fools are
green. Some nerds are green.
III. Cars are well-built. Well-built sustains.
Cars sustain.

19. Four arguments are given. Each is a IV. All cyclists are fast. Some fast are
set of three sentences. Decide which set wrestlers. Some cyclists are wrestlers.
has third sentence as logical conclusion of
first and second sentences considering
the first two to be correct. a. Only I
b. II, III and IV
I. Some A are B. All B are C. Some A are c. I and II
C.
d. I and III
II. All men are yellow. No yellow are
women. No men are women. Explanation:

III. No ring is chain. Some chains are


watches. Some rings are watches.
IV. Every P has a Q. All Q are R. No R
has a P.

a. I and III
b. II
c. I and II

pg. 208
10 Seconds

21. Four arguments are given. Each is a IV. All six are seven. No seven is eight. All
set of three sentences. Decide which set six are not eight.
has third sentence as logical conclusion of
first and second sentences considering
the first two to be correct. a. III and IV
b. II and IV
I. God is a human. Some humans are c. I and II
fragile. God is fragile.
d. II
II. Some MBA are employed. All employed
are aliens. Some MBA are aliens. Explanation:
III. All jacks are spades. Some kings are
diamonds. Some kings are spades.
IV. All humans are fragile. God is not a
human. God is not fragile.

a. III and IV
b. II
c. III
23. Six statements are given. Below them
d. I and IV sets of three statements are given. Decide
which set has third sentence as logical
Explanation: conclusion of first and second sentences
considering the first two statements to be
correct.

(A) All apples are pears.


(B) Some pears are mangoes.
(C) Some mangoes are not apples.
(D) All pears are apples.
(E) No mango is a pear.
(F) No apples are mangoes.
22. Four arguments are given. Each is a
set of three sentences. Decide which set
has third sentence as logical conclusion of a. ABF
first and second sentences considering
the first two to be correct. b. FDE
c. BCF

I. No bird can fly. All animals are black. d. EFD


Birds and animals can fly. Explanation:
II. No P is not Q. All R are P. All R are Q.
III. No nut is bolt. No bolt is screw. No nut
is screw.

pg. 209
10 Seconds

(A) No brother is a duck.


(B) No rabbit is hen.
(C) Some ducks like to play.
(D) Some ducks are rabbits.
(E) Some hen are ducks.
(F) All rabbits like to play.
24. Six statements are given. Below them
sets of three statements are given. Decide
which set has third sentence as logical a. FEB
conclusion of first and second sentences
considering the first two statements to be b. FDC
correct.
c. EDB
d. BED
(A) Some cleaners are janitors.
Explanation:
(B) All cleaners are sweeper.
(C) No sweeper is cleaner.
(D) Some janitors are not cleaner.
(E) Some sweepers are not janitors.
(F) Some cleaners are janitors.

a. BDE 26. Six statements are given. Below them


b. CEF sets of three statements are given. Decide
which set has third sentence as logical
c. FCD conclusion of first and second sentences
considering the first two statements to be
d. ABE correct.
Explanation:

(A) No cake is sweet.


(B) All candy are sweet.
(C) All onions are candy.
(D) No onion is a cake.
(E) No cake is a candy.
(F) Onions are sweet.

25. Six statements are given. Below them


sets of three statements are given. Decide a. FDA
which set has third sentence as logical
conclusion of first and second sentences b. CEF
considering the first two statements to be
correct. c. ABE
d. EFD

pg. 210
10 Seconds

Explanation:
Conclusions:
(I) All house being door is a possibility.
(II) At least some house is a window.

a. Only conclusion I follows


b. Only conclusion II follows
27. Considering given statements as true,
select a logical conclusion based on the c. Either conclusion I or II follows
given statements.
d. Neither conclusion I nor II follows
Explanation:
Statements:
All A are B
Some B are C

Conclusions:
(I) Some C are A.
(II) All A being C is a possibility.
29. Considering given statements as true,
select a logical conclusion based on the
given statements.
a. Only conclusion I follows
b. Only conclusion II follows
Statements:
c. Either conclusion I or II follows
All elephants are rat.
d. Neither conclusion I nor II follows
No rat is a dog
Explanation:
All dogs are lions

Conclusions:
(I) No lion is elephant.
(II) All rats being lions is a possibility.
(III) All elephants being lions is a
28. Considering given statements as true, possibility
select a logical conclusion based on the
given statements.
a. Only conclusion I and III follows

Statements: b. Only conclusion II follows

Some doors are window. c. Only conclusion II and III follows

No window is a house d. Neither conclusion I nor II follows

pg. 211
10 Seconds

e. All follow Cryptarithmetic


Explanation: Instructions for Cryptarithmetic
Problems
1)Alphabets can take up only one distinct
value.
2)Alphabets can only take up values
between 0-9.
3)Decoded numbers can’t begin with 0, for
example, 0813.
30. Considering given statements as true, 4)Problems are uni-solutional.
select a logical conclusion based on the
5)19 is the max value with a carryover for
given statements.
two one-digit number in the same column
addition
Statements: 6)Carryover can only be 1 for addition
problems
No stone is metal.
Some metals are papers.
Questions:
All papers are glass.
Q.1 HERE = COMES – SHE (Assume
S=8). Find the value of R + H + O?
Conclusions: a)12 b)14
(I) Some metals being glass is a c)15 d)18
possibility.
Ans: Option 2
(II) At least some glasses are metals.
Explanation: Another way of writing HERE
(III) No stone is paper + SHE = COMES is

a. Only conclusion I and II follows HERE


b. Only conclusion II follows + SHE
c. Only conclusion II and III follows ---------
d. Only I follows COMES
e. All follow Now let us assume C=1, O=0, H=9,
Explanation: E+E=S=8,2 E=8, & E= 4. So, COMES –
SHE = HERE, 9454 + 894 = 10348.
R + H + O = 5 + 9 + 0 = 14. So the final
answer is 14.

Q.2 If AA + BB + CC = ABC. Then what is


the value of A + B + C = ?
a)21 b)15
c)18 d)12

pg. 212
10 Seconds

Ans: Option 3. MAC


Explanation: AA + BB + CC = ABC can +MAAR
also be written as,
-----------
JOCKO
AA
In the question J is carry so, let us
+ BB consider J = 1, O = 0 with carry 1 and M =
9, C + R = O, 0 with carry 1. Therefore C =
+ CC 2 and R = 8, M + A = C,2 with the carry 1.
----------- A = 3, A + A + 1 = K, Now put value of A in
A + A + 1 = K. So the equation becomes,
ABC 3 + 3 + 1 = K i.e K = 7.
Here, the digits are positive and distinct. 932 + 9338 = 10270 So, the value of A =
So, let’s focus on value A. If we add 3 two- 3, M = 9 and C =2, After putting values in
digit numbers then most of you get the equation 3A + 2M + 2C = 9 + 18 + 4 =
answers in 200s. For example: AA + BB + 31. So the answer is 31.
CC (99 + 88 + 77 = 264) so the possible
largest value of A can be 2. Either value of
A will be 1 or 2 (A=2 or A=1). Q.4 SEND + MORE = MONEY, Now find
We get the value of A, Now let’s focus on the value of M + O + N + E + Y?
the value of B. Let’s take the unit digit a)14 b)15
value of the given question. A + B + C = C
(units). So, this can happen only if A + B = c)16 d)17
0 (units) A & B add up to 10.
Ans: Option 1.
There are two possibilities: 22 + 88 + CC
= 28C (1) and 11 + 99 + CC = 19C (2). Explanation: SEND + MORE = MONEY
can also be written as,
Now take equation number 1, 110 + CC =
28C. Let us focus on ten’s place, 1 + C = 8
where, C=7 then the equation becomes, SEND
22 + 88 + 77 = 187. So the equation 1 is
not the possible solution. +MORE
Now take equation number 2, 11 + 99 + —————–
CC = 19C, 110 + CC = 19C. 1 + C =9
MONEY
where, C=8 then the equation becomes,
11 + 99 +88 = 198, Hence solved. Here, the only possible value of M is 1
Therefore, A = 1, B = 9 and C = 8, A + B + because it’s only the carry-over possible
C = 18. The answer is 18. from the sum of 2 single-digit numbers in
column 4. So, M = 1, S + 1 = two-digit
number. Therefore S = 1 and O can be 0
Q.3 MAC + MAAR = JOCKO, Now find the but not 1.
value of 3A + 2M + 2C?
Here E and N are consecutive. So, with
a)33 b)38 the hit and trial possibility, we get SEND =
9567, MORE = 1085, and MONEY =
c)31 d)36 10652.
Ans: Option 3. i.e M + O + N + E + Y = 1 + 0 + 6 + 5 + 2 =
Explanation: MAC +MAAR = JOCKO can 14. So, we get the answer as 14.
be written as,

pg. 213
10 Seconds

Q.5 NO + GUN + NO = HUNT, Now find After adding only two single digits
the value of HUNT? numbers, the maximum carry we can get
is 1. Therefore the value of G is 1 i.e G=1.
a)1028 b)1082 Consider the equation O + A = A and from
c)1280 d)2180 this equation, we can say that O = 0. T +
N = O ( O must be a two-digit number that
Ans: Option 2. will end with 0, then only the value of G
will be 1).
Explanation: NO + GUN + NO = HUNT

The sum of T & N should be 10. After


NO
substituting the values equation becomes,
+ NO T(6) + N(4) = 10. From the equation M + G
= T we get value of M as 1 i.e M = 1. So,
+ GUN we get values as O = 0, G = 1, N = 4, M=
----------- 5, T = 6 and the left out numbers are 2, 3,
7, 8, 9. Therefore A can take any value
HUNT from this. A don’t have any definite value.
So the answer is cannot be determined.
Now, consider the value of H as 1 and
from the column NUNN we must have
carry 1 so, G = 9 and U = 0. We have
carry as 0, 1, or 2 from the column ONOT. Q.7 FORTY + TEN + TEN = SIXTY, Now
Similarly, we’ve N + U = 8, 9, or 10, but find the value of T + E + N?
duplication is not allowed. Therefore N = 8 a)64 b)24
with the carry 2 from column ONOT.
Hence, O + O = T + 20 – 8 = T + 12. Now c)22 d)21
after doing testing for T = 2, 4 or 6 only T
Ans: Option 3.
= 2 is acceptable, O = 7.
Explanation: FORTY + TEN + TEN =
Hence, we’ve 87 + 908 + 87 = 1082. So,
SIXTY
we get the answer as 1082.

F O R T Y
Q.6 TOM + NAG = GOAT Now find the
value of G + O + A + T? + T E N
a)17 + T E N
b)25 —————————-
c)27 S I X T Y
d)Cannot be Determined Consider the rightmost column where we
get an equation as Y + N + N = Y, where
Ans: Option 4.
N = 0. Now take the next column T + E +
Explanation: TOM + NAG = GOAT E = T, in which T should not be 0 but E +
E, if we get, the sum of ten then, 10 + T
we get unit digit as T. Therefore, 2E = 10
TOM and E = 5.

+ NAG The letter O must have carry then only


we’ll get the value of I i.e O + carry = I.
----------- The value of I should be a two-digit
number because the leftmost column
GOAT needs a carry (F + carry = S) then only
we’ll get the value of S.

pg. 214
10 Seconds

Explanation: YOUR + YOU = HEART can


be written as,
The addition of O + carry = I, here the
value of I should be a two-digit number. If
we want to get the value of I as 2 digit
number then, O must have to take the YOUR
value as 9. Let the value of carry be 1 (9 + + YOU
1 = 10). Here the value of I is a unit digit,
but I should not take the value as 0 -----------
because N has already taken the value 0.
HEART
Therefore we’ve to keep the carry as 2 so,
9 + 2 = 11 then the unit digit 1 is the actual In the question itself, the value of O is
value of I. given as 4 i.e O = 4. If you see in the
question the value of Y and E cannot be
the same, that’s why there should be a
Here the ones digit will be the carry for the carryover 1. Now consider the equation 1
next column i.e F (F + 1 = S). + Y = 10, here E will take the value as 0
and H will take 1. O + Y = A (4 + 9 = 13)
i.e A = 3.
Now, R + T + T + carry = X ( the addition
of these three numbers must give you the
value in the range between the 20s Here, U + O(4) = R (only a single-digit
because there is carry to the next column). number), but U cannot take the value like
Therefore R & T must be taken as the 5, 3, 4 so the only possible value of U will
maximum value to get a number in the be 2. And if the value of U is 2 then the R
range of 20s. will become 6.

Now, let the value of R = 7 and T = 8. R + After substituting all the values in R + U =
T + T + carry = 7 + 8 + 8 + 1 = 24. Then T, 6 + 2 = 8 where T = 8 we get Y + U + R
the equation becomes T + E + E = T ( 2 + E = 9 + 2 + 6 + 0 = 17. Therefore we get
digit number ) i.e 8 + 5 + 5 = 18. answer 17.

Here, the value of O = 9, T = 8, R = 7, E = Q.9 If EAT + THAT = APPLE, then what is


5, X = 4, I = 1 & the left out numbers are 6, the sum of A + P + P + L + E?
3, 2. Now, we know that F + 1 = S i.e 2
a)12 b)13
and 3 will be taken by F and S. Therefore
at last, the value of Y will be 6. c)14 d)15
Ans: Option 1.
After substituting the values in S + I + X +
T + Y = 3 + 1 + 4 + 8 + 6 = 22. So, the
answer we get is 22. Explanation: EAT + THAT = APPLE

Q.8 YOUR + YOU = HEART (Assume O = EAT


4). Now, find the value of Y + U + R + E? +THAT
a)18 b)15 -----------
c)17 d)16 APPLE
Ans: Option 3.

pg. 215
10 Seconds

From the given data we’ll get the value of values of G, R and I as 8, 7, 6 or 7, 6, 5
A as 1 because it’s only the carryover etc and check trial and error possibilities.
possible number from the sum of two We get POINT = 98504, ZERO = 3168
single-digit numbers. T can take only and ENERGY = 101672 so, E + N + E + R
maximum number 9 and there must be a + G + Y = 1 + 0 + 1 + 6 + 7 + 2 = 17.
carryover for T to get the sum as 2 digit Hence we get the answer as 17.
number.

Q.11 USSR + USA = PEACE, then what is


Here we get T = 9, P = 0, A = 1. T + T = the value of P + E + A + C + E?
18 and the value of E = 8 and 1 will be a
carryover to the next column. So, 1 + A + a)9 b)10
A = L = 3 and finally the value of H c)11 d)12
becomes 2 i.e H = 2.
Ans: Option 2.
Explanation: USSR + USA = PEACE
Hence, 819 + 9219 = 10038. Now
substitute the values of A, P, L and E in A
+ P + P + L + E then we get, 1 + 0 + 0 + 3
USA
+ 8 = 12. Therefore, the final answer is 12.
+USSR
-----------
Q.10 If POINT + ZERO = ENERGY, then
E+N+E+R+G+Y=? PEACE
a)16 b)17 Here, we can clearly see that the value of
P = 1, U = 9, and E = 0 so first replace all
c)18 d)19
the values in the equation. Now consider
Ans: Option 2. the last column A + R = 0, This is possible
only if A = 5 and R = 5. but both the values
Explanation: POINT + ZERO = ENERGY cannot take the same values. Therefore
it’s possible only with (8, 2), (7, 3), (6, 4),
(3, 7), (2, 8). Here we’ve to implement the
POINT hit and trial method and take (2, 8) and
replace it with the new values.
+ ZERO
-----------
Now we’ll assume value of S to be 3 i.e S
ENERGY = 3. We’ve S + S + 1 (carry) = C, After
From the data the value of E = 1 because substituting the values in S + S + 1 = C
it is the only carryover possible from the becomes 3 + 3 + 1 = C i.e C = 7 which is
sum of 2 single-digit numbers. Now unoccupied. Here all the values are
consider the values of E = 1, P = 9 and N founded but we just need to verify.
= 0, N + R = G, 0 + R = G. Here R = G is
could not be possible but 1 + R = G is
possible. Here G & R are consecutive, G > Here the values are replaced and all the
R, 1 + I = R that’s why I and R are operations are working fine. So, the values
consecutive. of P + E + A + C + E = 1 + 0 + 2 + 7 + 0 =
10. So we get the answer as 10.

Now, O + T should give carryover & O + Z


can also give a carryover. So, here O is Q.12 TO + GO = OUT, Then find the value
the biggest number. Now consider the of T + G + O + U?

pg. 216
10 Seconds

a)11 b)12 Let us say A can take a maximum value of


6.
c)13 d)14
Then 6! = 720 but we have taken
Ans: Option 1. maximum value is A is 6.
Explanation: TO + GO = OUT Therefore, the Maximum value of the
digits is less than 6.

TO So A should take 5. Then 5! = 120

+GO Now from the above we know that one the


B and C should take 1 as their value as
----- 120 consists of 1.
OUT ⇒
Here the value of O is clearly 1 but T + G 5! + 1! = 121
is generating O which is carry so the value
of O is 1. After substituting the values T = If we take 4 as one of the number then 5!+
1 + 1 we get the value of T as 2 i.e T = 2. 1! + 4! = 145 or 1! + 4! + 5! = 145
Now 2 + G is greater than 10 which is Sum = 1 + 4 + 5 = 10
resulting in the carry 1 on next. Here, the
possible values of G to get carry at the
next step is 8 or 9.
14. Find the value of TEN if
ONE+ONE+ONE+ONE=TEN
If the value of G is 9 then, U = 2 + 9 = 11. a. 11 b. 12
So, the value of U becomes 1, and 1 goes
to the carry. Here, the value of O is c. 15 d. 17
already 1 so, the value of U cannot be 1. Explanation:
Now, the value of G has to be 8. U = 2 + 8
= 10. So, the value of U becomes zero O=1,E=8,N=2,T=7
and there are no conflicts.

Therefore, the final values are T = 2, O =


15.If N O + G U N + N O = H U N T, find
1, G = 8 and U = 0. After substituting
the value of HUNT.
these values in T + G + O + U = 2 + 8 + 1
+ 0 we get the answer as 11. A. 1082 B. 1802
C. 1208 D. 1280
13. Find the sum of the values of A, B and Explanation:
C if
Here H = 1, from the NUNN column we
ABC=A!+B!+C! must have “carry 1,” so G = 9, U = zero.
Since we have “carry” zero or 1 or 2 from
where ABC is a three digit number
the ONOT column, correspondingly we
a. 7 b. 8 have N + U = 10 or 9 or 8. But duplication
is not allowed, so N = 8 with “carry 2” from
c. 9 d. 10 ONOT. Hence, O + O = T + 20 – 8 = T +
Answer: D 12. Testing for T = 2, 4 or 6, we find only T
= 2 acceptable, O = 7. So we have 87 +
Explanation: 908 + 87 = 1082.
By symmetry, we take A is the maximum
number of the three alphabets.

pg. 217
10 Seconds

16.If “EAT + THAT = APPLE”, what is the


sum of A+P+P+L+E?
A. 13 B. 14
C. 12 D. 15
Explanation:
From the given data, the value of A will be
1 because it is the only carry-over possible
from the sum of 2 single-digit number. T
maximum it can take only 9 and there
should a carryover for T to give sum as 2
digit number. So T =9, P = 0, A = 1. T + T
= 18, the value of E is 8 and 1 will be a
carry over to the next column. That is 1 +
A + A= L = 3. And finally H = 2. Hence,
819 + 9219 = 10038. A+P+P+L+E =
1+0+0+3+8 = 12.

pg. 218

You might also like